全部 1- 101- 201- 301- 401- 501- 601- 701- 801- 901- 1001- 最新50


面白い問題おしえてーな 九問目
1 名前:132人目の素数さん:04/08/28 15:55
過去スレ

[1]面白い問題教えて
http://cheese.2ch.net/math/kako/970/970737952.html
[2]面白い問題教えて 第2版
http://natto.2ch.net/math/kako/1004/10048/1004839697.html
[3]面白い問題おしえてーな
http://science.2ch.net/math/kako/1026/10262/1026218280.html
[4]面白い問題おしえて〜な 四問目
http://science.2ch.net/math/kako/1044/10441/1044116042.html
[5]面白い問題おしえて〜な 五問目
http://science.2ch.net/math/kako/1049/10495/1049561373.html
[6]面白い問題おしえて〜な 六問目
http://science.2ch.net/test/read.cgi/math/1057551605/
[7]面白い問題おしえて〜な 七問目
http://science2.2ch.net/test/read.cgi/math/1064941085/
[8]面白い問題おしえてーな 八問目
http://science3.2ch.net/test/read.cgi/math/1074751156/l50

2 名前:132人目の素数さん:04/08/28 16:29
2げっと

3 名前:132人目の素数さん:04/08/28 17:06
>1
乙。

4 名前:132人目の素数さん:04/08/28 17:30
10組の夫婦がいます。
いま、ある円卓に対して、妻が自分の夫と隣合わないように座る方法は何通りあるでしょう。
ただし、円卓の各席には1〜20までの番号が付いているものとします。

5 名前:132人目の素数さん:04/08/28 17:56
全ての座り方は20!通り。
妻が自分の夫と隣合うように座る方法は、
(妻が座る場所の数)×(妻の両隣)=20*2=40
よって、
20!-40=2432902008176639960通り。

って、桁が大きいことを除けば高校生レベルのような気が・・・

6 名前:132人目の素数さん:04/08/28 17:57
>>5
ダウツ

7 名前:132人目の素数さん:04/08/28 18:17
夫婦が隣り合う座り方は2*2*10!通りでは?


8 名前:132人目の素数さん:04/08/28 18:26
プロブレム・ド・メナージュ

9 名前:132人目の素数さん:04/08/28 18:34
>>7
ダウツ

10 名前:132人目の素数さん:04/08/28 18:34
>>7
10!だと「全ての夫婦が隣り合う場合」を意味しているよ。
余事象で考えると「少なくとも1組の夫婦が隣同士になる場合」を引かなければならない。

11 名前:132人目の素数さん:04/08/28 22:04
Retunner金城武age

12 名前:132人目の素数さん:04/08/28 22:24
Returnerじゃね?

13 名前:132人目の素数さん:04/08/28 22:38
とりあえずn=10として
納k=0,n]C[n,k](2n-k)!2^k-2・2納k=0,n-1]C[n-1,k](2n-2-k)!2^k
みたいだけど。もっと簡単になるのかな?

14 名前:132人目の素数さん:04/08/28 22:43
>>4
頂点数20,次数18の正則グラフにおけるハミルトン閉路の数,か.
めんどいな.

15 名前:132人目の素数さん:04/08/28 23:04
まちごうた。
とりあえずn=10として
納k=0,n]C[n,k](2n-k)!(-2)^k-2・2納k=0,n-1]C[n-1,k](2n-2-k)!(-2)^k
だった。

16 名前:132人目の素数さん:04/08/28 23:32
とりあえず前スレでのこってた香具師。
自然数nに対しその2進展開をn=納i=0,∞]c_i(n)2^iとする。c(n)=納i=0,∞]c_i(n)とおく。
(0でないのは有限個なのでwell-defined)。このとき数列c(n)は同一のパタ−ンが3回
くりかえさないことを示せ。つまり非負整数nと正の整数kで
c(n+i)=c(n+k+i)=c(n+2k+i) (0≦∀i≦k-1)をみたすものは存在しないことをしめせ。
↑これだれかでけた?

17 名前:132人目の素数さん:04/08/29 00:03
しもた。訂正。c(n)=納i=0,∞]c_i(n)を2でわった余りとおくだ。
つまりc(n) (0≦n)は
0110100110010110・・・
みたいな。(全スレの出題とは0と1が逆だけど実質同じ。)

18 名前:132人目の素数さん:04/08/29 00:20
>>11-12
映画の事ですか?
円卓に座るシーンがあるとか。

>>14
高校数学の範囲では解けませんか?

19 名前:132人目の素数さん:04/08/29 01:51
前スレ見た。
有名問題も多いが、一般的に云って、多くの人が問題の意味を理解できる
パズル的問題が多いな。それも知らない人は多いから、結構長い間かかずらわっていて、
実質的な問題の数は多いとはいえない。
しかし高校生でも理解できる数オリ的問題も少なかったな。
より数学的な問題(例えば定理を使って解く)様な問題は
http://www.iis.it-hiroshima.ac.jp/~ohkawa/math/math_problem.htm
に 250 題ほどあって、約 2/3 には解答が付いていないから面白いぞ。
ただし、問題自身が良く分からなかったり、定式化が不十分なものも多いが。

その中から解答の付いていつもの、いないものをそれぞれ一題。
問題214) 任意の有理数は(高々)3つの有理数の立方の和になる。(有名問題 解答)
問題9) 閉区間 [0, 1] から [0, 1] への連続関数 f で、 f (0) = 0, f (1) =1 
且つ、区分的線形関数である物(1変数の場合は、グラフが折れ線になる関数の事である。)
全体の集合を X とする。この時、 X に属する任意の 2 元 f, g を取ると、
X に属する適当な 2 元 p, q で、恒等式 f (p(t)) = g (q(t)), t ∈ [0, 1]
が成立する物が取れる事を示せ。(新作問題、解答・準備中)

20 名前:132人目の素数さん:04/08/29 03:21
関数電卓機能もなく、メモリ機能もない至極単純な電卓で、
Σ[n = 0 〜 10] 1/n! 等を紙にメモする事もなく
電卓のキー操作だけで行うとしたら、
最低何回キーを押さねばならないか?
(あふれ誤差は除く。)

21 名前:132人目の素数さん:04/08/29 03:48
>>20
「等」ってなんだよ。
どう答えりゃいいんだよ。

22 名前:132人目の素数さん:04/08/29 05:08
>>4
Sp+1=Sp(p(p-1)2),S4=1 ?

23 名前:132人目の素数さん:04/08/29 05:22
>>4
S2m=(S2(m-1))(2(m-1)-1)2,S4=1
S20=S18(17)2=S16(17*15)2^2=...
=(17!/9!)2^8 ?

24 名前:132人目の素数さん:04/08/29 05:25
>>21
関数電卓機能がない、
即ち e を一発で計算できないという意味だよ。
「関数電卓」は知っているよな。
Windows にも付属している。

25 名前:132人目の素数さん:04/08/29 05:26
=(17!/9!)2^8/2^9=(1/2)17!/9! ?


26 名前:132人目の素数さん:04/08/29 06:30
>>20
色色。


27 名前:132人目の素数さん:04/08/29 06:35
>>16>>17
こういう定式化ができたんですね

でも解けないなあ・・・

28 名前:132人目の素数さん:04/08/29 07:22
(問題)
XY平面上に、面積 S の、内部を含む、有界で凸な図形 F が存在する。
直線
x cos(θ) + y sin(θ) = r
と F が交わるとき、その共通部分の線分の長さを L(θ,r) とし、
交わらないとき、L(θ,r) = 0 とする。
このとき、
∫[0,2π] ∫[0,∞] {L(θ,r)}^3 dr dθ = 3S^2
となることを証明せよ。

29 名前:132人目の素数さん:04/08/29 08:49
x cos(θ) + y sin(θ) = r
は(cos,sin)と距離rのところで直行する直線
∫[0,2π] ∫[0,∞] {L(θ,r)}^3 dr dθ = 3S^2
はその直線を平行にずらしてFをスキャンすることを
θをまわしやる操作




30 名前:132人目の素数さん:04/08/29 09:38
>>26
具体的に

31 名前:132人目の素数さん:04/08/29 16:51
【問題】

A,Bの二人がじゃんけんをして、グーで勝てば3歩、チョキで勝てば5歩、パーで勝てば6歩進む遊びをしている。
Bがグー、チョキ、パーを出す確率はすべて等しいとする。1回のじゃんけんで、AがBより前に最も進むことが期待されるのはAがどのような確率でグー、チョキ、パーを出すときか。またn回じゃんけんをするときのそれぞれの確率も求めよ。

32 名前:132人目の素数さん:04/08/29 17:00
>>31
勝つ確率も負ける確率もあいこの確率もそれぞれ1/3なので
パーを出し続ければいい

33 名前:132人目の素数さん:04/08/29 17:07
>>32 よく考えてください。パーを出して負けたときは相手が5歩進むんですよ。ちなみに数式を使って解いたほうが分かりやすいです。

34 名前:132人目の素数さん:04/08/29 17:10
じゃあチョキを出し続ければいい

35 名前:132人目の素数さん:04/08/29 17:13
>>34 GJ!!

36 名前:132人目の素数さん:04/08/29 17:44
>>28>>29
積分幾何の本(栗田)に書いてあったような。

37 名前:132人目の素数さん:04/08/29 17:56
お小遣いのもらい方で

・・・あ〜やっぱやめた

38 名前:132人目の素数さん:04/08/29 18:49
(3pg+5qc+6rp)(1/3G+1/3C+1/3P)
A=1pgG+5/3qcP+2rpG
B=1qcG+5/3rPC+2pgP
A>B
(1p+5/3q+2r)>(1q+5/3r+2p)
-p+1/3r+2/3q>0
p+q+r=1,p,q,r>=0
4/3r+5/3q>1
r>3/4-5/4q



39 名前:132人目の素数さん:04/08/29 22:01


 室伏が金確定!



40 名前:132人目の素数さん:04/08/29 22:20
>>30
具体的に。


41 名前:132人目の素数さん:04/08/29 22:43
ハリ麻酔でやれば、ドーピングはかからなかったのに

42 名前:132人目の素数さん:04/08/29 23:47
項数nの多項式を2乗したとき、その項数が最小になる多項式の項数をQ(n)とおく。
例えば、Q(1)は項数1の多項式、ax^kを二乗したとき、a^2x^2kになるので、Q(1)=1である。
同様に、Q(2)=3が得られる。このようにして、Q(n)を定義したとき、
liminf Q(n)/n
を求めよ。


すげー意外な結果になったので、報告する。

43 名前:132人目の素数さん:04/08/30 00:18
Q(n) は、(n-1)次の多項式を自乗した式の(次数+1)に等しいから、
Q(n) = 2(n-1)+1 = 2n-1
よって、
liminf Q(n)/n = 2

44 名前:132人目の素数さん:04/08/30 00:29
説明が不足しておりました。申し訳ありません。
t^4 + 2t^3 - 2t^2 + 4t + 4
のような式を考えたとき、これを二乗すると
16 + 32t + 28t^4 + 4t^7 + t^8
と、なりますので、n=5の時、Q(n)≦5となります。

45 名前:132人目の素数さん:04/08/30 00:31
ある、項が0になるとき、それはカウントしません。
0以外の項を数えるというルールです。

従って、1,2ぐらいまでは簡単にQ(1)=1、Q(2)=3が得られますが、
それ以降になると、Q(n)の値を実際に求めることが困難になると思われます。

46 名前:132人目の素数さん:04/08/30 00:40
>>45
ヒントおながいしまつ。

47 名前:132人目の素数さん:04/08/30 00:43
g(n)=Q(n)/nとして、
g(mn)≦g(m)g(n)
g(n)≦3/2
g(4n+1)≦28/29
を示しちゃおう。

48 名前:132人目の素数さん:04/08/30 01:10
>>47
大きいほうからの評価はいいとして小さいほうからの評価は?
つまり一般項≦××からliminf≦××がでたとしてもそれが
liminfにぴったり等しいことはどうやってしめすの?

49 名前:132人目の素数さん:04/08/30 01:18
ま、そっちはヒント抜きでやってくれ

50 名前:132人目の素数さん:04/08/30 01:21
結論として答えは7/29になるの?てかホントに下からの評価できてるの?
なんかぁゃιぃ

51 名前:132人目の素数さん:04/08/30 01:25
あ、いやなるほど。勘違い。スマン。

52 名前:132人目の素数さん:04/08/30 01:36
>>42の答えは0

53 名前:132人目の素数さん:04/08/30 01:37
>>52
証明おながいしまつ。

54 名前:132人目の素数さん:04/08/30 02:36
>>28>>36
答えきぼんアゲ

55 名前:132人目の素数さん:04/08/30 10:23
>>52
証明きぼんぬ

56 名前:132人目の素数さん:04/08/30 13:51
Q(n)を与える多項式fn(x)とQ(m)を与える多項式fm(x)を考えて
fn(x^k)も同じ性質を持つからfn(x^k)fm(x)がちょうどQ(n)Q(m)になるようにkを調節できる
だからQ(nm)≦Q(n)Q(m)でいい?
でQ(29)≦28が示せるとg(29^k)≦(28/29)^kだから0

57 名前:132人目の素数さん:04/08/30 16:36
>でQ(29)≦28が示せると

つまり、こういうことだな。
f(x)=1 + 2x - 2x^2 + 4x^3 + 6x^4 + 4x^5 - 4x^6 + 8x^7 + 6x^8 - 4x^9 + 4x^10 - 8x^11 - 4x^12 + 8x^13
- 8x^14 + 16x^15 + 6x^16 - 20^17 + 20x^18 - 40x^19 - 12x^20 + 56x^21 -
56x^22 + 112x^23 + 28x^24 - 168x^25 + 168x^26 - 336x^27 - 336x^28
が29項の多項式で、その2乗が28項と。

それでOKなんだな。

58 名前:132人目の素数さん:04/08/30 16:54
>>56-57
なるほど。すばらすぃ。

59 名前:132人目の素数さん:04/08/30 16:57
で、Q(n)のnによる表示は?

60 名前:132人目の素数さん:04/08/30 17:16
>>59
もうliminf=0までしめせてるのでは?

61 名前:132人目の素数さん:04/08/30 17:17
もう少し頑張れば、lim=0が示せそうだが。
Q(n)の具体的表記は得られそうもない。

62 名前:132人目の素数さん:04/08/30 17:51
>>28
できたぜ。所与の領域をDとするとDは曲線Cで囲われたJordan領域。
Cはsmoothな曲線でかつその囲う領域が凸になるものでいくらでも近似できるので
最初からCはsmoothとして一般性をうしなわない。I=∬[C×C]l^3drdθはストークスの
定理から∬[D×C]3t^2dtdrdθに等しい。ただしt:D×C→Rは(p,q)∈D×Cにp,q間の距離
を与える関数。p∈Dを固定して∫[{p}×C3t^2dθはpによらない定数3Sだからそれを
D上で積分すればその値は3S^2。□

63 名前:132人目の素数さん:04/08/30 17:53
あとオレ的に気になるのは16-17かな?これどうやんだろ?

64 名前:132人目の素数さん:04/08/31 00:50
>>16-17
できた。そのような(n,k)が存在すると仮定する。そのなかでkが最小のものをとってくる。
するとkは奇数。なぜならkが偶数なら([n/2],k/2)も条件をみたすのでkの最小性に矛盾。
kの2進展開の桁数をuとする。まず[n,n+2k-1](=最初の2つのブロック)内の整数は
2進展開の末尾u桁が0〜2^u-1までのすべての自然数が出現することに注意する。
(i)k+1が2べきでないとき。
このときkの2進展開のあるv桁目が1でv+1桁目が0である0≦v≦u-1がとれる。
このとき2進展開のu桁目以下の部分がすべて0であるx∈[n,n+2k-1]と
u桁目以下の部分がv桁目以外が0でv桁目が1であるy∈[n,n+2k-1]をとれば
c(x)≠c(x+k) (mod 2)であるかc(y)≠c(y+k) (mod 2)であるゆえ矛盾。
(ii)k+1が2べきのとき。
kが1のときは容易。k≧3と仮定してよい。a∈[n,n+2k-1]で2進展開の末尾u桁がすべて
0であるものをとる。[n,n+2k-1]には6つ以上の整数があるゆえaの前後3つ以内にある2数
x,yでu+1桁め以降がすべて等しくかつ(xの末尾u桁、yの末尾u桁)=(2,3) or (2^u-2、2^u-1)
となるものがとれる。
するとc(x)≠c(x+k) (mod 2)であるかc(y)≠c(y+k) (mod 2)であるゆえ矛盾。□

65 名前:132人目の素数さん:04/08/31 01:13
>>56
>g(29^k)≦(28/29)^kだから0

29のk乗で無限に飛ばせば0かもしれんが、
それ以外の形で無限に飛ばしたらどうなるかは言えてないんじゃないか?

66 名前:132人目の素数さん:04/08/31 01:31
>>65
いやいけてるとおもうよ。
liminf[n→∞]g(n)=lim[n→∞]inf{g(k) | k≦n}
だから非負項数列g(n)に対して
liminf[n→∞]g(n)=0をいうにはi→∞でni→∞なる数列niでlim[i→∞]g(ni)=0
となるものをひとつでもみつけてくればいい。もっというならそのような場合inf{g(k) | k≦n}
が任意のnに対して0になってしまう。

67 名前:132人目の素数さん:04/08/31 01:32
まちがった。
>inf{g(k) | k≦n}
は全部inf{g(k) | k≧n}だった。


68 名前:132人目の素数さん:04/08/31 08:02
>>4
P[2n,k]を2n人いて,k組の夫婦が隣合う場合の数とすると

P[4,0]=2
P[4,1]=0
P[4,2]=4
P[2n+2,k] = 2(2n-k+1)*P[2n,k-1] + { (2n-k)(2n-k-1) + 2k }*P[2n,k] + 2(k+1)(2n-k-1)*P[2n,k+1] + (k+1)(k+2)*P[2n,k+2]
ただしP[2n,k] = 0 for k<0 or n<k

という漸化式がたつ.すると,およそP(20,0) ≒ 4.13 × 10^16

69 名前:132人目の素数さん:04/08/31 08:45
>>57
この式はどうやって見つけたのか教えて頂けないですか?

70 名前:132人目の素数さん:04/08/31 11:43
g(n)=1であるnが無数に存在することが示せないものかな

71 名前:132人目の素数さん:04/08/31 14:48
>>70
>>42>>47のg(n)?つまりlimsup[n→∞]g(n)≧1じゃないかってこと?
なんとなくlimsupのほうも0にいきそうな悪寒。

72 名前:132人目の素数さん:04/08/31 15:34
まだ解答でてないやつ。
Nを2以上の整数とするときΠ[k=1,N-1]Γ(k/N)を求めよ。

73 名前:132人目の素数さん:04/08/31 19:05
>>72
ヒント:Γ(a)Γ(1-a)を計算せよ。

74 名前:132人目の素数さん:04/08/31 22:48
Γ'(1)を求めよ。

75 名前:FeaturesOfTheGod ◆UdoWOLrsDM :04/08/31 22:50
Re:>74 真面ですか?

76 名前:132人目の素数さん:04/08/31 23:18
いろいろカキコしたけどこのスレが一番面白かった
短い間だったけど2ch数学板これからも頑張ってね
じゃサヨナラ

77 名前:132人目の素数さん:04/09/01 01:33
>>75
ダッシュみえてる?まあついててもそんなに難しくないけど。

78 名前:132人目の素数さん:04/09/01 03:27
関数f:[0,1]→Rが次の性質を持つとする:
∀x∈[0,1], ∃ε>0, ∀y,z∈(x-ε,x+ε)∩[0,1], f(y)=f(z).
このとき,
∀x,y∈[0,1], f(x)=f(y)
だろうか?

79 名前:132人目の素数さん:04/09/01 03:35
>>78
どうみてもYesにみえる。

80 名前:132人目の素数さん:04/09/01 03:37
また,[0,1]を(0,1)とした場合はどうか?

81 名前:132人目の素数さん:04/09/01 03:47
>>80
それでもYesであるような・・・ちがうの?

82 名前:132人目の素数さん:04/09/01 04:13
x=0を中心として[0,a_1)までfは定数になる.
次に,x=a_1ととって[0,a_2)まで定数になる.
このように{a_n}を定義したとして,果たしてsup{a_n}=1だろうか?
というところを問題にしたつもりです.
局所的なことが大域的に広げられるかどうかということが背景にあります.

83 名前:132人目の素数さん:04/09/01 04:19
>>82
これでいい?
a<b∈(0,1)をとる。x∈[a,b]に対し開近傍Uxをy∈Ux⇒f(y)=f(x)を満たすようにとる。
[a,b]はコンパクトなのでUx1・・・Uxnが[a,b]を覆うようにできる。
各Ui上でfは定数。かつUxi∩Uxj≠φ⇒Ui∪Ujでも定数ゆえ結局帰納法つかって
[a,b]上で定数。

84 名前:132人目の素数さん:04/09/01 05:25
過去ログ見てないからガイシュツかもしれないが。

【問】A,B,Cは複素数を成分にもつn次正方行列で、
AB-BA=C
CA-AC=2A
CB-BC=-2B
が成り立つとする。このとき C の固有値が整数である事を示せ。

85 名前:132人目の素数さん:04/09/01 05:56
>>84
これって正標数では反例あるんだっけ、ないんだっけ?

86 名前:132人目の素数さん:04/09/01 09:03
>>74 発散では?

87 名前:28:04/09/02 03:57
>>28 を書いた者だけど、考えてくれてありがと >>62
これは実は前スレの↓を考えてるときに気がついた。

面白い問題おしえてーな 八問目
http://science3.2ch.net/test/read.cgi/math/1074751156/901
> 901 名前:132人目の素数さん[] 投稿日:04/08/26 15:10
> かなり挑戦的な問題考えました。
> もし解けたら号泣です。
> 面積1の三角形上に2点を独立の一様分布から選ぶ。
> これら2点を通る直線は確率1で元の三角形を三角形と四辺形に分割する。
> 分割してできた三角形の面積をA、四辺形の面積をBとするとき、|A-B|を求めよ。

>>28 と同じようにして、
F の2点を一様かつ独立に選ぶとき、その2点が直線
x cos(θ) + y sin(θ) = r
上にある確率は、L(θ,r)^3/(3S^2) dr dθ になることが分かる。
これを使って、上の問題が計算できる。

88 名前:28:04/09/02 03:58
で、前スレの 901 だけど、A-B の絶対値の期待値 E(|A-B|) を計算する。
三角形の面積を S とすると E(|A-B|)/S は三角形によらない値をとるので、
O(0,0), P((√3)/2,1/2), Q(0,1) の正三角形に限定して考える。(S = (√3)/4)
分割されてできた三角形に頂点 O が含まれていて、
直線と OP, OQ の交点を、それぞれ P', Q' とするとき、
OP'≧OQ' となる場合を考える。(確率 1/6)
選ばれた2点を通る直線を x cos(θ) + y sin(θ) = r とすると、
上の条件は、-π/6≦θ≦π/6, 0≦r≦cos((π/3)-θ) で、
この直線が選ばれる確率は、
P(θ,r) drdθ = (1/(3S^2)) (P'Q')^3 drdθ
= (2/√3)r^3/{cos(θ)cos((π/3)-θ)}^3 drdθ。
分割されてできた三角形と四角形の面積は、
A = △OP'Q' = S*OP'*OQ' = Sr^2/{cos(θ)cos((π/3)-θ)}, B = S - A。
以上を使って
E(|A-B|) = 6 ∫[-π/6,π/6] ∫[0,cos((π/3)-θ)] |A-B| P(θ,r) dr dθ
を計算すると、
E(|A-B|)/S = (1 + 2*log(2))/6 ≒ 0.39772。
ついでに、A-B の期待値は、E(A-B)/S = -1/9。

89 名前:132人目の素数さん:04/09/02 04:39
× O(0,0), P((√3)/2,1/2), Q(0,1) の正三角形に限定して考える。
○ O(0,0), P(1/2,(√3)/2), Q(1,0) の正三角形 OPQ に限定して考える。

90 名前:132人目の素数さん:04/09/02 07:04
>>86
んなわけない。Γ(s)=∫[0,1]x^(s-1)e^(-x)dxはRe(s)>0で正則だからF'(1)も有限確定値。

91 名前:132人目の素数さん:04/09/02 07:05
>>87
なるほど。エレ解だ。すばらしぃ。

92 名前:132人目の素数さん:04/09/02 12:16
>>90 スマソ。 Γ'(1) = -γ = 0.577...(オイラー定数) ですか?

93 名前:FeaturesOfTheGod ◆UdoWOLrsDM :04/09/02 16:55
π=∫_{-1}^{1}(1/√(1-x^2)dx)として、
実数(複素数でもいい。)xに対して、sin(x)=∑_{n=0}^{∞}((-1)^nx^(2n+1)/(2n+1)!)とする。
sin(π/2)=1を証明せよ。

94 名前:羽村:04/09/02 17:02
>>93
π=2∫_{-1}^{1}(1/√(1-x^2)dx)じゃないの?

95 名前:羽村:04/09/02 17:11
↑間違えた。

96 名前:132人目の素数さん:04/09/02 17:25
n次の多項式f(x)=Σ[k=0,n] a(k)x^kが -1≦x≦1の範囲で
√(1-x^2) * |f(x)|≦1
を満たすとき、
|a(n)|の最大値を求めよ。


答えはめる欄。

97 名前:132人目の素数さん:04/09/02 20:12
        _    / ̄ ̄ ̄ ̄ ̄ ̄ ̄ ̄
      /_ \ <  自分の名前間違えちゃ〜(ry
      (  _ノ` )  \________
    /     \
   / /\   / ̄\
 _| ̄ ̄ \ /  ヽ \_
 \ ̄ ̄ ̄ ̄ ̄ ̄ \__)
  ||\            \
  ||\|| ̄ ̄ ̄ ̄ ̄ ̄ ̄|| ̄
  ||  || ̄ ̄ ̄ ̄ ̄ ̄ ̄||
  ||  ||           ||










98 名前:132人目の素数さん:04/09/05 12:59
まずは易しい問題から。
n, k は自然数とし、 2 ≦ k ≦ 3^n とする。
k 個の玉がって、見かけは皆同じがが、、
k - 1 個の玉は同じ重さで、他の 1 個だけ少し重い。
天秤ばかりを高々 n 回使用し、その重い玉を見つけよ。

99 名前:132人目の素数さん:04/09/05 14:15
>>98
(1)k が3の倍数のとき
k/3 個ずつ3グループ(A, B, C)に分け、A と B を比較する。
(a) A>B のとき
 少し重い玉は A の中にある。
(b) A<B のとき
 少し重い玉は B の中にある。
(c) A=B のとき
 少し重い玉は C の中にある。
(2)k が3の倍数でないとき
[k/3+1/2] 個のグループ2つ(A, B) と k-2*[k/3+1/2]個1つ(C)に分け、(1)の方法でA と B を比較する。

この方法によって、(k-1)個の玉の重さが同じで、他の(k-1)個より重い玉を1つ含む k 個の玉を
重い玉を1個含む高々 3^(n-1) 個のグループにすることが出来る。

同じ操作を高々(n-1) 回繰り返すと、3個以下のグループに分けることが出来る。残りが
(1)1個のとき、
それが重い玉である。
(2)2個のとき、
二つの玉を天秤にかけて、重いほうが求める玉である。
(3)3個のとき、
玉a, b, c について、a と b を比較すると、
a>b → a, a<b → b, a=b → c と判別できる。

よって、題意が示された。

100 名前:132人目の素数さん:04/09/05 14:20
>98
玉をぶつけていっしょに転がればそいつが重い玉


101 名前:FeaturesOfTheGod ◆UdoWOLrsDM :04/09/05 14:39
Uを、一次元ユークリッド空間の空でない連結開集合とする。
(D)'をU上のdistribution全体のなす空間とする。
(D)'の元で超関数微分が0になるものは、定数関数から誘導される汎関数しかないことを証明せよ。

102 名前:132人目の素数さん:04/09/05 15:20
FeaturesOfTheGod
は暇でいいな

103 名前:132人目の素数さん:04/09/05 22:20
>>100
完全弾性体であるとは仮定されていない。
女性体?

104 名前:132人目の素数さん:04/09/06 10:07
ここの問題46-3の解法を教えろ!
ttp://www.asahi-net.or.jp/~nj7h-ktr/kadai02-03.html


105 名前:132人目の素数さん:04/09/06 22:37
>>92
正解じゃ!!!

106 名前:132人目の素数さん:04/09/07 11:02
サッサと教えろ!

107 名前:132人目の素数さん:04/09/07 13:14
>106
104のことか?

sin(18°)やsin(75°)が(√(5)-1)/4だったり(√(6)+√(2))/4だったりすることや
加法定理を使っていいなら力技でできると思うが。3°と15°かいな。

だが、いわゆる「エレガントな解法」ではないわな。
「エレガントな解法」があるのかどうかは知らぬ。


108 名前:132人目の素数さん:04/09/07 20:02
エレガントな解答をキボンヌ!

109 名前:132人目の素数さん:04/09/07 20:23
Γ'(1/2)を求めよ。

110 名前:132人目の素数さん:04/09/07 21:21
>>106
計算すると、きれいな答えにはならないな。
18°と15°が逆なら両方とも9°になるからエレガントな解き方もありそうだね。
多分間違ってるんじゃない?

111 名前:107:04/09/08 10:36
すまぬ。

えらくシンプルで馬鹿な勘違いをしていた。たぶん、3と15にはならんわ。
台風19号に向かって逝ってくる。

112 名前:132人目の素数さん:04/09/10 21:55:00
【問題】
A,B,C,Dの4人は、独立に 2/3 の確率で嘘をつく。
Aが嘘をついたとBが主張したことをCが否定したとDが主張したとき、
Aが真実を言った確率を求めよ。

これって既出ですか? ハァハァ…。

113 名前:132人目の素数さん:04/09/10 22:11:09
チンチン! まだぁ〜

114 名前:132人目の素数さん:04/09/10 22:14:20
1/3

115 名前:132人目の素数さん:04/09/10 22:59:20
ここに 8 個の球がある。見かけは皆同じだが、このうち 6 個は重さが同じで、
あと 2 個は少し重い。その 2 個の重さは同じである。
天秤ばかりを 3 回使用してその重い 2 個を見つけよ。

http://science3.2ch.net/test/read.cgi/math/1091972996/13
にヒントを書いたけど見ないで解いてね。

116 名前:132人目の素数さん:04/09/10 23:02:48
>114
せめて高校の数学を終了してからにしろ。
それまではチラシの裏に書いてろ、な!

>>112
解答きぼん

117 名前:132人目の素数さん:04/09/10 23:19:11
>A,B,C,Dの4人は、独立に 2/3 の確率で嘘をつく。

答えは1/3以外ありえない。 

118 名前:132人目の素数さん:04/09/10 23:25:26
>>115
できる?ほんとに?一回目のせるのは
1+1、2+2、3+3、4+4のいづれかだけど
つりあったばあい。のこる可能性は
1+1でつりあった場合
重いのが1つづつ=1通り or 乗せたやつは全部軽い=C[6,2]=15通りで計16通り。
2+2でつりあった場合
重いのが1つづつ=4通り or 乗せたやつは全部軽い=C[4,2]=6通りで計10通り。
3+3でつりあった場合
重いのが1つづつ=9通り or 乗せたやつは全部軽い=C[2,2]=1通りで計10通り。
4+4でつりあった場合
重いのが1つづつ=16通り。
どれやっても10通り以上の可能性がのこるんだけど。しかし天秤はあと2回で9通りしか
場合分けできないのに。できると思えないんだけど。

119 名前:132人目の素数さん:04/09/10 23:33:42
なんか読みづらいと思ったら
>1つづつ
これだ。「1つずつ」だろ。

120 名前:132人目の素数さん:04/09/10 23:34:08
>>112
ようは独立な事象A,B,C,DがあってP(A)=P(B)=P(C)=P(D)=1/3のとき
X=D⇒(C⇒not(B⇒notA))という条件下でのP(A)をもとめよ。
だとおもうんだけど
X⇔notD or (notC or not(notB or notA))
⇔notD or (notC or (B & A))
⇔notD or notC or (B&A)
でもとめるのはP(X&A)/P(X)でいいの?

121 名前:132人目の素数さん:04/09/10 23:34:56
>>119
センターの国語110点のオレにこれ以上要求すな。

122 名前:132人目の素数さん:04/09/10 23:38:20
>>112
Aが嘘をつく確率は2/3

「Aが嘘をついた」とBが主張する確率は、
P(Bが「Aが嘘をついた」と主張|Aが嘘をつく)*P(Aが嘘をつく)+
P(Bが「Aが嘘をついた」と主張|Aが嘘をつかない)*P(Aが嘘をつかない)
=1/3 * 2/3 + 2/3 * 1/3 = 4/9

「Aが嘘をついた」とBが主張したことをCが否定する確率は、
P(Cが否定|「Aが嘘をついた」とBが主張)*P(「Aが嘘をついた」とBが主張)
P(Cが否定|「Aが嘘をついた」とBが主張しない)*P(「Aが嘘をついた」とBが主張しない)
= 1/3 * 4/9 + 2/3 * 5/9 = 14/27

「Aが嘘をついた」とBが主張したことをCが否定することをDが主張する確率は、
P(Dが主張|Cが否定)*P(Cが主張)
P(Dが主張|Cが否定しない)*P(Cが否定しない)
= 1/3 * 14/27 + 2/3 * 13/27 = 40/81

よって答えは 40/81

123 名前:132人目の素数さん:04/09/10 23:54:25
>122
これ合ってるの?

124 名前:132人目の素数さん:04/09/11 00:03:14
>>123
各人が独立に一定確率で嘘をつくためには、全ての事実を知っている必要がある。

125 名前:132人目の素数さん:04/09/11 00:05:37
>>123
合ってるも何も122は答えるものを間違ってる。

126 名前:132人目の素数さん:04/09/11 00:09:08
すくなくとも>>120>>122は両立しないな。

127 名前:132人目の素数さん:04/09/11 00:39:51
まぁ模範解答が気になる

128 名前:122:04/09/11 00:46:56
『Aが』真実を言った確率だったorz
今から首吊ってきます。

129 名前:132人目の素数さん:04/09/11 00:47:19
>>112の問題は、下のP.40問10で、解答はついてない。
https://sslserver.sbs-serv.net/nippyo/books/bookinfo.asp?No=2406

130 名前:132人目の素数さん:04/09/11 00:47:49
違った、P.24問16だった。

131 名前:132人目の素数さん:04/09/11 01:13:36
>>115
できた.3,3,2にわけて,始めに3,3を天秤で量る.

132 名前:132人目の素数さん:04/09/11 01:14:36
>>131
でつりあったら?

133 名前:132人目の素数さん:04/09/11 01:23:04
>>131
おもりをABCDEFGHとしてABC、DEFをはかってつりあった場合重いおもりの可能性として
AD、AE、AF、BD、BE、BF、CD、CD、CF、GH
の10通りがのこる。次どうすんの?

134 名前:132人目の素数さん:04/09/11 01:23:58
AD、AE、AF、BD、BE、BF、CD、CE、CF、GH
の間違い。あと2回でできんの?


135 名前:132人目の素数さん:04/09/11 01:32:16
ちょっとまって,今,解答作るから.

136 名前:132人目の素数さん:04/09/11 01:40:14

まず8個の玉をA:○○○,B:○○○,C:○○に分け,A, Bを天秤で量る(一回目).
つりあった場合はCが二つとも重い玉.
つりあわなかった場合.重い方をA,軽い方をBとする.このとき,重い玉を●として次の2通りがある.
(1) A:○○●,B:○○○,C:○●
(2) A:○●●,B:○○○,C:○○
したがってCの2つを天秤で量れば,(1), (2)のどちらであるかがわかる(2回目).
あとは,Aから2つとってそれを天秤で量ればよい(3回目).

(1)の場合
まず,Cの計量で重い玉が1つ見つかる.
Aの測定でつりあった場合は量らなかったものが重い玉.傾いたらもちろんそれが残りの重い玉.

(2)の場合
Aの測定でつりあったらその二つが重い玉.傾いたら,その重いほうと,量らなかった残りの1個が重い玉.

137 名前:132人目の素数さん:04/09/11 01:43:12
間違えました.出来てません.

138 名前:132人目の素数さん:04/09/11 01:46:50
一回目どう計っても10通り以上の可能性がのこる場合がおこりうると思うんだが。
出題ミスくさい。

139 名前:132人目の素数さん:04/09/11 01:48:04
いや,やっぱできる.AとBがつりあった時は
Aから2つとってはかる(2回目)
Bから2つとってはかる(3回目)
を追加.

140 名前:132人目の素数さん:04/09/11 01:48:18
なんじゃ。>>115みたら「そんなこと不可能」だって。なめとんか。

141 名前:132人目の素数さん:04/09/11 01:50:30
いや,やっぱ出来ない.すまん.

142 名前:132人目の素数さん:04/09/11 01:50:31
>>139
それだと全部つりあったときに
・一回目はかってない2つが重い
・ABのなかで2回目、3回目にえらばれなかった2つが重い
の2つの可能性をくべつできない。

143 名前:132人目の素数さん:04/09/11 02:01:19
壮大な釣りだった訳か。

144 名前:132人目の素数さん:04/09/11 04:56:14
>>112
まず、「ABCDは順番に発言し、BCDの3人は前の発言者の発言の真偽について述べていることは分かっているものとする。」は条件に加えなきゃいけない。
BとかCが「キタ━━━━(゚∀゚)━━━━ッ!!」とか意味ないことを言う場合は除かなきゃいかん。

結論から言うと、答えは13/41。

もし問題が、
「Aが嘘をついたとBが主張したとき、Aが真実を言った確率は?」

だったら、Bがこの発言をする場合は、(A嘘、Bほんと)か、(Aほんと、B嘘)
の2通りで、確率は
2/3*1/3+1/3*2/3=4/9
で、そのうちAがほんとを言ってる場確率は後者の2/9だから、求める確率は、
(2/9)/(4/9)=1/2
となる。

今の問題の場合は、Dからこの発言が出る場合は、
(Aほんと、Bほんと、C嘘、D嘘)等の8通り。(ほんとと嘘の数が偶数である場合になる。)
この8通りのそれぞれの確率を求めて合計したものは、41/81。
うち、「Aほんと」をふくむものの確率の合計は、13/81。
したがって、答えは13/41になる。

145 名前:132人目の素数さん:04/09/11 13:58:33
n を 3 の倍数で奇数とする。
一辺の長さが n√13 の正方形を、直角をはさむ二辺の長さが
2, 3 となる直角三角形で敷き詰める事は不可能である事を示せ。

n が 3 の倍数で偶数の時、多くの場合に不可能が示せたが、
一般的にはどうなるのだろう。

146 名前:132人目の素数さん:04/09/11 23:29:13
>>145
>直角をはさむ二辺の長さが2, 3 となる直角三角形
って2種類あるけど両方つかっても不可能であることをしめさないといけないの?

147 名前:132人目の素数さん:04/09/11 23:43:03
>>146
2種類あるか?

148 名前:132人目の素数さん:04/09/11 23:44:31
>>147
あるじゃん。(0,0)、(2,0)、(0,3)をむすんだヤツと(0,0)、(3,0)、(0,2)をむすんだヤツ。

149 名前:132人目の素数さん:04/09/11 23:47:52
ひっくり返せ

150 名前:132人目の素数さん:04/09/11 23:52:27
>>149
だからひっくりかえさないで同じものばかりでは埋め尽くせないことをしめせといってるのか
それともひっくりかえせば同じになる2種類をつかってすら埋めつくせないことまで
しめせないといけないのかを聞いてる。まあ後者だろうけど。前者だとそんなに
むずかしくなさそうだし。

151 名前:145:04/09/12 09:24:06
裏返しを許しても不可能です。

152 名前:132人目の素数さん:04/09/12 18:25:06
>>145
結局は4角形の4辺に三角形の斜辺をならべていって(しかも同じ向きに)
のこりの部分を2×3の長方形でうめられるかという問題に帰着できるけど
それはn=3mとして面積39m^2の領域を面積6の長方形でうめる問題になって
奇偶性をみて不可能となった。しかし前半部分の問題を還元する部分の記述が
めちゃめちゃめんどい。すぱっととけるの?

153 名前:132人目の素数さん:04/09/12 18:26:59
39m^2じゃなくて39m^2-36mね。

154 名前:145:04/09/12 19:07:32
>>152
類似の考え方でスパッといけるよ。
そろそろ答え言おうか?

155 名前:132人目の素数さん:04/09/12 19:25:20
>>154
おながいします

156 名前:145:04/09/12 20:48:18
もし可能とすると三角形の数は (13*n^2)/3 個。これは奇数。
√13 は無理数だから、正方形の辺の一部となるのは三角形の斜辺のみ。。
従って辺に張り付く三角形(の斜辺)は 4n 個。
それ以外に (n^2*√13)/3 - 4n 個の斜辺が内部に現れるが、
内部にある斜辺の延長線分で、三角形の辺の和集合になるものを考えると、
必ず両側にこれを斜辺とする三角形が同数個現れる。

なぜなら、その延長線分の片側にこれを斜辺の一部とする三角形がが
p 個、もう一方の側に q 個あるとする。
残りは長さ 2, 3 の辺が幾つかあるだけだから、
p√13 + m = q√13 + n, m, n は非負整数。
p ≠ q とすると、 √13 が有理数となる。
よって、 p = q.
この三角形の3辺はすべて長さが違うから重複して数えられる事は無い。
よって斜辺が内部にある三角形は偶数個。

寄って矛盾。

157 名前:132人目の素数さん:04/09/14 03:09:44
>>156
×寄って→○因って

158 名前:132人目の素数さん:04/09/14 08:22:03
一羽半の鶏が一日半で一個半の卵を産むとしたら、
九羽の鶏が九日で何個の卵を産むか?

          アイザック・アシモフ 「われはロボット」 より


159 名前:132人目の素数さん:04/09/14 08:27:20
9個




1個半の卵って何だよ!?
半分の殻にプカプカと黄身が浮いているのかよ!
前提が偽であるならばいかなる結論も真である。
ということで、9個と答えても間違いないだろう。
出題者は54個を正解にしたかったのかも知れないがな。

160 名前:132人目の素数さん:04/09/14 08:42:49
>159
正解は54個。

161 名前:132人目の素数さん:04/09/14 13:36:38
一羽の鶏が、一日に平均一個の卵を産むとしたら、
一羽の鶏が、ある一日に一個の卵を産む確率は?

162 名前:FeaturesOfTheGod ◆UdoWOLrsDM :04/09/14 13:38:39
Re:>161 Poisson分布?

163 名前:161:04/09/14 14:41:40
>>162
Yes. ポアソン過程といった方がいいのかな。事象間無相関。

164 名前:FeaturesOfTheGod ◆UdoWOLrsDM :04/09/14 15:43:21
Re:>163 exp(-1)

165 名前:161:04/09/14 18:02:44
>>164
正解。まあ鶏が一日にいくつも卵を産むかという問題もあるけど。
一個も産まない確率=一個産む確率=1/eみたいのをうまく使った
面白い問題とかできないかなと思ったり。

166 名前:132人目の素数さん:04/09/15 21:10:10
サイコロを6回振って、1の目が1回出る確率とかと同じようなものか。
似たような問題で6種類のオマケのカードが入っているお菓子をいくつ買えば、全種類のカードを集めることができるかとかいうものもあるな。


167 名前:FeaturesOfTheGod ◆UdoWOLrsDM :04/09/15 21:57:59
Re:>166
それと何が同じだと?
まさかPoisson分布と同じというのではないだろうな。
B(6,1/6)とP(1)は全然違うものだ。
十分大きいnに対してベルヌーイ分布B(1/n)に従う確率変数列(独立とする。)X_k(k=1,…n)を考えるとき、
Y=X_1+…+X_nの従う分布は、
P(1)の分布の原点に近いあたりはこれに似るようになる。

168 名前:132人目の素数さん:04/09/15 22:05:05
そろそろ「これは」という問題を頼む。

169 名前:132人目の素数さん:04/09/15 22:13:18
>>167
原点に近くなくてもいいだろ。
Bin(n,1/n)→P(1) (n→∞)
だ。

170 名前:132人目の素数さん:04/09/16 02:10:05
FeaturesOfTheGod ◆
出てくるなよ

171 名前:132人目の素数さん:04/09/16 06:54:35
D={(x,y)|0≦x<1, 0≦y<1}とし、次のようなDからDへの写像Tを考える。
T:(x,y)→(X,Y)
X=2x, Y=y/2 (0≦x<1/2のとき)
X=2x-1, Y=y/2+1/2 (1/2≦x<1のとき)

nを自然数とするとき、T^n(P)=PとなるようなD上の点Pの個数を求めよ。
(すなわち、n回Tで変換して下に戻るような点の個数を求めよ。)


172 名前:132人目の素数さん:04/09/16 06:56:47
×下に戻る→○元に戻る
ごめん。

173 名前:132人目の素数さん:04/09/16 11:45:02
>>171
2^n-1

174 名前:132人目の素数さん:04/09/16 22:17:29
>166

全てのオマケの出る確率が同じだとすると、種類数の3倍買えば90%超えるぞ。
ただ、シークレットの存在があるから難儀だな。

175 名前:132人目の素数さん:04/09/16 23:16:53
>>173
正解。

176 名前:132人目の素数さん:04/09/17 06:14:35
別スレみてて思いついた
――問題――
P1・・・P(n+1)をn次元ユークリッド空間の点としてΔをその凸包とする。
行列A=(aij)を次でさだめる。
aij=
0      (i=j)
1      (i≠j, n+2∈{i,j})
d(Pi,Pj)^2 (otherwise)
(ただしd(Q,R)はQとRの距離)
このときdetA=-(-2)^n(n!volΔ)^2が成立することをしめせ。
――――――
たぶんいけると思うんだけど。

177 名前:132人目の素数さん:04/09/18 08:14:31
高木貞治、代数学講義、共立
に n = 4 の場合が書いてあるけど同じ考え方でいけるよ

178 名前:132人目の素数さん:04/09/18 18:44:56
n = 3 の場合だった。

179 名前:132人目の素数さん:04/09/18 20:27:35
>>176の公式ってなんか名前ついてんの?

180 名前:132人目の素数さん:04/09/19 01:55:29
いやいや、n = 6 の場合だったかな。

181 名前:132人目の素数さん:04/09/19 13:37:42
高校のころ思いついたんだけど結局解けなかった問題を誰かに解決していただきたく。

同じ長さのマッチ棒で1辺がn本の正三角形を作る。
また、中にもマッチ棒の端と端がくっつくように配置して小さい正三角形も作るとする。
なんつーかこう、ピラミッド型に規則的に作ると考えてください。
正三角形の内部に、各辺と平行に√3/2間隔で1本〜n-1本のマッチ棒が配置されるとでも言えばいいのかな。

ex. 1辺を1本で作ると上向きの正三角形がひとつできる。
  1辺を2本で作ると中に上向きの正三角形が大小あわせて4つ、下向きの正三角形がひとつできる。
  1辺を3本で作ると中に上向きの正三角形が大小あわせて10個、下向きの正三角形が3つできる。


このとき、
1.上向きの正三角形の総数は?
2.下向きの正三角形の総数は?
この二つをnを用いて表すことができるでしょうか?という問題です。
1.は割と簡単にできた気がするのですが2.はお手上げだったような・・・。
思いついたときには面白い問題かなと思ったんですが・・・どうでしょ?

182 名前:132人目の素数さん:04/09/19 14:37:42
n(n+1)(n+2)/6。
(4n^3+6n^2−4n−3+3(−1)^n)/48。


183 名前:181:04/09/20 02:28:33
そんなあっさりと・・・。
簡単に説明きぼんぬ。

184 名前:132人目の素数さん:04/09/20 11:37:14
新作問題

ジョーカーを除いたトランプ 52 枚の中から1枚のカードを抜き出し、
表を見ないで箱の中にしまった。
そして、残りのカードをよく切ってから 51 枚抜き出したところ、
スペード、ハート、クラブが 13 枚、ダイヤが 12 枚であった。

このとき、箱の中のカードがダイヤであった確率はいくらか。

185 名前:132人目の素数さん:04/09/20 12:58:54
問題訂正

ジョーカーを除いたトランプ 52 枚の中から1枚のカードを抜き出し、
表を見ないで箱の中にしまった。
そして、別の人が残りのカードをよく切ってから 51 枚抜き出したところ、
スペード、ハート、クラブが 13 枚、ダイヤが 12 枚であった。

箱の中にしまった人はこの事を知らないとする。このとき、箱にカードをしまった人にとって、
箱の中のカードがダイヤである確率はいくらか。

186 名前:132人目の素数さん:04/09/20 13:26:09
三角形ABCがある。 ∠B=60°であり、∠Bに向かい合う辺ACの長さbは整数である。
また、他の辺の長さa,cは素数である。
このとき、△ABCは正三角形であることを証明せよ。

187 名前:132人目の素数さん:04/09/20 14:40:26
余弦定理から b^2 = a^2 + c^2 - ac.変形して (b-c)(b+c) = a(a-c).
この式からa=c⇔b=cとわかるのでa≠c∧b≠cと仮定して矛盾を導く.
k = (b+c)/a = (a-c)/(b-c)≠0 とおいてbについて解くと (b =) ka - c = (a-c-kc)/k.
したがってc = (1-k^2)aを得るが,a,cは素数だから1-k^2 = 1.よってk=0となり矛盾.

188 名前:132人目の素数さん:04/09/20 14:48:17
1辺が1の正方形の中に,相異なる3点A,B,Cを配置するとき、
僊BCは決して鈍角三角形にはならない事を示せ。

189 名前:132人目の素数さん:04/09/20 15:02:22
>>188
ダウト

190 名前:189:04/09/20 15:08:59
訂正

1辺が1の正方形の中に,相異なる3点A、B、Cを配置する。
僊BCにおて、3辺の長さがすべて1以上のとき、
決して鈍角三角形にはならない事を示せ。


191 名前:187:04/09/20 15:51:10
しょうもない計算ミスがあったので全面的に書き直す.

(b-c)(b+c) = a(a-c)であるが,aが素数なので b+c=ka(kは1以上の整数)(b-c=kaとするとはじめの等式は成り立たない).
ゆえに b= ka-c = c + (a-c)/k.したがって(k^2-1)a = (2k-1)c.
c≦aとしても一般性を失わないが,その場合k=2のみが適する.つまりa=c.

192 名前:190:04/09/20 23:29:01
誰かわかりませんか?

193 名前:132人目の素数さん:04/09/20 23:58:12
鈍角をはさむ2辺の長さが1以上の時、他の一辺の長さは√2より大となるから矛盾。

194 名前:132人目の素数さん:04/09/21 08:58:33
>>193
Blow Job!

195 名前:186:04/09/21 10:55:56
>>191
Good job!

196 名前:132人目の素数さん:04/09/22 16:29:19
あげてみよ

197 名前:132人目の素数さん:04/09/22 21:10:54
f(x)をxに関する実係数の多項式であるとする。
f(x)=0を満たすxのうち、絶対値が最大であるものの絶対値をa(1)とおく。
同様にf'(x)=0を満たすxのうち、絶対値が最大であるものの絶対値をa(2)とおく。
以下、f''(x),f'''(x)の零点に対して、どうように数列a(n)を定義するとき、
a(n)は単調非増加数列であることを証明せよ。

198 名前:132人目の素数さん:04/09/22 21:17:40
>>197
 
>f(x)=0を満たすxのうち、絶対値が最大であるものの絶対値をa(1)とおく。
>同様にf'(x)=0を満たすxのうち、絶対値が最大であるものの絶対値をa(2)とおく。
 
この「f(x)=0を満たすxのうち、」というのは複素数解もふくめての全部のなかでの最大?

199 名前:132人目の素数さん:04/09/22 21:24:52
実数の範囲に限定すると反例あるね。

200 名前:197:04/09/22 21:33:08
正直、スマソ

201 名前:132人目の素数さん:04/09/22 21:39:38
>>197
n次式をn回微分するとき、

f(x)=1+2*x+3*x^2+・・・+(n+1)*x^n

とすると、f(k)(0) = (k+1)! (f(k)(0) は、f(x)の k 次導関数に 0 を代入したもの)
だから、n回微分までは単調増加数列になる。

n+1回以上微分すると 0 となるので、どちらにしても単調数列にはならない。

202 名前:132人目の素数さん:04/09/22 21:44:20
Lucas's theoremを流用すれば示せそうだが.
そうすると実係数多項式に限らず成り立つね.


203 名前:132人目の素数さん:04/09/22 21:47:43
Lucas's theoremってどんなん?

204 名前:132人目の素数さん:04/09/22 22:05:02
Lucas's Theorem:
If all zeros of a polynomial P(z) lie in a half plane,
then all zeros of the derivative P'(z) lie in the same plane.
[Ahlfors, p.29]

ここに証明を書くのはちとめんどい.
この系として,P(z)の零点を含む最小の凸多角形の中にP'(z)の零点は含まれる.
ということとかが言える.

205 名前:132人目の素数さん:04/09/22 22:09:56
>>204
すばらしい!!それで終了だ。その証明なににのってるの?証明むずい?

206 名前:132人目の素数さん:04/09/22 22:13:01
高木貞治 代数学講義
P72あたり

207 名前:132人目の素数さん:04/09/22 22:40:00
Pの零点の実部が全て0以下のとき
Pの零点をaとしzの実部が正のときとすると
z−aの実部は正で1/(z−a)の実部も正。
(dP/dz)(z)/P(z)=Σ(k/(z−a))の実部も正になるので
(dP/dz)(z)は0にならないのでdP/dzの零点の実部は0以下。
一般の場合はw=bz+cによって移せばいい。


208 名前:132人目の素数さん:04/09/22 22:40:37
>>205
Ahlfors, Complex Analysis 3rd.ed., p.29にある.
証明は難しくない.長くも無い.初等的.
>>206
そこにもあるのか.今度見てみようかな.

209 名前:132人目の素数さん:04/09/22 22:49:18
>>206に載ってるのは>>204の系の方
別のページを探せば載ってるかも知れないけど、
まだ見てない。

210 名前:132人目の素数さん:04/09/22 22:52:10
>>207
すばらしい。もう完全解決。

211 名前:132人目の素数さん:04/09/23 00:15:57
なんで0÷1はいいのに1÷0はだめなの?

212 名前:132人目の素数さん:04/09/23 00:41:33
>>211
0で割ることは出来ない。
理由はワカランが、恐らく答えを出すと無限となる

213 名前:132人目の素数さん:04/09/23 03:00:28
>>211
0÷1=□
とすると、割り算は掛け算の反対だから、
0=□×1
となって、□=0 と答えが出る。

1÷0=□
とすると、
1=□×0
だけど、□にどんな数を入れてもイコールが成り立たない。
だから、1÷0は計算できない。



214 名前:132人目の素数さん:04/09/23 04:25:37
http://page6.auctions.yahoo.co.jp/jp/auction/f28494711

215 名前:132人目の素数さん:04/09/23 15:12:15
自然数nと三次元空間上の点(a,b,c)に対し、na,nb,ncがいずれも整数となる時、
点(a,b,c)をn格子点と呼ぶ事にする。

頂点がいずれも1格子点であるような3次元空間上の凸多面体Aがある時、
自然数nに対してAに含まれるn格子点の数をf(n)とする。

この時ある整数係数多項式g(n)が存在し、
全ての自然数に対してf(n)=g(n)となる事を証明せよ。

216 名前:132人目の素数さん:04/09/23 17:25:27
ひびせんせかなんかの本で見たことあるな。

217 名前:132人目の素数さん:04/09/23 22:10:55
素数と関係する?

218 名前:132人目の素数さん:04/09/24 02:30:18
ごめんなさい。>>215の三段落目を修正。

この時ある整数係数多項式g(n)が存在し、

この時ある多項式g(n)が存在し、

219 名前:132人目の素数さん:04/09/27 23:26:01
1クラス40人の学級に、同じ誕生日の人がいる確率はどのくらいでしょうか?
閏年は考慮しなくてもいいです。

220 名前:132人目の素数さん:04/09/28 01:22:29
↑そのスレにあった問題なのだが↓どう?

>毎日が誰かの誕生日という状態にするには最低366人が必要だが
>無作為に集めた集団が毎日が誕生日になるために必要な人数の期待値は?


221 名前:132人目の素数さん:04/09/28 01:46:13
計算面倒なだけな気が……

222 名前:132人目の素数さん:04/09/28 02:00:00
365×(1/1+1/2+...+1/365)。


223 名前:132人目の素数さん:04/09/28 02:03:34
>>220
一般にnΣ[k=1 to n](1/k)だ。

違う誕生日がk人すでにそろっている状態で、全部の誕生日が揃うまでに必要な人数の期待値をE(k)とすると、n=365として、

E(k)=(k/n)(1+E(k))+(1-k/n)(1+E(k+1))
より、
E(k)=E(k+1)+(n/(n-k))

これとE(n)=0から、E(0)=nΣ[k=1 to n](1/k)が求まる。

224 名前:132人目の素数さん:04/09/28 04:25:01
てことはEをオイラーの定数とすると
nΣ[k=1 to n](1/k) は n(E+Γ'(n)/Γ(n)) さらに n(E+(Ψ(n+1)) なので
面倒な計算は省略するが
だいたい2365人くらいになるってことだな。


225 名前:132人目の素数さん:04/09/28 04:27:32
n(E+Γ'(n)/Γ(n)) は n(E+Γ'(n+1)/Γ(n+1)) のまちがいだ
忘れてくれ


226 名前:132人目の素数さん:04/09/28 15:41:56
20%くらいだろ

227 名前:132人目の素数さん:04/09/28 17:06:08
くらいだな

228 名前:132人目の素数さん:04/09/28 19:34:55
信じられないくらい長い文章題教えて

229 名前:132人目の素数さん:04/09/28 20:19:20
a,b,cにより構成される以下のような列を文章という。
abcacabc……

文章は以下の条件を満たす。
babcabc などのように、同じ部分を繰り返さない。
    (abcの繰り返し)
abcbabc のように途中に別のアルファベットを挟む場合は文章として良い。


この条件を満たす文章のうち、最大の長さを持つ文章を求めよ。


信じられないぐらい長くなるよw

230 名前:132人目の素数さん:04/09/28 20:32:09
漏れは文章とは認めない

231 名前:132人目の素数さん:04/09/30 03:12:47
>>228
北朝鮮の文章題は賛辞がやたら多いから長くなるな。

232 名前:132人目の素数さん:04/09/30 06:47:02
>>229
56くらい?

233 名前:132人目の素数さん:04/09/30 10:35:40
ttp://plaza.harmonix.ne.jp/~k-miwa/magic/something/brother.html
このページの「問題1」なんだけど、どう考えても1/2だよなぁ。

234 名前:132人目の素数さん:04/09/30 11:30:59
>>233
そのページを見て思ったこと。
出題者馬鹿だ。

235 名前:132人目の素数さん:04/09/30 17:01:05
>>233
夜聞こえてきた女の子の声は、その日たまたま遊びに来ていた従姉妹の娘だったのです。

236 名前:132人目の素数さん:04/09/30 18:23:12
>>171の解

x,yの2進表示を、それぞれ、
x=0.abc…
y=0.αβγ…
とする。ただし、有限で終わる場合は残りをすべて0と考える。
(例えば1/2は、0.1000…。0.0111…の表現は使わない。)
このとき、T(x,y)=(X,Y)とすると、
X=0.bc…
Y=0.aαβγ…
となる。したがって、点(x,y)を、その2進表示をxについては原点から右方向に、yについては原点(正確には-1の位置)から左方向に並べた両側無限点列
(…γβαabc…)
と同一視すると、Tはこの両側無限点列空間における左側へのシフトとみなせる。
したがって、n回シフトして同じ点列になる点列は、n個が循環している点列であり、その個数は2^n-1個となる。(2^n個のうち、1がn個続くもののみ除外)

237 名前:132人目の素数さん:04/09/30 22:30:09
>>233
女の子の声が聞こえる場合は以下の4つ
1.姉、妹の組み合わせで姉の声が聞こえた。
2.姉、妹の組み合わせで妹の声が聞こえた。
3.姉、弟の組み合わせで姉の声が聞こえた。
4.兄、妹の組み合わせで妹の声が聞こえた。




238 名前:132人目の素数さん:04/10/01 00:14:11
>>233
「古典と言ってよい問題ですが、確率の基本的な考え方を理解するのにはよい問題です」
って、5年以上も訂正されてないのが恥ずかしいよ。
これが日本の数学教育のレベルなんだろうなぁ。

239 名前:132人目の素数さん:04/10/01 00:26:00
>>238
丁半ばくちの話をヒントに出してたけど、そのページの管理人に次のような質問をしたらなんて返ってくるかな。
『サイコロを二回振りました。
 一回目に偶数が出たことは分かっています。
 では、二回目に偶数が出る確率はいくつでしょうか?』


管理人理論が正しければ、1/2にはならないw

240 名前:132人目の素数さん:04/10/01 00:31:11
おっと、管理人理論によれば一回目と二回目は区別されるんだよな・・・
っていうことは、あれか?

『サイコロを二回振りました。
 一回目に偶数が出たことは分かっています。
 では、二回目に偶数が出る確率はいくつでしょうか?』

だと1/2になって

『サイコロを二個振りました。
 一つは偶数が出ました
 もう一つが偶数である確率は何でしょう』

だと、確率が変わる訳か。 もうアフォとしか……

241 名前:132人目の素数さん:04/10/01 00:37:25
本当は答えわかってるようにも思えるけど。


242 名前:132人目の素数さん:04/10/01 00:48:07
>>241
「丁半ばくち」のヒントをわざわざ述べていることから、
本気で2/3だと主張していると思うべきだろう。

243 名前:132人目の素数さん:04/10/01 00:53:16
http://plaza.harmonix.ne.jp/~k-miwa/magic/something/brother4.html
>実際のところ、本当の正解は1/2という説もあります。どちらが正解なのでしょう(笑)。

という記述があるところから、正解を知っている可能性も否定できない。

244 名前:132人目の素数さん:04/10/01 01:00:04
>>243
実際のところ、本当はどうなのでしょう(笑)。考えてください。
なお、この正解を私に求められましても、答えないことにしています。

245 名前:132人目の素数さん:04/10/01 02:09:46
1/2+1/(2^2)+1/(2^3)+1/(2^4)+....∞=??

1/a+1/(a^2)+1/(a^3)+1/(a^4)+....∞=??のときは??

246 名前:132人目の素数さん:04/10/01 02:28:40
>>245
ただの無限等比級数じゃないのか……?

247 名前:132人目の素数さん:04/10/01 04:23:58
x-y平面上の有理点をいくつかの色で塗り分ける。
距離が1だけ離れた二点を同じ色で塗るとき、最大で何色色が必要か答えよ。


1色だと、条件を満たすようにぬれるけど、それより多くね♪

248 名前:132人目の素数さん:04/10/01 04:30:50
三角形の内部に点Pを取る。
点Pから三角形の各頂点への距離の和をS
点Pから三角形の各辺への距離の和をTとするとき
S≧2Tを示し、等号成立条件を求めよ。


同様に、四面体において内部に点Pを取り
点Pから四面体の各頂点への距離の和をS
点Pから四面体の各辺への距離の和をTとするとき、
S≧T√8を示し、等号成立条件を求めよ。

249 名前:132人目の素数さん:04/10/01 04:34:03
>>247
一色以外の解あるの?
とりあえず単位円上は全部原点と同じ色ぬらなきゃいけないし。まあ白だとして
すると単位円上の同点を中心とする円の軌跡は全部白。つまり半径2の円の周と内側全部白。
この作業繰り返したら真っ白じゃないの?

250 名前:132人目の素数さん:04/10/01 04:39:00
>>249
有理点だからそう簡単じゃないんじゃない?
簡単にいくのかもしれんが。

251 名前:132人目の素数さん:04/10/01 04:39:40
>>248
>各辺への距離の和をTとするとき
 
これ鈍角三角形のときは辺を延長した直線との距離で桶?

252 名前:132人目の素数さん:04/10/01 04:40:17
>>250
ああ、有理点か。なるほど。

253 名前:132人目の素数さん:04/10/01 04:47:59
>>251
それでOK。

254 名前:132人目の素数さん:04/10/01 04:53:01
>>247
これもしかして(a,b)と(c,d)の距離が1
⇒a-cとb-dの分母の素因子≡1 (mod 4)な希ガス。
そうならG={q∈Q | qの分母の素因子≡1 (mod4)}とおくときQ/Gが無限群だから
いくらでもたくさんとれたりする希ガス。

255 名前:132人目の素数さん:04/10/01 04:55:48
>>240
下だと、「ひとつは偶数が出てるよ」ということだけ知らされた、ようにもとれるぞ。それだと2/3。

256 名前:132人目の素数さん:04/10/01 04:57:40
>>255
ごめん、1/3。

257 名前:132人目の素数さん:04/10/01 05:10:39
>>247
そうだ。やっぱり
a,b,c∈Z\{0}、a^2+b^2=c^2、(a,b,c)=1⇒cの任意の素因子≡1 (mod 4)
もし2|cだと(a,b,c)=1からa,bは奇数でa^2+b^2≡2 (mod4)は平方数になりえない。
p|cだと(a/b)^2≡-1 (mod p)であるがこのときZ/pZの乗法群が位数4の
元をもつので4|p-1。つまりp≡1 (mod4)でなければならない。
これから原点中心の単位円上の有理点(a,b)の分母の素因子≡1 (mod 4)がいえる。
よって有理点P,Qの同値関係P〜QをP〜Q⇔P=∃P0・・・∃Pn=Q s.t PiとP(i+1)間の距離が1
でさだめると(a,b)〜(c,d)⇒a-c、b-dの分母の素因子≡1 (mod 4)になるがとくに
(0,0),(1/3,0),(1/9,0),(1/27,0)・・・は全部ちがう類に入る。
よって結局無限に多くの色をつかうことができる。
あってる?

258 名前:132人目の素数さん:04/10/01 05:16:32
OK

259 名前:132人目の素数さん:04/10/01 12:54:00
>243
いや、単純に本人も訳わからんようになっているだけと思われ。

260 名前:132人目の素数さん:04/10/02 05:26:16
「子供が2人いる」
男男1/4、男女1/2、女女1/4

「少なくともどっちかは女の子だったはずだけど…」
男女2/3、女女1/3

「女の子の声がしたよ」
男女1/2、女女1/2

3番目は男の子の声が聞こえる可能性もある状況において、女の子の声が聞こえている(すなわち任意に一人取り出したら女だった、という状況である)わけだから、2番目より得ている情報は多いんだよね。

261 名前:132人目の素数さん:04/10/02 11:02:10
>>248の後半が・・・前半は数オリの問題かなんかでみた記憶があるんだけど。
この手の問題ってたいがい正多面体の重心とか垂心のときとかが答えになりそうな
もんだけど後半は正四面体の重心のときが等号じゃないみたい。すげー厄介な悪寒。

262 名前:132人目の素数さん:04/10/03 19:01:53
>>248
答えおながいします。

263 名前:132人目の素数さん:04/10/03 19:45:46
n番目の素数をP(n)とするとき
P(n+1)<=P(n)+n が成り立つ 

264 名前:132人目の素数さん:04/10/03 23:01:54
>>248
答えおながいします。

265 名前:132人目の素数さん:04/10/04 05:01:05
>>248
答えおながいします。

266 名前:132人目の素数さん:04/10/04 09:20:02
>>263
これも難しいな。素数定理で反例が有限個しかないのはすぐでるけど
その可能性の最大値が天文学的数字になって計算機でも手も足もでない・・・
これあっても有限個しかないことをしめせじゃなくてホントに全然ないこと示せなの?

267 名前:132人目の素数さん:04/10/04 23:22:46
>>248
前半は分かった。三角形ABCの内部に点Pをとる。PからBCにおろした垂線の足をD、
CAにおろした垂線の足をE、ABにおろした垂線の足をFとする。
明らかに、四角形AFPEは円に内接し、外接円の直径はAPである。従って正弦定理より
FE/sin(∠FPE)=APが成立し、∠FPE=π-∠Aであるため、FE=APsinA  ∠は省略が成り立つ。
今、点F,Eから直線BCに垂線をおろし、その足をF',E'とすればEF≧E'F'が成立する。従って
APsinA≧E'F'が成り立つ。また、E'F'=E'D+DF'が成立し、E'D=PEsinB DF'=PFsinCが成り立つ。
このことから、AP≧(sinB/sinA)PE+(sinC/sinA)PFが成立する。

同様にして、PB,PCについても同等の不等式を得れば、
AP+BP+CP
≧((sinC/sinB)+(sinB/sinC))PD + ((sinB/sinA)+(sinA/sinB))PE + ((sinC/sinA)+(sinA/sinC))PF
≧2(PD+PE+PF)

等号成立条件は∠A=∠B=∠Cかつ、EF=E'F'等なので、△ABCが正三角形でかつ
点Pが重心に位置するとき。

268 名前:132人目の素数さん:04/10/05 10:02:14
m^2=Σ[k=1,n] k^2
を満たす、自然数m,nを全て求めよ。

269 名前:数学科布施 ◆FUSEz5Eqyo :04/10/05 11:33:38
>>268
手でやろうとしたらほとんど無理

270 名前:132人目の素数さん:04/10/05 20:38:52
>>248
答えおながいします。

271 名前:132人目の素数さん:04/10/05 21:36:01
>>268
右辺=n(n+1)(2n+1)/6 で n,n+1,2n+1 はどの2つも互いに素なので、
約分したあとそれぞれ a,b,c になったとすると、それらは全てが平方数に
なっていなければならない。

272 名前:132人目の素数さん:04/10/05 21:41:02
なるほど。そこからは?

273 名前:132人目の素数さん:04/10/05 22:08:51
>>270
未解決とぐぐったらでましたが?

274 名前:132人目の素数さん:04/10/05 22:11:55
>>273
>>248が?

275 名前:数学科布施 ◆FUSEz5Eqyo :04/10/05 22:12:57
>>248は数オリの問題だな

276 名前:132人目の素数さん:04/10/05 22:14:15
前半は解決しただろ。>>267参照。

後半は未解決だ。

277 名前:132人目の素数さん:04/10/05 22:19:38
>>275
>>248の後半も答えもってんの?

278 名前:数学科布施 ◆FUSEz5Eqyo :04/10/05 22:22:25
>>277
ちょ、ちょっと待ってね。過去問収集が趣味で、探すのが大変ほど溜まってる。
似た問題だけかもしれないけど探してみる。
俺の記憶は後半のほうがすごい見覚えあるんだよなぁ

279 名前:132人目の素数さん:04/10/05 22:23:04
>>277
『数学セミナー』2001年5月号 pp.91-94
>>275
@だろ?

280 名前:132人目の素数さん:04/10/05 22:25:05
>>276
後半だ

281 名前:数学科布施 ◆FUSEz5Eqyo :04/10/05 22:25:06
>>279
そのころの数セミもってねぇぇえl!!!

282 名前:132人目の素数さん:04/10/05 22:25:48
>>281
コピペしてググれ

283 名前:132人目の素数さん:04/10/05 22:26:24
>>279
それ答えももってたらうpしてちょ

284 名前:132人目の素数さん:04/10/05 22:26:33
http://www.geocities.jp/ikuro_kotaro/koramu/hutou1.htm
4面体では
  R1+R2+R3+R4≧√8(r1+r2+r3+r4)
  R1R2R3R4≧81r1r2r3r4
であることが示されています.
====

未解決ではないらしい。
解決方法は書いてないな。

285 名前:132人目の素数さん:04/10/05 22:27:50
http://homepage2.nifty.com/PAF00305/math/triangle/node7.html#tex2html6
>>284
すまんここみてた。予想とあったから。

286 名前:132人目の素数さん:04/10/05 22:29:27
>>283
前半は書いてあるが後半解説なし

287 名前:132人目の素数さん:04/10/05 22:30:20
Kazarinoffこれって何て読むの?

288 名前:284:04/10/05 22:30:25
まぁ、俺が張ったアドレスの方が間違いって言う可能性もあるんだし、
とりあえず、証明出てくるまでは、2ch的未解決っていうことで。

289 名前:132人目の素数さん:04/10/05 22:31:15
>>286
前半は>>267であってたの?

290 名前:132人目の素数さん:04/10/05 22:32:30
>>289
解答は長くなるから書かないけど三角関数使って大体そんな感じ

291 名前:132人目の素数さん:04/10/05 22:33:18
>>287
かざりのふ?

292 名前:132人目の素数さん:04/10/05 22:35:22
前半の証明では、途中四角形が円に内接する事を使ってるよな。
そのままじゃ、四面体に応用できないか……

293 名前:132人目の素数さん:04/10/05 22:37:39
mathnori上位者がここ見てたら挑戦してくれ!!

294 名前:132人目の素数さん:04/10/05 22:40:14
>>292=エコブラックだけど、無理

295 名前:132人目の素数さん:04/10/05 22:46:39
>>286
後半は参考書の提示もないの?
一切解説無し? 問題文だけ?

296 名前:132人目の素数さん:04/10/05 22:50:53
解説ないよ。ごめんな。紹介だけ
エコブラックなら数日のうちに解決しそうだ。頼んだ。

297 名前:132人目の素数さん:04/10/05 22:51:18
>>248が気持ちわるいのは直感的には答えは>>267と同様等号成立は
正四面体の重心のときっぽいんだけど実際その場合やってみると
等号成立しないよね?だからな〜んか気持ちわるい。√8じゃなくって3なら
わかるんだけど。3だと反例あるんかな?

298 名前:132人目の素数さん:04/10/05 22:52:19
エコブラックって何?

299 名前:132人目の素数さん:04/10/05 22:53:36
>>296
誰の出題かだけでも教えてくれ。 それでなにがしかのヒントになるかも知れない。

300 名前:132人目の素数さん:04/10/05 23:03:49
リンク先のページに文献とあるが

301 名前:132人目の素数さん:04/10/05 23:07:38
すまそ、とりあえず、Kazarinoff, tetrahedronでぐぐってみた。
色々文献が出ているので、調べてみる。

302 名前:数学科布施 ◆FUSEz5Eqyo :04/10/05 23:47:54
あ〜だめだ解答見つからん。
S≧T√8ってすげぇ見覚えあるんだけどな。
難問だね。

303 名前:132人目の素数さん:04/10/06 17:17:28
このヘタレ! ___ オラッ!      
    ドッカン |   | でてこい、エコブラック!
    ∩∩  |   |   |  ∩∩    
   | | | |  |   |   |  | | | |    
  ..(  ,,) .|   |   | (・x・ )  
  /  .つ━━ロ|ロ ドカン l   |U 
〜(   /   |   |   |⊂_ |〜
  し'∪   |   |   |   ∪ 

304 名前:132人目の素数さん:04/10/06 17:40:49
わかんね、つってんだろ。

305 名前:132人目の素数さん:04/10/06 21:07:17
>>248教えれあげ。ついでにエコブラックって何?

306 名前:132人目の素数さん:04/10/06 21:43:44
エコアザラクの息子だよ

307 名前:132人目の素数さん:04/10/06 21:50:34
マジレスするとエコピンク

308 名前:132人目の素数さん:04/10/06 21:55:37
じゃおれエコイエロー

309 名前:132人目の素数さん:04/10/06 22:07:21
リーダーはグリーンだ

310 名前:132人目の素数さん:04/10/06 22:10:39
2ちゃんねるに環境板があったのか

311 名前:紺ヲタ:04/10/06 23:31:17
これ難しいね

312 名前:132人目の素数さん:04/10/06 23:45:17
エコレッド:自己中心。
エコブルー:冷静沈着。
エコイエロ:デブヲタ。
エコグリン:頭数合わすためのオマケ。
エコピンク:公衆b…

313 名前:132人目の素数さん:04/10/06 23:48:48
エコブラックの立場が・・・いれてやれよ

314 名前:132人目の素数さん:04/10/06 23:50:28
ちげー

エコグリーン:環境戦隊エコレンジャーのリーダー
エコレッド :男気あふれる熱血漢
エコブルー:クールな二枚目戦士
エコイエローとエコピンクは途中参加の戦士達だ。

315 名前:132人目の素数さん:04/10/06 23:50:47
エコブラク:Noriを小馬鹿にする天狗。

316 名前:132人目の素数さん:04/10/06 23:54:36
別掲示板で(゚∀゚)キターと叫んでいる人がいますよ

317 名前:132人目の素数さん:04/10/06 23:57:45
どこでつか?

318 名前:132人目の素数さん:04/10/07 01:55:53
実況板にいけばいくらでもいるがそういうこと?>316


319 名前:132人目の素数さん:04/10/07 21:59:42
>>318
つまらん8点

進展あったなら誰か書き込んでくれ

320 名前:132人目の素数さん:04/10/07 23:07:53
2つの三角形があり、3つの角が等しく(つまり相似)、
3辺のうち2つの長さが等しいという。
これら2つの三角形は合同と言えるか。

321 名前:320:04/10/07 23:10:59
ちょっと意味不明だったので書き直し。

2つの相似な三角形があり、
3辺の長さをそれぞれ (a,b,c)、(d,e,f) とすると
a=d、b=eが成立している。
これら2つの三角形は合同と言えるか。

322 名前:132人目の素数さん:04/10/07 23:29:58
r=3/2として
(1,r,r^2)、(r,r^2,r^3)は相似かつ2辺が等しいけど合同じゃない。

323 名前:320:04/10/07 23:49:50
お見事。

324 名前:132人目の素数さん:04/10/08 11:29:44
感動した!

325 名前:132人目の素数さん:04/10/08 14:12:26
腹減った!

326 名前:LettersOfLiberty ◇rCz1Zr6hLw :04/10/08 16:03:52
>325
うんちでも食ってろ!

327 名前:LettersOfLiberty ◆rCz1Zr6hLw :04/10/08 16:51:55
Re:>325 まだ飯食ってないのか?何やってんだよ。

328 名前:132人目の素数さん:04/10/08 18:00:14
>>327
うんち食ってろ

329 名前:LettersOfLiberty ◆rCz1Zr6hLw :04/10/08 18:42:29
Re:>328 お前は普段何している?

330 名前:132人目の素数さん:04/10/08 19:07:39
すごい勢いで蒸発する飲用液体があります。
それを2つのコップにわけます。
ひとつは100ml もうひとつは400mlだとします。
それぞれのコップから毎秒5ml蒸発します。
8人の人がどちらかにストローをつっこんで一斉に飲みはじめます。
それぞれは毎秒5mlで飲みます。
片方がなくなったら間髪いれずもう片方に移って飲みます。
飲み始めるまでの蒸発やストローの長さや
コップを移るロスなどは考えないとします。

その場合どういう人数配分でスタートするのが
もっとも8人合計で多くの液体を飲めますか?

また数値を変えた場合についてもわかるように
できれば法則化してください。

331 名前:LettersOfLiberty ◆rCz1Zr6hLw :04/10/08 19:24:23
Re:>330
100mlのコップにx人ついて、残りに8-x人つくとしよう。
他方に移らない場合は、20/(x+1)秒間で、100mlのコップがからになり、
80/(9-x)秒間で、400mlのコップがからになる。
20/(x+1)>80/(9-x)(⇔x<1)のとき、
80/(9-x)秒間でちょうど400mlのコップが空になり、他方のコップには(100-100/(x+1))mlの液体が残っている。
だから合計(400/(9-x)+800/9*x/(x+1))ml飲める。
20/(x+1)≤80/(9-x)(⇔x≥1)のとき、
20/(x+1)秒間で、100mlのコップが空になり、他方のコップには(400-400/(9-x))mlの液体が残っている。
だから合計(100/(x+1)+3200/9*(8-x)/(9-x))ml飲める。
これで、数値が最大になるところを求めればよい。

332 名前:LettersOfLiberty ◆rCz1Zr6hLw :04/10/08 19:32:12
Re:>330
訂正。
x<1のとき、(3200/(9-x)+8/9*(100-400(x+1)/(9-x)))ml飲めて、
x≥1のとき、(800/(x+1)+8/9*(400-100(9-x)/(x+1)))ml飲める。

333 名前:132人目の素数さん:04/10/08 19:42:14
さんくす

334 名前:132人目の素数さん:04/10/08 19:50:51
>>330
蒸発量を最小化すれば良いんだね
だから容量の少ないコップから先に飲み干せばよい
飲み干してしまえばもう蒸発しないからね

ということで
(1)最初の2秒間は100mlコップから
8人がかりで80ml飲む.10ml残して7人は
400mlコップに移動.
(3)3秒目は
残った1人は100mlコップから更に5mlのむ.
5ml蒸発するから100mlコップはこの時点で空になる.
移動した7人は400mlコップから35ml飲む
3秒間の累積蒸発量は15mlだから400mlコップの
残存量は400−15−35=350mlとなっている.
(4)4秒目以降は8人掛かりで400mlコップを飲みほす.
飲める量は350*40/45=約311ml

以上合計すると
80+5+35+311=431ml


335 名前:330:04/10/08 19:56:54
ありがとう
一つ目を空にしてしまうと蒸発しないけど
その分二つ目を飲みきるのが遅くなるから
結局同じだと思ってました

336 名前:334:04/10/08 19:59:22
>>334
自己レスだけど...

下の部分は2秒間に限定するのではなくて
8人がかりで100mlコップを飲み干して
しまう方が本当は飲める量を最大化できる
興味があったら計算してみて

> (1)最初の2秒間は100mlコップから
> 8人がかりで80ml飲む.10ml残して7人は
> 400mlコップに移動.


337 名前:330:04/10/08 20:00:34
hai

338 名前:132人目の素数さん:04/10/09 11:29:15
次の条件を満たすように、平面上に有限個の有界な凸集合を並べよ。
1) どの3つの凸集合も共通の内点を持つ。
2) ある二つの凸集合が存在し、それらは共通の内点を持たない。

339 名前:LettersOfLiberty ◆rCz1Zr6hLw :04/10/09 11:43:38
Re:>338 共通内点を持たない二つの有界凸集合を作ればいいのだな。

340 名前:132人目の素数さん:04/10/09 11:47:46
>>338
自明的に出来る。
A = B = 空集合と置けばよいb

341 名前:132人目の素数さん:04/10/09 11:48:12
>>338
共通内点を持たないような有界凸集合2個を
平面上に配置すればよい

凸集合が3個以上存在する場合は条件1)によって
条件2)の否定が成立するので,題意を満たす配置は不可能

342 名前:132人目の素数さん:04/10/09 11:48:39
これは
1) どの3つの凸集合も共通の内点を持つ。
という仮定を満たす。
集合が二つだから

343 名前:132人目の素数さん:04/10/09 20:07:35
>>338
ヘリーの定理

344 名前:132人目の素数さん:04/10/10 05:33:32
>>343
なんじゃ>ヘリーの定理?できないってのがヘリーの定理?
証明おながいします。

345 名前:132人目の素数さん:04/10/10 05:43:59
ヘリーの定理

--------------------------------------------------------------------------------
[定理](Helly) 平面上に、有限個の凸集合があり、これらのうちのどの 3つを選ん
でも、それらに共通な点が含まれるとすると、最初の凸集合すべてに共通な点が存在
する。

証明はごく普通の帰納法でできる。特別な知識は何もいらない。
難しいとすれば、たぶん、凸集合の定義だけ。(まあ、誰でも凸集合がどういうもの
であるかは知っている。ただ、「へこんでない形」というイメージだけにたよると
証明は難しいと思う。)
一応、凸集合の定義を書いておきます。

[定義] 集合Xは、Xの任意の2点を結ぶ線分が必ずXに含まれているとき凸集合と呼ば
れる

応用としては、次のような問題が考えられます。

[応用問題] 平面上に有限個の点 があり、どの3点を選んでも、この3点を半径1 の
円で覆うことができるならば、すべての点を 半径1 の円で覆うことができる。

応用問題の方は、メーリングリストでは話題になっていませんが、これからメーリン
グリストに参加しようと思っている人はちょっと考えてみてはいかが?

>>http://www.hamaint.co.jp/math/math_th_helly.html

346 名前:132人目の素数さん:04/10/10 05:57:30
>>http://coolee.at.infoseek.co.jp/convex.html#1

347 名前:132人目の素数さん:04/10/10 09:25:16
>>345
>証明はごく普通の帰納法でできる。特別な知識は何もいらない。
 
これそんなに簡単なんですか?4つのときいえればあとは帰納法ってのは
わかるけど。4つのときができん。だれか証明してたも。

348 名前:132人目の素数さん:04/10/10 09:35:33
4つで球面や凸はずした時の反例が浮かんだ。
>>346しこしこ読めば?

349 名前:132人目の素数さん:04/10/10 09:36:41
>>348
4つのときの証明はできたの?

350 名前:132人目の素数さん:04/10/10 09:40:18
>>346に証明のってるのか・・・結構難しいじゃん。

351 名前:132人目の素数さん:04/10/10 09:54:13
うん?帰納法だろ。4考えなくていいじゃん。

352 名前:132人目の素数さん:04/10/10 10:06:54
3で言える。
nで成立としてn+1で成り立つ事を示す。
ここから、これおもしろいね。
結局、n+1個でどの任意の3個じゃなくてどの任意のnもまで自然に行っちゃう
じゃん。

353 名前:132人目の素数さん:04/10/10 10:13:49
結局
「平面上に、(n+1)個の凸集合があり、これらのうちのどのn個を選ん
でも、それらに共通な点が含まれるとすると、(n+1)の凸集合すべて
に共通な点が存在する。」
を示せばいいんだな。

354 名前:132人目の素数さん:04/10/10 14:47:41
なるほど。で、それを>>346より簡単に証明できます?

355 名前:132人目の素数さん:04/10/10 19:37:29
>>353を平面である事と凸である事を使って示すのが、証明の残りだが、
ここで反例が存在するとして矛盾を示そう。
(n+1)個の凸集合に番号を付ける。A1,A2,,,,Ai,,,,An+1としよう。
全てのi(1<=i<=n+1,iは自然数)に対して、Ai以外のn個の凸には共通な点が必ず
存在するから、この点をaiと言う事にする。Aiはai以外の全ての点aj(1<=j<=n+1,i<>j,jは自然数)
を含んでいる。
ここでBiをR^2からAiを除いた集合とする。つまり、Biは平面から凸を除いた形になる。
しかもBiは点aiしか含まない。

続く。

356 名前:132人目の素数さん:04/10/10 23:39:50
続きがきになるあげ

357 名前:あぼーん:あぼーん
あぼーん

358 名前:132人目の素数さん:04/10/11 00:54:21
続きはどうした

359 名前:LettersOfLiberty ◆rCz1Zr6hLw :04/10/11 11:49:54
Re:>357 おい、それは私のメアドだぞ。勝手に載せるなよ。

360 名前:語っていただければ幸いです。:04/10/11 13:01:15
続き
後は以下を示せば良い。
1.点aiで図凸(n+1)多角形を構成できる。
そうでないなら、点aiで凸多角形と内点を得られるが、この内点のうちの一つ
の点だけを含まない様な凸図形が得られない(Ajのどれかでそれはあるはず。)。
2.この凸(n+1)多角形内にすべてのAiに含まれる点が存在する事を示す。
(これが明らかに矛盾になり、反例があるとした仮定は誤りになる。)
凸(n+1)多角形に対角線を2本引こう。その交点はこの対角線を構成する
4点に対応するAiすべてに含まれる。
対角線分をaiaj,akalとしてみると、AkとAlは凸性から、交点を含む。AiとAjも同様。

もう一息です。続く。(2が終了してない。)

361 名前:あぼーん:あぼーん
あぼーん

362 名前:LettersOfLiberty ◆rCz1Zr6hLw :04/10/11 13:43:25
Re:>361 お前何やってんだよ?

363 名前:132人目の素数さん:04/10/11 19:39:29
続き(2の証明)
対角線の話だけではn=3つまり凸4角形についてしか2を言えない。
凸5角形については対角線をすべて引いてできる星型の中央にできる凸5角形がこの
凸4角形の交点に相当する。この中央の凸5角形の一つの辺に着目するとこの辺は全ての
Aiに含まれることがわかる。
凸(n+1)角形についてもn>=4以上の場合は同様に、その凸(n+1)角形を構成する周上の
連続する勝手な5点を選び、これをai,aj,ak,al,amとした時に(周上にこの順で並んで
いるとする。)対角線aiakとajakの交点をb1,amakとajakの交点をb2とすれば、
線分b1b2が全てのAiに含まれる事が納得できる。
したがって2が示された。

証明了(集合Biは1を示す時に使うと了解しやすいが、なくても証明できる。)

364 名前:訂正:04/10/12 13:37:44
>>363の訂正
対角線の話だけで凸(n+1)角形についても言える事に気がつきました。

凸(n+1)角形を構成する周上の連続する勝手な4点を選び、これをai,aj,ak,al
とした時に(周上にこの順で並んでいるとする。)対角線aiakとajalの交点は
全てのAiに含まれる。
したがって2が示された。

定理をとおさなくても、直接問題を解いた方が、(わざわざその定理を証明する
なら)わかりやすかったと思います。
質問された方こんな説明でよかったでしょうか?

365 名前:132人目の素数さん:04/10/12 14:50:26
と言う訳で>>338を見たのだが、何これ?

どの3つも満たすなら、当然どの2つもみたすよな?意味不明。

まあ、いいか。ヘリ―の定理はおもしろかったから。

366 名前:132人目の素数さん:04/10/12 15:08:36
>>346よりちっとも簡単になってないような…

367 名前:132人目の素数さん:04/10/12 15:11:46
平面上に、有限個の凸集合があり、これらのうちのどの 2つを選ん
でも、それらに共通な点が含まれている時、最初の凸集合すべてに共通な点は存在
するか?しない。三角形の各一辺だけを含む凸集合を思い浮かべればよい。

でも3つにした途端、、

368 名前:132人目の素数さん:04/10/12 15:32:42
正直、もっと簡単に証明したいよ。>>346は読んでないが。

369 名前:LettersOfLiberty ◆rCz1Zr6hLw :04/10/12 15:48:22
それでは凸の条件をはずすとどうなるか?
連結でない場合は明らかに反例ありだが、
連結な場合はどうだろう?
(ヘリーの定理について。)

370 名前:LettersOfLiberty ◆rCz1Zr6hLw :04/10/12 15:56:07
/*少し考えたら出来た。*/
#include<stdio.h>
#include<stdlib.h>
main(){
FILE *f1;
int c[372]={
137,80,78,71,13,10,26,10,0,0,0,13,73,72,68,82,0,0,0,64,
0,0,0,64,1,3,0,0,0,144,167,227,157,0,0,0,6,80,76,84,
69,0,0,0,255,255,255,165,217,159,221,0,0,1,41,73,68,65,84,120,
156,189,209,63,74,3,65,24,5,240,1,11,11,145,28,97,241,8,214,130,
115,17,11,239,96,161,32,56,118,41,211,6,82,236,25,172,68,155,41,68,
108,148,37,149,69,140,19,8,178,77,200,26,22,156,89,178,59,207,247,205,
138,133,7,112,6,134,31,204,255,247,41,252,52,37,195,117,214,227,211,148,
130,69,28,160,17,228,7,64,39,48,5,16,137,136,130,115,6,170,195,107,
179,132,134,218,98,178,23,96,5,227,77,52,14,202,227,241,11,61,62,74,
163,19,166,133,181,149,224,45,179,214,19,113,31,61,154,27,56,91,115,187,
191,35,182,196,206,125,143,118,118,107,18,194,172,48,86,80,31,185,145,181,
29,119,77,178,67,107,4,167,171,19,35,168,46,55,239,199,61,194,116,140,
132,122,240,144,112,230,29,18,206,203,191,24,121,155,176,214,97,55,97,229,
112,1,180,80,47,14,87,154,127,86,243,42,34,79,240,29,156,76,205,195,
146,104,8,100,4,31,255,204,236,114,240,59,107,194,162,36,12,152,78,193,
43,12,162,145,124,136,22,18,93,165,177,17,170,42,199,144,132,42,139,160,
241,68,248,197,46,90,77,212,60,103,41,213,97,250,188,66,146,215,224,34,
169,197,144,61,33,40,253,91,193,255,198,55,25,93,194,199,47,132,109,70,
0,0,0,0,73,69,78,68,174,66,96,130};long i;
if((f1=fopen("lettersoflibertynoekaki20041012.png","wb"))==NULL){puts("fopen error");exit(1);}
for(i=0;i<372;++i){putc(c[i],f1);}
fclose(f1);
return 0;
}
/* これをコンパイルして実行すると、同一フォルダにPNGファイルができる。 */

371 名前:132人目の素数さん:04/10/12 18:37:37
実数列f(1),f(2),f(3),…,f((2n−1)!!)がある。この中から、
(広義)単調増加または(広義)単調減少するn項の部分列が選べることを
示して下さい。

372 名前:挑発筋肉 ◆POWERPUfXE :04/10/12 18:51:07
>>371
数オリ?鳩ノ巣原理かな

373 名前:132人目の素数さん:04/10/12 19:11:17
>>372
離散数学の本に、時々載っている有名(?)問題です。本当は(2n-1)!!個も
必要無いのですが、私の趣味で(2n-1)!!に変えました。

374 名前:132人目の素数さん:04/10/13 00:41:34
ある凸多面体はどの方向への正射影も三角形、または四角形になるという。
この多面体は四面体であることを示せ。

375 名前:132人目の素数さん:04/10/13 00:42:14
>>371
http://web2.incl.ne.jp/yaoki/zgseq.htm

376 名前:132人目の素数さん:04/10/13 07:05:24
○| ̄|_ =3 ブッ

377 名前:132人目の素数さん:04/10/13 15:12:44
毎時平均10個の流星が流れる流星群があるとする.
(流星は互いに関係なくランダムに,すなわち定常ポアソン過程として流れる).
事実:ある流星が流れてから次の流星が流れるまでの平均時間は6分である.

A氏は夕方からこの流星群を観測し,流れた時刻をノートにつけた.
B氏は夜中0時からA氏の隣で同じ流星群を観測した.

Q1.B氏が始めに観測する流星の時刻の期待値は何時何分か?
Q2.A氏がB氏が来る直前に記録した流星の時刻の期待値は何時何分か?
Q3.0時を挟んだ2つの流星の間隔の期待値は何分か?
Q4.Q3の答は上の「事実」の間隔と異なるか?異なるとしたらなぜか?

378 名前:132人目の素数さん:04/10/14 18:16:30
Q1. 0時6分
Q2. 0時6分前
Q3. 6分
じゃないの?
事象が無相関でも、その間隔がある確率分布に従うなら
<t1+t2>≠<t1>+<t2>なのは不思議じゃないと。

379 名前:132人目の素数さん:04/10/14 23:06:28
>>378
Q3は12分だろ。

380 名前:132人目の素数さん:04/10/14 23:48:38
>>378
Q3.12分。
流星の間隔は平均6分だけど、0時の前後の流星の間隔は倍の平均12分。
べつに1時でも2時でもいいけど。

381 名前:132人目の素数さん:04/10/15 00:36:43
で、Q4なわけか。
なぜそうなるの?

382 名前:132人目の素数さん:04/10/15 01:00:51
ある時刻より前に記録した流星の時刻からその時刻までは流星が流れていない、という条件がつくからね。その分、平均時間が増える、って感覚。
その増えた分がQ2であり、ちょうど2倍になる。

具体的に密度関数を書けば、時間間隔Tの分布f(t)は、
通常: f(t)=λe^(-λt)
ある時刻をはさむ: f(t)=(λ^2)te^(-λt)
となる。後者は結局、これから2個目の流星が流れるまでの時間の分布と同じで、ガンマ分布になる。

383 名前:132人目の素数さん:04/10/15 01:11:14
>>382だけど、λ=10ね。平均時間は1/λ(時間)=6分。

384 名前:378:04/10/15 08:59:43
Q3は0時の時点での期待値、という意味なのかな。
すでに流星の流れた時刻のデータがあって、任意にある時刻をえらぶ。
直前の流星からその時刻までの時間の期待値、その時刻から次の流星まで
の期待値、その時刻をはさむ2つの流星の間隔の期待値ならば全部6分だよね。

385 名前:132人目の素数さん:04/10/15 10:25:56
>>384
あんまり理解できてないようだな。

例えば1年間取り続けて、流星の流れた時刻のデータが、いっぱいたまったとしよう。
で、時刻を固定せずに、間隔だけを見てそのすべての平均をとれば6分。
ある特定の時刻をはさむ時間間隔だけ抽出して、その平均をとれば12分になる。

もちろん、厳密には開始時間と終了時間が有限だから多少誤差がでるけど。

386 名前:384:04/10/15 14:01:06
>>385
あー、やっとわかった。>>382は、
物差しがたくさんあって、その長さの分布(密度関数)が
f(t)=λe^(-λt)
時刻をランダムに決めると、長さtの物差しにあたる確率は
t*f(t)に比例するはずで、規格化すれば特定の時刻を含む物差しの
密度関数は、
g(t)=(λ^2)te^(-λt)
ってことか。

387 名前:132人目の素数さん:04/10/15 14:38:10
>>382 >>386
正解です。

「0時」とか「Bが来たとき」とか「電話が鳴ったとき」とか流星と無関係な条件がつくと、
それだけで長い間隔にひいきがあってtがかかる、ともいえるし、
こういう事象が流星が流れたのと変わらず、Q3が2回分の間隔を求めているともいえる。
というわけで密度関数は2次のアーラン分布 f(t)=(λ^2)te^(-λt) になります。
最初気づいたときは直観に反する気がしてずいぶん悩んだけど。

388 名前:132人目の素数さん:04/10/15 16:34:46
xについての複素係数多項式、f(x),g(x)を考える。
F(k) = { x | k=f(x) , x∈C }  G(k) = { x | k=g(x) , x∈C }
と定義し、F(0)=G(0)かつF(1)=G(1)ならばf(x)=g(x)が恒等的に成立することを示せ。


389 名前:132人目の素数さん:04/10/15 17:00:12
半径 1 の円に内接する、正 $n$ 角形がある。このと
き、

(i) 正 $n$ 角形の面積が最大になるのはどういうときか?
(ii) 正 $n$ 角形の周の長さが最大になるのはどういうときか?

390 名前:LettersOfLiberty ◆rCz1Zr6hLw :04/10/15 17:03:12
Re:>389 どんな問題だよ?

391 名前:132人目の素数さん:04/10/15 17:10:59
>>389
半径1の円に内接する正n角形って……
ほとんど自由度がないよね。簡単すぎ、

1) 最大値は存在しない。 nに対する単調増加
2) n=3の時、最小値

392 名前:132人目の素数さん:04/10/16 00:07:08
任意の自然数nに対して、(n^2)!/(n!)^nは自然数になることを示して
下さい。

393 名前:132人目の素数さん:04/10/16 11:01:15
>>392
=Π[k=0 to n-1](C[n(n-k),n])
C[m,n] (m≧n)が自然数であることの証明もいる?

394 名前:132人目の素数さん:04/10/16 15:39:36
>>393
おお… 正解です。一瞬で見破られてしまいました。

395 名前:132人目の素数さん:04/10/17 11:26:56
age

396 名前:132人目の素数さん:04/10/17 13:26:47
>>369
楕円型に3個の集合をとり、(ちょうど三菱を太らせた様に)
最後に平面から円を抜いた型をここに当てはめれば
これでできあがり。

397 名前:132人目の素数さん:04/10/17 13:35:46
ロジスティック写像について

0 < x_0 < 1
3 < r < 4
のとき、

x_(n+1) = r * x_n * (1-x_n) (n≧0)

としたとき、x_n の一般式あるいは誤差を最小限におさえるような一般項を求めよ。

398 名前:あぼーん:あぼーん
あぼーん

399 名前:LettersOfLiberty ◆rCz1Zr6hLw :04/10/17 13:41:26
Re:>398 人のメアドを勝手に載せるな。

400 名前:カオス:04/10/17 13:51:35
収束すれば、{rx+(1-r)}x=0で
x=0,r/(r-1)これ収束条件とかあんの?

401 名前:132人目の素数さん:04/10/22 07:50:36
線形代数のスレに載せたがレスが無いのでこちらへ

n 次実正方行列 A, B が A^2 = B^2 = (AB)^2 = -E
を満たすなら n は 4 の倍数


402 名前:132人目の素数さん:04/10/22 09:34:36
419

403 名前:132人目の素数さん:04/10/22 10:30:50
>>402アホ厨

404 名前:132人目の素数さん:04/10/22 16:58:08
同感です。
カウント厨は、コテハン名乗れ!
アボーンしやすいから

405 名前:132人目の素数さん:04/10/23 06:13:37
すごいな、
夕方に上がってたのに
無茶苦茶沈んでたぞ!

406 名前:132人目の素数さん:04/10/24 16:31:36
2ch はお日様と反対

407 名前:132人目の素数さん:04/10/24 16:33:52
2ch はお日様と反対


408 名前:132人目の素数さん:04/10/24 16:51:08
真似するな

409 名前:132人目の素数さん:04/10/29 13:57:29
1+2+3+4+,,,と
1+1/2+1/3+1/4+,,,はanと1/anが∞に発散する例になるが、
この他には
1+1+1+,,,,しか思い浮かばない。
他に例があればそれを希望。
なければ、できればそれを示してください。
すいません。純粋なただの単なる疑問です。

410 名前:132人目の素数さん:04/10/29 14:00:47
n*log(n)
n*log(n)*log(log(n))
....

411 名前:132人目の素数さん:04/10/29 14:08:10
thnx
1+2^0.5+3^0.5+,,,,
1+1/2^0.5+1/3^0.5+,,,,
-1<d<1
1+2^d+3^d+,,,,以外でおながいします。

412 名前:132人目の素数さん:04/10/30 09:01:00
>>409
単に
log n

413 名前:132人目の素数さん:04/10/30 13:01:45
亀?115が解けた。
8個の玉をA:3 B:3 C:2に分けて、AとBを比べる。
釣り合わなかったら136の通り。
釣り合ったら、下の二つのうちどちらかである。
(1) A:○○●,B:○○●,C:○○
(2) A:○○○,B:○○○,C:●●
次に、Aから2個、Bから2個とって比べる。
 また釣り合ったらAとBの残ったやつとCの二つを比べる。
 釣り合わなかったら、重かった二つを比べる。
ふう。

414 名前:132人目の素数さん:04/10/30 13:57:16
>>413
>次に、Aから2個、Bから2個とって比べる。
ここで、○●/○●でつりあったら、後1回では特定不可能では。
というか、可能性は2C8=28とおりあって、天秤で分離可能なパターンは
3^3=27パターンなんだから元々無理なのでは。

415 名前:409:04/10/31 11:44:40
anは収束、Σanが∞に発散をさがせば良い。事はわかった。

416 名前:132人目の素数さん:04/11/02 20:41:45
>>415>>409
で、あなたに見つけた例は?
つまらんもの書くなよ。

417 名前:132人目の素数さん:04/11/02 20:59:42
単純に、0<lim[n→∞]an<∞ なら 0<lim[n→∞](1/an)<∞ で
Σan と Σ(1/an) はともに発散するんじゃねーの?

418 名前:132人目の素数さん:04/11/02 22:49:03
はっさん、にじゅうし


あってる?

419 名前:132人目の素数さん:04/11/02 22:51:26
Hassanに住し、
発散に重視、
8324、

420 名前:132人目の素数さん:04/11/02 22:52:27
あってんじゃん。

421 名前:132人目の素数さん:04/11/02 22:57:48
>>417
an=1/nを考えてて似たのをさがしてた。これはもう終わりにしよう。
スレ違いだしな。

422 名前:132人目の素数さん:04/11/07 21:37:50
169

423 名前:132人目の素数さん:04/11/08 22:06:25
二つの円に関するポンスレーの閉形定理は二つの楕円でも成立する。

424 名前:132人目の素数さん:04/11/11 03:09:39
851

425 名前:132人目の素数さん:04/11/11 03:15:44
http://www.google.com/search?q=%83%7C%83%93%83%58%83%8C%81%5B%82%CC%95%C2%8C%60%92%E8%97%9D&hl=ja&ie=Shift_JIS&lr=
なんだこれは。googleめ間違った事教えるんじゃねぇ

426 名前:132人目の素数さん:04/11/11 10:59:45
>>425
ロイターとか、婦女子とかで検索すると結構面白い事に

427 名前:132人目の素数さん:04/11/11 17:56:52
>>426
おもしろいw

428 名前:132人目の素数さん:04/11/11 18:06:45
驚いたー

429 名前:132人目の素数さん:04/11/11 23:10:46
>>426
「ロイター」「婦女子」「就職しない」「ふたりはプリキュア」

いずれも「もしかして:」が無くなってたよorz

430 名前:132人目の素数さん:04/11/12 02:46:58
>>429
全部あるよ?

431 名前:132人目の素数さん:04/11/12 04:26:56
「就職しない」に対して
もしかして: 就職できない
はおもしろ過ぎるな。

432 名前:132人目の素数さん:04/11/12 04:30:34
漏れはふたな(ryにビクーリ
>>429
googleの検索結果はどうも時(とgoogle側サーバ?)によるみたいだから。

433 名前:workinmg woman:04/11/13 13:05:49
私には当てはまらないわね。
(当然だけど)

434 名前:132人目の素数さん:04/11/14 20:10:00
この問題、どうやって解くんだ?


問題:
厚さが2cmのカーペットがある
これを直径1mの円筒の容器に丸めて入れたい
最大で何m分のカーペットをこの容器に収める事が出来るか?

円筒の深さとカーペットの横の長さは一致しててぴったり収まるものとして、そこは考慮にいれなくていい
問題は、丸めて入れられるカーペットの縦の長さだ

誰か解き方を教えてくれ

435 名前:434:04/11/15 00:16:48
あ、俺が馬鹿だったかも
ぴったり正確な値はわからなくても、だいたいこのくらいって予測は簡単に出来るや

436 名前:132人目の素数さん:04/11/15 02:31:13
かも?

437 名前:132人目の素数さん:04/11/15 05:43:00
小学校の問題です。
立方体ABCD-EFGHがあってBCの中点をM,CDの中点をNとする。
このときMNGを通る平面で立方体を切断した時の三角形MNGの面積を求めよ。
三平方の定理は使えません。解答が物凄いやり方なので驚きました。


438 名前:132人目の素数さん:04/11/15 06:18:25
√5って使えんのか?

439 名前:132人目の素数さん:04/11/15 06:45:15
>>437
一辺1とする。

たぶん求める三角形を底面と見たときの三角錐の高さが1/3ってことを出して、
そこから求める三角形の面積を逆算するんだろう。

でもその1/3の求め方が分からない。
(このレスの1/3は三平方使って計算した)

440 名前:132人目の素数さん:04/11/15 06:48:47
>>438
答えに√は出てこないよ。

441 名前:132人目の素数さん:04/11/15 09:38:43
>>437
ありがちな問題だな.
△MNG≡△MNAを見抜けばよい.

442 名前:439:04/11/15 15:22:25
>>441
なるほど! 一発だな! すげー。

俺の予想は全然外れたわけか。

えーと、じゃあ、
「点Cから△MNGに垂線を下ろしたときの交点をPとする。
 CPの長さを求めよ。」
だったら更に奥が深くなるわけかな?

443 名前:132人目の素数さん:04/11/15 18:31:52
f:R→Rは連続である。また、任意のx,y∈Rに対して
f(x+y)={f(x)+f(y)}/{1+f(x)f(y)}が成り立っている。このような
f(x)を全て求めて下さい。

444 名前:132人目の素数さん:04/11/15 18:34:08
>>443
宿題は自分でやれ!
氏ね!!

445 名前:132人目の素数さん:04/11/15 19:34:06
>443
tanの加法定理だね。それ。

446 名前:132人目の素数さん:04/11/15 19:39:51
で、>>434の解き方は?

447 名前:132人目の素数さん:04/11/15 19:47:56
>>443
http://science3.2ch.net/test/read.cgi/math/1055809155/80
答はこの後を探せ。

448 名前:132人目の素数さん:04/11/15 19:48:43
f:R→Rは連続で、任意のx,y∈Rに対してf(x+y)={f(x)+f(y)}/{1+f(x)f(y)}
x=y=0をまずとるだす。
f(0)=2f(0)/{1+f(0)^2}だす。
整理して、f(0)=0 or f(0)^2=1だす。
x=x,y=0とるだす。
f(x)={f(x)+f(0)}/{1+f(0)f(x)}だす。
いろいろやってみるだす。

449 名前:437:04/11/15 21:27:35
切断した立体のうちCを含むほうの展開図を考え、正方形から周りの三角形
の面積を引くというのが解答に載ってたものでした。面積を求めるのに
展開図を使うという発想がこの問題以外で通用するのか謎です。



450 名前:132人目の素数さん:04/11/17 10:25:00
>>446
カーペットが変形しても体積が変わらないとすると
2cm*x=1m*1m*πとなるようなxが上限なのは確か。
だけどカーペットはどんな変形が出来るのか分からんから
答えがxよりどれだけ小さくなるのかワカラン。
何でもありならxという限界まで、いやそれを超えて詰め込めるけど。

451 名前:132人目の素数さん:04/11/17 10:56:25
>>449
>切断した立体のうちCを含むほうの展開図を考え、正方形から

切断した立体の展開図が正方形になるってどうして分かるの?

452 名前:132人目の素数さん:04/11/17 12:13:23
数学で計算問題とか解いてるときお前等頭の中で000+000だから00000000になるなとか思いながらといてんの?
というか頭の中で言語にしてから問題といてんのか?

453 名前:132人目の素数さん:04/11/17 20:10:39
計算問題だと有る程度は言葉にしてるんじゃないかな
というか手続きとして覚えている、といった方が適切かも

ちなみにどういう計算問題をいってるのか?
ラマヌジャンのノートに出てくる訳わかんない級数の和を求めるのも
「計算問題」ではあるが

454 名前:132人目の素数さん:04/11/18 02:09:13
>>451
>>449ではないが
△MNCの直角二等辺三角形の、MC=NCを1とすると
△MCGと△NCGは直角をはさむ辺が1、2の直角三角形となる
3つの三角形を展開図になるように組みたてると
都合よく一辺2の正方形におさまる。

 C  N  C

 M

 C     G

CMCの角が180度になることの証明は
小学生向けだとどうやればいいのかな…
答えを知ってないと正方形になることを思いつかないかもしれない

455 名前:454:04/11/18 02:18:12
おっと、証明も何も
>>441が全てを物語ってますね。

456 名前:132人目の素数さん:04/11/18 03:28:09
そう、展開図が正方形になることは>>441を使わないと分からないよな?
んで>>441を使うのであれば「展開図」なんて回りくどいものを考えなくても一発で答えは出る。
>>449の模範解答は疑問だ。

457 名前:132人目の素数さん:04/11/18 04:49:11
なるほどな。(441がおもしろい)
展開図がそうそう正方形にきちんとなる訳ではない。
つまり、ある種のひらめきで解ける特殊例なんだな。一般例ではない。

458 名前:132人目の素数さん:04/11/20 03:42:25
本来は合同を使って正方形ABCD内の問題にできるところを
合同を使わずに何とかしようとして
苦肉の策が(結果的にABCDと合同になる)展開図なのかな。

 >>449>面積を求めるのに
     >展開図を使うという発想がこの問題以外で通用するのか謎です。
への答えは
 >>457>特殊例なんだな。一般例ではない。
ということになるのか。

他に展開図が綺麗な形になって断面積を求めることができるケースってあるかな。

459 名前:132人目の素数さん:04/11/20 21:46:59
M,Nが中点なのがミソ。少しずらせば、たちどころに、きれいにはならないのがわかる。
このタイプの他の特殊例ね。ないとは言えないや、そこが(出題者も含めた)ひらめき
ではないかと、、、。

460 名前:132人目の素数さん:04/11/26 21:53:03
正方形ABCDの内部にPA=1, PB=2, PC=3となる点Pが取れるとき、
正方形の一辺の長さを求めよ。

461 名前:132人目の素数さん:04/11/26 22:16:59
>>460
x^2+y^2=4 と
x^2+(y−a)^2=1 の第一象限の交点の座標をaを用いて表して、
その座標と(a,0)との距離=3 の方程式を解けば終了?

462 名前:132人目の素数さん:04/11/27 14:33:38
辺が1、2の直角三角形の対角線の長さに違いは無いということで、
点Gを点Aに移せれば氷解なんだな。空間把握能力でも試す問題か?

463 名前:132人目の素数さん:04/12/01 23:08:18
面白い問題教えてくれ

464 名前:132人目の素数さん:04/12/03 02:24:59
>>460
√(5+2√2)

465 名前:132人目の素数さん:04/12/03 22:13:12
Σ[n=1,∞]n^2/2^n を求めよ。

466 名前:132人目の素数さん:04/12/08 10:56:00
>>465
標準的すぎてつまらん

467 名前:132人目の素数さん:04/12/10 05:07:52
>>413
>>115のヒントをみると…ずるい。

同じ重さの軽い玉A個と
同じ重さの重い玉B個
合計(A+B)個があるとき

ABの数字は与えられてるとき
天秤をつかってすべての玉について重い玉か軽い玉かを判別するための
最小回数N(A、B)を求めよ
という一般形になるかな。

N(1,1)=1
N(1,2)=1
N(1、3)=2
N(2,2)=2
N(1,4)=2
N(2,3)=3

N(2,6)=4 4回で出来る方法はわかるが3回でできないことは証明してない

一般のN(A、B)を数字ABをつかった式で表わせるだろうか

468 名前:132は素数ではない:04/12/11 23:01:43
3次関数f(x)=ax^3+bx^2+cx+dがある。
f(x)が極小値と極大値を持つとき、極小となる点と、極大となる点を結ぶ
直線の方程式を求めよ。
工房にはできないヤシ多いだろうな。


469 名前:132は素数ではない:04/12/11 23:02:49
hage


470 名前:132人目の素数さん:04/12/11 23:09:59
>>468
f(x)をf’(x)で割った余り

471 名前:132人目の素数さん:04/12/11 23:26:56
(゚д゚)ファイ?

472 名前:132人目の素数さん:04/12/12 16:22:15
余りに決まってるだろうが

473 名前:132人目の素数さん:04/12/12 17:39:06
l:y={(f(a)-f(b))/(a-b)}(x-a)+f(a)

f'(a)=f'(b)=0
f(x)=f'(x)(px+q)+rx+s
f(a)-f(b)=r(a-b)⇔r=(f(a)-f(b))/(a-b)
f(a)=ar+s⇔s=f(a)-ar

474 名前:132人目の素数さん:04/12/15 00:07:35
記号ばかり使うんじゃなくて、ケイタイでも簡単に出題・解答出来る問題ってありますか?
1から2000の総和を求めよ、とかそのぐらいの易しいレベルの

475 名前:伊丹公理:04/12/15 00:17:29
自分でケータイ専用のスレ立てろ

476 名前:132人目の素数さん:04/12/15 00:22:54
>>474
携帯サイトで探せよ

477 名前:132人目の素数さん:04/12/15 00:34:00
>>474
>1から2000の総和を求めよ、とかそのぐらいの易しいレベルの

( ´,_ゝ`) プッ
そんな単純計算が面白いの?
おまえはココ立ち入り禁止な!
一人でシコシコやってろ! プケラ。

478 名前:132人目の素数さん:04/12/15 00:51:04
次の命題の真偽を判定し、それを証明せよ

「年末ジャンボ宝くじで5000万円以上当たる確率は、
連番で買ったときよりもバラで買ったときの方が高い。」

なお、当選本数はリンクの通りとする。
ttp://www.takarakuji.mizuhobank.co.jp/topics1.html

479 名前:132人目の素数さん:04/12/15 01:20:56
>>478
報酬があるならやってやるよ

480 名前:132人目の素数さん:04/12/15 01:23:03
>>478
年末ジャンボ33枚、約1万円分くれたら解いてやってもいい。

481 名前:132人目の素数さん:04/12/17 05:40:31
>>478
1等、前後賞、2等が当る確率?

それぞれは5000万未満でも
合計金額が5000万以上になる確率?

買った値段を差し引いて
利益が5000万以上になる確率?

482 名前:132人目の素数さん:04/12/17 05:57:27
>>481
2番目のでしょう。

483 名前:132人目の素数さん:04/12/17 12:14:54
x_0〜x_nについて次の操作を考える。
(1) x_a=x_b=N となるa,bがあったら x_a=N-1、x_b=N+1 と変化させる。
(2) ( a≠b → x_b=M and x_a=N<M ) となるaがあったら
  x_a=N+1と変化させ、a≠bとなるbに対しては x_b=M-1 と変化させる。

最初x_0=x_1=…=x_n=0となっている状態から(1),(2)の操作を何回か行なう時、
max|x_i-x_j|の取りえる最大値を求めんさい。

484 名前:132人目の素数さん:04/12/17 23:56:29
>>483
(2)の操作がよくわからない。
bが特定の元を指すのか、全てなのか。

485 名前:132人目の素数さん:04/12/18 01:54:43
他の全てを指すと思ってたけど、今元ネタ読み直したら勘違いだった。
えぇとbは特定の元を指します。ちなみに元ネタは↓

In a game of patience, played on a finite set of counters on Z,
there are two legal moves:
(1)if there are two counters on the same integer k,
we may move one to k-1 and the other to k+1.
(2)if there is a counter on k and anothere on m>k
then we may move them to k+1 and m-1.
Starting with n counters, all on 0, what is the maximal distance between
two counters that can be achieved by a sequence of legal moves?

486 名前:132人目の素数さん:04/12/18 04:20:39
いくつかの整数を並べ、それに対し次のルールを考える。

(1) 2個の同じ数があったら、片方を+1、他方を-1してよい。
(2) 2個の異なる数があったら、小さい方を+1、大きい方を-1してよい。

n個の0から始め、このルールで操作を行なっていったとき、
この列中の2数の差は最大でいくつになりうるか?

487 名前:132人目の素数さん:04/12/18 04:36:40
えっと、今トライ中だけど
(2) ( a≠b → x_b=M and x_a=N<M ) となるaがあったら
  x_a=N+1と変化させ、a≠bとなるbに対しては x_b=M-1 と変化させる。

(2) x_b=M かつ x_a=N<M となるようなaおよびaとは別の(←当たり前)bがあったら
  x_a=N+1、x_b=M-1 と変化させる。
でいいんだよね?
誰かa game of patience, counters(計数器?)の訳し方教えれ

で、なんとなく[log 2]の気がするけど、証明が難しいな
それとも、実はそれより大きくて吃驚!ということなのかな?
数学は予想を間違うと、絶対証明できないから怖いな

488 名前:132人目の素数さん:04/12/18 04:40:29
あーごめん考えてたら十分も遅れて被っちまった
今はこの問題を考え中だけど、別の問題だしとくわ
ttp://science3.2ch.net/test/read.cgi/math/1103204972/31
(Fermat点のやつ)見てて思いついた奴。たぶん有名問題かもだった気もするが

三角形ABCの外部に正三角形BCD、CAE、ABFをつくる。このとき
AD、BE、CFは一点において交わることを示せ



489 名前:132人目の素数さん:04/12/18 11:21:38
counters=カウント厨

490 名前:132人目の素数さん:04/12/18 17:45:38
>>485の訳は>>486でいいのかな?
だとしたら、2*[n/2] になりそう。

[補題]
全て0からスタートした列は、全体の和を取ると常に0である。
逆に、和が0になるいかなる列も、ルールに従って操作を施す
ことにより全部0に戻すことができる。

[証明]
前半は2つのルールが和を変えないので明らか。
後半については、列の中の0でない要素を集めると、
最大元は必ず正、最小元は必ず負である。
それら2つに(2)を何回か施せば、少なくとも一方を0にできる。
その0を除外し、残った元に同じことを繰り返すと、0は
着実に増えていき、総和が0なので最後は全部0になる。

491 名前:490:04/12/18 17:55:20
さて、nが偶数である場合を考え、n=2m とおく。このとき、
列を -m,m,0,0,‥‥,0,0 とすることができる。(*)

[証明]
m=1 のときは明らか。m=kのときに(*)が正しいと仮定する。
m=k+1 のとき。2k+2個の0からスタートして、最初から
2個の0を無視すれば、仮定により -k,k,0,0,‥‥,0,0 (0がk個)
という状態に持っていける。後ろのk個の0にさらに仮定を適用
すれば、-k,k,-k,k,0,0,‥‥,0 とできる。この(-k,-k)と
(k,k)に(1)を施して、(-k-1,-k+1),(k+1,k-1)、これと
補題より -k-1,k+1,0,0,‥‥,0 とできる。
つまり m=k+1 のときも(*)が成り立つ。

∴nが偶数の時は、差の最大は少なくともnにはなる。
nが奇数の時は、最初から0を1つ無視すれば偶数の場合に
帰着でき、少なくとも(n-1)/2にはなることがわかる。

‥んで、どうやらこれより大きくはなれないみたいなんだけど、
なんかうまくいかない。n=2mとして、正の最大をm+1にする
ためには、その直前のステップでmが2個存在する必要があって、
長さ2mの列で2個のmを生成するのが不可能であることを
言えばいいんだけど。誰かに任せた。

492 名前:132人目の素数さん:04/12/18 20:20:27
↑の結論を出すには、
 AP ≧ {sin(C)/sin(A)}PE + {sin(B)/sin(A)}PF
が必要では....。 

不等式スレッド
http://science3.2ch.net/test/read.cgi/math/1072510082/814

493 名前:132人目の素数さん:04/12/19 16:18:29
この問題にsin関係あるか?

494 名前:132人目の素数さん:04/12/19 16:23:57
【問題】
1円、5円、10円、50円、100円、500円、1000円、5000円、10000円の各通貨を使い
買い物をする。(2000円札は入れない)
自分の所持金額に対し、常に最小枚数の状態であることが要請されているとして、
請求金額に対する払い方が一意になることを証明せよ。


495 名前:132人目の素数さん:04/12/19 16:43:38
http://www1.kiy.jp/~yoka/integ-ranking/top.cgi

定積分を楽しく学べるサイト!!きみはどこまで上り詰められるかな?

496 名前:132人目の素数さん:04/12/19 17:09:03
マルチ!

497 名前:132人目の素数さん:04/12/19 18:23:19
11円持ってるときに6円のものを買う場合
10円玉だけでなく、1円玉も出す払い方でないといけないわけだね

498 名前:494:04/12/21 01:23:33
>>497
いえす。
ちなみに、お店側が払うお釣りも最小枚数状態というルール、
自分の財布の中身は最小枚数状態を維持し続けなければならない…って、これは言ったか。
あと、余計な払い方(300円の請求に対して2400円払うとか)、こういうのもなし、ということで。


499 名前:132人目の素数さん:04/12/21 01:34:30
>>498自己レス。
> あと、余計な払い方(300円の請求に対して2400円払うとか)、こういうのもなし、ということで。

わかりづらいな。
支払金種とお釣りの金種に重複があってはならない、と言いかえたほうがいいのだろうか。
まあ、「一意」ということなので、これを認めるとなんでもありになってしまう、と。


500 名前:132人目の素数さん:04/12/21 01:42:43
要するに任意の金額Xに対して合計してX円になるような貨幣の組み合わせのうち
貨幣の枚数が最小値をとる組み合わせは一通りに限る。ということですね。

1円玉と8円玉と15円玉があるとすると、

16 = 1 + 15 = 8 + 8

とかなって不味いわけだ。

501 名前:132人目の素数さん:04/12/21 09:31:34
なんか付帯条件がやたらとうざったいようなw

まあ貨幣・硬貨が 2a_n<a_{n+1} を満たしているから明らか、ではだめなの?

502 名前:132人目の素数さん:04/12/21 12:15:53
一列に並んだ、N個の異なるデータがある。
これを、1つのスタックを用いて並べ替えることを考える。

・入力は、先頭から順にスタックに積んでいく。
・取り出しは、任意のタイミングで任意個行なうことができる。
・取り出された順にデータを並べて出力とする。
・どのデータも、一度だけスタックに積まれる。
・最終的に、スタックは空にする。

このとき、出力の並び方は何通りか。

スタックを知らない人は↓
http://www2.cc.niigata-u.ac.jp/~takeuchi/tbasic/BackGround/Stack.html

503 名前:502:04/12/21 12:20:01
俺も考察中なので答えはわかりません。

504 名前:132人目の素数さん:04/12/21 13:30:00
(2n)!/n!(n+1)!。


505 名前:132人目の素数さん:04/12/21 16:24:55
>>500
>要するに任意の金額Xに対して合計してX円になるような貨幣の組み合わせの
>うち貨幣の枚数が最小値をとる組み合わせは一通りに限る。
これは出題の意図と一致してるんだろうか。
例えば、お店には50円玉がないかもしれない。でも手元に10円と100円が
あるなら、題意を満たしつつ60円払う「払い方」は10円+100円の一通りだよ、
という問題だと思ってた。

506 名前:506人目の素数さん:04/12/21 17:16:49
>>500
解答を大雑把にいうと
1円玉より大きい金額の貨幣や硬貨の金額はそれより小さい金額の
貨幣や硬貨全て1種類だけで構成することができる。
(2000円では5000円は構成できない。そのため問題から除かれたのだろう。)

そのため、任意の金額X以下の金額の貨幣・硬貨でXを割り、その余りを
次に大きな金額の貨幣・硬貨で割り、余りが0になるまで繰り返したとき
その商の和は最小になる。
(もし、ならないとすると、
ある金額の貨幣・硬貨Yの個数1つ<Yより小さい金額で同じ金額構成したときの個数
 に矛盾する。)

この計算は1通りの答しかないため、
貨幣の枚数が最小値をとる組み合わせは一通りに限る。

507 名前:506人目の素数さん:04/12/21 17:19:38
訂正
>>506>>494 ですた。

508 名前:506人目の素数さん:04/12/21 17:21:18
訂正の訂正
>>506>>494 ですた。」
の506→500ですた。


509 名前:132人目の素数さん:04/12/21 17:33:22
直径nの円の中に直径1の円は重ならずに何個いれられるか。

これ一般に数列化できるのかね・・

510 名前:132人目の素数さん:04/12/21 17:39:23
なんだよ数列化って

511 名前:132人目の素数さん:04/12/21 17:41:46
>510
computerで計算できるような関数になるかな〜とか思って・・
別に一般的な言葉使いではないです。

512 名前:132人目の素数さん:04/12/21 17:51:25
computerで計算できる自然数から自然数への函数のことは
recursive function(帰納的函数)といいます。
一般的な言葉遣いです。

513 名前:506人目の素数さん:04/12/21 18:04:23
>>502さんへ
 N個のデータの1つ1つを1緒にするか否か(連結の仕方)で
データの並び方は区別できます。
(異なった連結の仕方でデータの並び方が異なることは自明)
N個のデータの連結部分はN-1考えられ、1緒にするか否かなので

      2^(N-1)通り
が答です。
>>504さんの解答は残念ながら間違いです。
例えばN=3では4通りですが504さんの解答では
5になってしまいます。

514 名前:132人目の素数さん:04/12/21 20:10:00
123。
132。
213。
231。
321。


515 名前:132人目の素数さん:04/12/21 20:27:25
>>513
>>514嫁。てめーが間違ってんだよ!脳味噌、膿んでんじゃねーのか?
偉大な「mathnori」トップクラス常連者の○○様が、お間違えになるはずねーだろが!
死ね!ボケ!二度とレスつけんな!w

516 名前:132人目の素数さん:04/12/21 20:28:26
アホ

517 名前:494:04/12/21 22:41:35
>>500 >>501 >>506
すみませんが不正解です。
問題の趣旨をご理解いただいてないようで残念です。

>>494 の趣旨は
「貨幣の枚数が最小値をとる組み合わせは一通り」
ではなく
「貨幣の枚数が最小値をとるための、請求金額に対する払い方が一通り」
です。

>>497 さんと >>505 さんがご理解いただいており、光栄でございます。


518 名前:132人目の素数さん:04/12/22 00:51:40
>>517
最小枚数となる組が一意なんだから,支払い前の財布の中身と支払い後
の財布の中身を見比べて支払い前のみに存在する貨幣を払えばいいん
でしょ.

519 名前:132人目の素数さん:04/12/22 09:16:01
>>518

現在の所持金が72円(50円1枚、10円2枚、1円2枚)だとする。
ここに6円の請求がきた場合、
支払後の残高は66円(50円1枚、10円1枚、5円1枚、1円1枚)となるわけだが、

(50円1枚、10円2枚、1円2枚)
 と
(50円1枚、10円1枚、5円1枚、1円1枚)

の差分を見て、
> 支払い前のみに存在する貨幣を払えばいい

つまりここでそれに該当するのが
(10円1枚、1円1枚)
となり、これを支払えばいい…

という主張で良いんだよね?


さて、これが任意の所持金と任意の請求金額に対して
一般的に成り立つかどうかなんだけど…。


520 名前:502:04/12/22 11:21:49
うお、すげえ。1時間で答え出てるし。
てめぇで問題出しときながら解けないとは情けなや。

最初、出力の先頭に来るデータがどれかによって
場合分けして漸化式を立てようと考えたんだけど
どうもうまくいかない。

風呂に入って熟考中、、pushとpopの発動順序に着目
することを思い立つ。つまりpushを I、popを O としたとき、
N個のデータではIとOがそれぞれN回ずつ発動し、
これらの並び方が、出力と一対一に対応する。
そこで次を考えればよい。

・N個のI、N個のOを一列に並べる。このとき、
どの部分で列を左右に分割しても、
「左側に含まれるIの数≧左側に含まれるOの数」
となるようなものは何通りか。

ところがこう変形しても、うまく式になってくれなくて
困っている。>>504が正しそうなことは何となく
わかるんだけどね‥‥

521 名前:132人目の素数さん:04/12/22 11:30:26
>>494
その条件だけだと一意的には定まらんぞ?
519の例でいくと、10円一個と1円二個払ってもいいことになる

522 名前:132人目の素数さん:04/12/22 11:34:47
>520
カタラン数 でぐぐれ。

523 名前:502:04/12/22 11:46:15
さらに、I、Oをそれぞれ左カッコ、右カッコと読み替えれば、
問題は次のようにも変形できる。

・N組のカッコを、文法的に正しくつける並べ方は何通りか。

N=3 のとき、この解釈を>>514の順番で並べれば、
( ) ( ) ( )
( ) (( ))
(( )) ( )
(( ) ( ))
((( )))
となる。つまり、結合則を満たさない2項演算@があって、
a@b@c@d みたいに並べられてたとき、どの順序で
演算を実行していくか?ということを考えればいい
ような気がする、といったところで現在停滞中。

524 名前:502:04/12/22 11:50:33
>>522
これだ。
その言葉は聞いたことしかなく、
内容は全く知らなかったんだけど、
まさにこれみたいです。ありがとう。

525 名前:132人目の素数さん:04/12/22 13:02:15
>>521

>>498-499


526 名前:132人目の素数さん:04/12/22 13:19:04
百円硬貨3枚が机の上に、左から順に表表表の状態で一列に並んでいます。
さいころを振り、出た目が1,2であれば左端の百円硬貨を裏返し、3,4であれば
まん中の百円硬貨を裏返し、5,6であれば、右端の百円硬貨を裏返します。
このような操作を21回繰り返した結果、表表表または表裏表となる確率は、
何分の何ですか。

527 名前:132人目の素数さん:04/12/22 13:19:49
1つのサイコロを10回投げて、投げた順に出た目の数の積を作っていきます。
このとき、目の数の積が第10回目までのどこかで4となる確率は、
1259712分の何ですか。

528 名前:132人目の素数さん:04/12/22 15:33:18
(6^11 - 56)/(25*6^10)

529 名前:132人目の素数さん:04/12/22 15:38:15
>>526-527
糞マルチ氏ね


530 名前:132人目の素数さん:04/12/23 23:02:39
あげ

531 名前:132人目の素数さん:04/12/24 04:52:45
真似したり、関係の無い事言ったり、適当な事書いたり、無茶苦茶書くな 

荒らしは
 〜〜〜終了〜〜〜
 
ageるな馬鹿タレ

お前が数学出来ないのはわかるが八つ当たりするな

532 名前:132人目の素数さん:04/12/27 07:00:50
ほしゅ


533 名前:132人目の素数さん:04/12/28 13:09:04
>>467
軽いほうの A 枚を判定せよという問題なら、
重いほうの B 枚が重さ一定にしろ相異なるにしろ、
回数に変わりは無いのではないか?


534 名前:132人目の素数さん:04/12/28 13:26:21
http://anti.bne.jp/upload/updata/up0611.mp3
こいつがおれに電話かけてくる確率を計算せよ

535 名前:132人目の素数さん:04/12/28 13:35:53
>>534
恐くて踏めんのだが…

536 名前:132人目の素数さん:04/12/28 13:59:05
>>534
今聞いたよ。
かけているのは雑誌社か?警察か?

537 名前:132人目の素数さん:04/12/28 14:20:00
>>533
A=1,B=2。
1,2,2は一回。
1,2,3は二回。


538 名前:132人目の素数さん:04/12/28 14:34:54
>>537
そうか、なるほど。

539 名前:132人目の素数さん:04/12/29 00:22:58
結局>>494の証明はだめでしたか。


540 名前:132人目の素数さん:04/12/29 03:30:47
>>467
「同じ重さの軽い玉A個と
同じ重さの重い玉B個
合計(A+B)個があるとき」
の最小回数をN(A、B)

「同じ重さ(重さa)の玉A個と
同じ重さ(重さb)の玉B個  (a≠b)
合計(A+B)個があるとき」
の最小回数をN’(A、B)とするとき

儂(A、B)=N’(A、B) - N(A、B)
はどうなるかな。


541 名前:132人目の素数さん:04/12/29 16:56:25
age

542 名前:132人目の素数さん:05/01/06 01:13:27
ほしゅ


543 名前:132人目の素数さん:05/01/06 01:30:18
別スレで未解決の問題

ttp://science3.2ch.net/test/read.cgi/math/1104474105/159
159 名前:132人目の素数さん[] 投稿日:05/01/03 13:29:52
1+3+3^2+3^2+3^4+・・・・+3^nが平方数になるようなnを求めよ

お願いいたします。

544 名前:132人目の素数さん:05/01/06 01:46:44
>>543
規則性がわからん


545 名前:132人目の素数さん:05/01/06 01:50:50
Σ[k=0,n] 3^k = 1 + 3 + 3^2 + 3^3 + 3^4 +・・・・+ 3^n
が平方数になるようなnを求めよ

の間違いだねw

546 名前:犬笠銀次郎:05/01/06 13:09:15
次の問題を微分を用いずに解いてみよ。

半径1の球に内接する四面体のうち、体積が最大になるものは正四面体であるこ
とを示せ。

547 名前:132人目の素数さん:05/01/06 14:59:39
>>546
略解.美しいから。

548 名前:132人目の素数さん:05/01/07 10:15:38
2005^2005の各桁の和をA、Aの各桁の和をB、Bの各桁の和をCとする。
Cを求めよ。

549 名前:132人目の素数さん:05/01/07 19:21:14
C=7

550 名前:伊丹公理 ◆EniJeTU7ko :05/01/08 00:17:09
>>546
体積最大のものの存在は自明的にいえるから、
後は容易

551 名前:132人目の素数さん:05/01/10 12:56:16
どして自明的なの?

552 名前:はなう ◆dZlodismQg :05/01/10 20:18:16
(=゚ω゚)ノ
ひさびさに数学板来てみたら。。。まだあるのかこのスレ。
ってかわからんスレはまだ2つ平行であるのか。。。


高校の頃ちょっと見て、面白かったから2番を自作した。非数学科学部生レベル。美しいので是非解いてみてください。


1.数列Anを以下とする

A1=√2
A2=(√2)^(√2)
A3=(√2)^A2
A4=(√2)^A3

An=(√2)^(An-1)

このとき数列Anが収束することを示し、
lim(n→∞)Anを求めよ


2.pを任意の実数(>0)とする時、数列Bnを以下とする

B1=p
B2=p^p
B3=p^B2
B4=p^B3

Bn=p^(Bn-1)

このとき、Bnが収束するためのpの条件を求めよ。

553 名前:伊丹公理 ◆EniJeTU7ko :05/01/10 20:38:02
>>551
★東大入試作問者になったつもりのスレ★ 第4問
http://science3.2ch.net/test/read.cgi/math/1099493043/
にも類題があったが、球面上の 4 点を並べた点の空間
{ (x, y, z, w) | x, y, z, w はその球面上の点 }
はコンパクト。
その凸包のの体積はその空間上の連続関数だから最大値を取る。

554 名前:132人目の素数さん:05/01/11 00:43:03
そりゃあ大学でεδ式の微積を
まじめに勉強した人間には明らかでしょうやwww

高校生にとってそうかな?普通の高校生や東大受験生が
最大値の存在は証明すべき事であるとか
そういう意識を持つこと自体がアブノーマルじゃないかな?

555 名前:132人目の素数さん:05/01/11 18:53:15
N面サイコロ振って
全部の面が出る確率が50%を越える回数とか。

M回振った場合、全部の面が出てる確率を求める数式。
高校レベルよりは難しい大学レベルの初歩かと思うんだけど。
ずっと前数学パズルのページで、パズル好きからまともな回答がつかなかった問題。

556 名前:132人目の素数さん:05/01/11 19:00:24
>>554
ノーマルじゃないからどうした?
ノーマルがそんなに重要か?

557 名前:132人目の素数さん:05/01/11 20:08:22
>>555
前にどっかで書いたやつ

1〜n を取る m 個の乱数を振ったとき k 種類の数が出る確率を P(m,k) とすると
P(0,0)=1, P(0,k)=0 (k>0)
P(m+1,k) = (k/n)*P(m,k) + {1-((k-1)/n)}*P(m,k-1)。
よって、x の多項式 f[m](x) = Σ[k=0,n]{P(m,k)x^k} について
演算子 A を A = (1/n){x(d/dx) + x(n-x(d/dx))} としたとき
f[m+1](x) = A・f[m](x) なので f[m](x) = A^m・f[0](x) = A^m・1。
g[k](x) = x^k*(1-x)^(n-k) とすると A・g[k](x) = (k/n)g[k](x)。
一方、C[n,k] を二項係数として
Σ[k=0,n]{C[n,k]g[k](x)} = {x+(1-x)}^n = 1。
よって
f[m](x) = A^m・1 = A^m・Σ[k=0,n]{C[n,k]g[k](x)}
= Σ[k=0,n]{C[n,k](k/n)^m*g[k](x)}。
x^n の係数を比較して
P(m,n) = Σ[k=0,n]{C[n,k](-1)^(n-k)*(k/n)^m}。

558 名前:伊丹公理 ◆EniJeTU7ko :05/01/11 21:19:02
>>552
http://www.akanekodou.sytes.net/math/pdf/towerp.pdf
参照

559 名前:132人目の素数さん:05/01/11 22:02:26
2とは何ですか。
回答例:
最初にゼロがある。そこに1が現れる。他には何も無い。
1が占有した領域と同じだけの領域を1以外に確保したとすれば
そこには1とまた別の1がある。このどちらかを2と呼んでも
構わない。どうよ?


560 名前:132人目の素数さん:05/01/11 22:07:05
占有とか領域とかあまり本質的じゃないと思われ

561 名前:132人目の素数さん:05/01/11 22:11:01
>>560
その理由は?

562 名前:132人目の素数さん:05/01/11 22:21:59
>>561
占有,領域,確保が定義されていないから

563 名前:132人目の素数さん:05/01/11 22:25:39
>>562
じゃあ、まず君の考えで「領域」を定義してみてくれ。話はそれからだ。

564 名前:560:05/01/12 00:37:08
2とは何か?
⇒定義: 2 =_{def} 0 + 1 + 1(つまり、 S(S(0)) あるいは 0'' )じゃないのか?

で、何のためにそのような概念を考えるのか?
⇒ものの個数や順序を数えるため。

個数とは何か、順序とは何か?
⇒哲学板へGO!

2 とは何かを本当に知らない人に>>559みたいな
答えかたしても絶対分からないだろ。

565 名前:132人目の素数さん:05/01/12 05:38:37
こんなコピペを発見した。
丸の中には、0から9までの数字が一つづつ入るってことかな?
でないと問題にならんわな。

>544 名前:132人目の素数さん[] 投稿日:05/01/05(水) 05:37:54
>これ答えれる方!
>〇〇〇/〇〇〇+〇〇/〇〇=1
>わかる方いますか?
>後半のフタ桁の分母は36〜56以内です
>お願いします

566 名前:132人目の素数さん:05/01/12 20:35:28
こうじゃないの?

定義1:「自然数」と名づけた集合を考える。

定義2:「1」は自然数とする。

定義3:自然数の任意の元に対して「1を加える」という操作を定義する。
  (便宜上、記号で+1と書くことにする)

定義4:任意の自然数に+1の操作を加えて生成されたものも自然数とみなす。

で、1+1の生成結果をとりあえず「2」と呼ぶことにしましょ

ってことでしょ?


567 名前:132人目の素数さん:05/01/12 21:27:54
>>566
>定義1:「自然数」と名づけた集合を考える。
定義1:「自然数」と名づけられた物の集合を考える。

568 名前:132人目の素数さん:05/01/13 00:31:06
>>565
○の中に 0〜9 が一回ずつ入り、
最上位桁の数字は 0 にならないとして探索してみた

287/369 + 10/45 = 1
728/936 + 10/45 = 1
169/507 + 32/48 = 1
269/807 + 34/51 = 1
204/867 + 39/51 = 1
678/904 + 13/52 = 1
609/783 + 12/54 = 1
309/618 + 27/54 = 1

569 名前:132人目の素数さん:05/01/13 01:09:36
>>568
下品な人

570 名前:132人目の素数さん:05/01/13 02:12:42
>>568
いやいや、乙です。
やっぱ、解が一つじゃないと
パズル問題としてはおもしろくないよね

571 名前:132人目の素数さん:05/01/13 11:01:30
>>570
二つの分数は既約分数とする、という条件を加えると4番目だけが正解になる

・・・と思ったけどならない

572 名前:132人目の素数さん:05/01/13 13:25:47
>>571
1 - 32/48 = 16/48

573 名前:132人目の素数さん:05/01/13 19:26:27

そもそも分母が違うんだから今回の場合
少なくともどちらか一方は既約ではありえない罠



574 名前:犬笠銀次郎:05/01/14 18:09:32
そろそろ >>546 の答え書こうか、、、。

正四面体でないならば、体積が最大にならないことを示せば良い。

四面体 ABCD が正四面体でないならば、この四面体の内、少なくとも等しく
ない二辺が存在する。それが、AB と AC だったとしよう。今度は、三点
A, B, C を通る平面の上で考える。このとき、弧 BAC 上に、BE = CE とな
るように点 E を取ると、三角形 BCE の面積は三角形 BCA のそれよりも大き
い。よって、四面体 BCED の体積は四面体 BCAD のそれよりも大きい。(終)

対偶と初等幾何を知ってれば解ける言うわけやな。


575 名前:132人目の素数さん:05/01/14 19:03:33
それはダメやっちゅうに。

自然数の中では1が最大である。
なぜなら1以外のnに対してはn<n^2で最大で無いから

と同じ論法。


576 名前:132人目の素数さん:05/01/14 20:21:49
臭せぇ〜、臭せぇ〜よ! 銀次!

577 名前:132人目の素数さん:05/01/14 21:46:36
>>574
弧BACがわからないんだが


578 名前:伊丹公理 ◆EniJeTU7ko :05/01/15 02:07:18
空間内の四面体において、買ツ はそれの6つ辺のの二面角の和を、買ヨ はそれの4つ頂点のの立体角の和を表すとする。
(ただし一頂点における立体角の全角は 4π とする。)この時、2買ツ −買ヨ = 4π となる事を示せ。

579 名前:132人目の素数さん:05/01/15 12:11:47
>>575
意味不明。

>>574 に関しては、正四面体でない、四面体に対して、>>574 の「平面」上で
三点を二等辺三角形になるように(=四面体の体積がより大きくなるように)
点を動かす操作を点A以外、点B以外、点C以外、点D以外についてこの順で繰り
返せば、正四面体に近づくことを言えばなお良い。

580 名前:132人目の素数さん:05/01/15 13:05:46
>>579
「体積最大の図形が存在する」(1)
が言えてないので、
「正四面体でないならば、体積最大でない」(2)から「正四面体が体積最大である」(3)は導けない。

>>575も同じ。
「最大の自然数が存在する」(4)
が言えてないから、
「1でないならば、最大の自然数となり得ない」(5)から「1が最大の自然数である」(6)は導けない。


ちなみに事実は (1)真(2)真(3)真 (4)偽(5)真(6)偽 であるが、それは今は関係ない。

581 名前:挑発痴女:05/01/15 16:23:04
グッジョブ

582 名前:132人目の素数さん:05/01/15 16:47:11
>>579
どの四面体からスターとさえても
>三点を二等辺三角形になるように(=四面体の体積がより大きくなるように)
>点を動かす操作
この操作を繰り返すことで四面体に近づくことを言えば、確かに間違いがないといえる。
けど、>>574にはこれが全く触れられていないので、関係なし。

583 名前:挑発痴女:05/01/15 16:56:21
「体積の最大値が存在し、最大をとるとき、それは正四面体である」が正しい命題だな。
受験数学じゃ「最大値を求めろ」だけだけど、どういうときに最大値をとるか書かないと減点される。
対偶を取るならば、「正四面体で最大値をとらなければ、最大値をとるような四面体は存在しない」だろう。

584 名前:挑発痴女:05/01/15 17:14:46
あ、ちょっと対偶変だけど、まぁ影響ないか。

結局、微分使わずにどうやって最大値の存在を言えばいいかは難しいかな?

585 名前:挑発痴女:05/01/15 17:23:05
ごめん、あまり意味はないことだけど、
最大値の存在は証明できることなのに、
「最大値が存在するならば〜〜」なんて言い方はちょっと変だな。正しくは、
「内接する四面体のうち、最大値をとる四面体が存在して、それは正四面体である」だ。
対偶取れないな。

586 名前:132人目の素数さん:05/01/15 18:11:04
で?

587 名前:132人目の素数さん:05/01/16 21:30:54
微分では最大最小が出るけど、それ使わないなら平方完成的な
方法か。任意の一点からそれを含まない球内の平面に三角形を投影した場合
の最大面積は正三角形か。まずはそれを証明してみたらどうだろう。

588 名前:132人目の素数さん:05/01/18 07:04:01
n を自然数とするとき、a! + b! + c! = 2^n をみたす自然数解 (a, b, c) をすべて求めよ。

589 名前:132人目の素数さん:05/01/20 11:08:39
7以上の素数pについて、次の命題は真か?
証明するか反例を挙げよ。

p^2≡1(mod 5) ⇔ あるaが存在し、a^2≡5(mod p)

590 名前:132人目の素数さん:05/01/20 13:04:03
>>579 でいいんじゃねーの? 結局与四面体は半径1の球に内接してるから、
四面体の体積は上限を持つ。>>579 提唱の方法の操作をくり返す度に、四面
体の体積を v_1 , v_2 ,... としけていけば、{v_n} は単調増加な数列。上
に有界な単調増加数列は収束し、その収束するときの状態は正四面体。(こ
の事実は証明可能だが、マンドクサいのでやらねー。)

591 名前:132人目の素数さん:05/01/20 14:42:55
>上に有界な単調増加数列は収束し、
ぼみょー。何の証明をし

592 名前:132人目の素数さん:05/01/20 16:11:25
>>588
これはどうするの?

593 名前:132人目の素数さん:05/01/20 16:50:00
a!≦b!≦c!とするとa!は2^nの約数。


594 名前:132人目の素数さん:05/01/20 16:58:26
>>590
>>590
>>590
>>590
>>590
>>590
>>590
>>590
>>590


595 名前:132人目の素数さん:05/01/20 18:21:49
>>588

(a,b,c)=(1,1,2),(1,1,3)(2,3,5)

これであってますか?

596 名前:132人目の素数さん:05/01/20 18:23:58
↑a≦b≦cとしてあります


597 名前:132人目の素数さん:05/01/20 19:53:39
>>595
(2,3,4)はどうなった

598 名前:132人目の素数さん:05/01/20 20:43:16
>>589
真。
略証。(5/p)(p/5) = 1.

599 名前:132人目の素数さん:05/01/21 19:52:59
紙に書いた円にむかって上から棒を落とします。
棒は必ず円と、2点で交わるか、接するものとします。
交わった場合の2点をそれぞれA,Bとします。
この時、AB間の距離と接する場合の点が、
この円に内接する正三角形の一つの辺の長さより長くなる確率は
いくつでしょう?

600 名前:132人目の素数さん:05/01/21 20:15:29
決定不能

601 名前:132人目の素数さん:05/01/21 20:23:14
お前は不能 (インポ)か?

602 名前:494:05/01/21 20:31:17
>>599
題意を満たす確率の定めかた次第で変わってくる。

@ 棒が落ちた状態の図を、解りやすいように
  点Aが真下に来るように回転させて、
  点Bの取りうる位置という考え方で逝った場合。
  点Aを頂点の一つとする内接正三角形を思い浮かべて下さい。
  「正三角形の一つの辺の長さより長くなる」点Bの取りかたは
  内接正三角形の上部にある場合がそれに当たる。
  円周上、その範囲は120度であり、全体に対する割合は1/3となる。

A 棒が落ちた状態の図を、解りやすいように
  棒が水平になるように回転させて考え、
  弦ABの中点の取りうる範囲、という考え方で逝った場合。
  取りうる弦ABの中点の軌跡は円の直径となるわけだが、
  上部1/4と下部1/4の部分に弦ABの中点が来ると
  題意を満たさないので、それ以外の中央の部分。
  よって題意を満たす弦ABの中点の取りうる範囲は1/2。

でいかがでしょう?

ところで、私が>>494で出した問題は未解決のままでつねorz


603 名前:599:05/01/21 20:42:31
>>602
授業ではそうならったんですが、
実際に、試行回数をふやしていくと、
1/2か1/3かどちらかに収束しないんでしょうか?

604 名前:132人目の素数さん:05/01/21 20:51:05
>>603
そもそもが、すべての弦ABの取り方というのが
「同様に確からしい」のでしょうか?既にその段階で疑問があります。
棒の長さも、落とす高さも決まっていない。

> 棒は必ず円と、2点で交わるか、接するものとします。

そうならなかった場合を排除してるけど、これは問題ないのでしょうか?


605 名前:494:05/01/21 20:52:39
おっと>>604名前書くの忘れた


606 名前:132人目の素数さん:05/01/23 01:38:16
線形代数
(1) Aが正則であることとAの余因子行列が正則であることが
   同値であることを証明せよ。
(2) Aが実対称行列ならばAの余因子行列も対称行列であつことを
   証明せよ。」
          是非解いてみてください。

607 名前:132人目の素数さん:05/01/23 01:45:31
>>606
ここは宿題を貼るスレではない

608 名前:132人目の素数さん:05/01/23 05:19:16
606誰かといてください。お願いします。

609 名前:132人目の素数さん:05/01/23 05:31:31


610 名前:132人目の素数さん:05/01/23 05:52:31


611 名前:132人目の素数さん:05/01/23 05:57:21


612 名前:132人目の素数さん:05/01/23 06:08:06
>>606
> 是非解いてみてください。

それが質問者の態度か?
( ゚∀゚)プケラッチョ!

613 名前:132人目の素数さん:05/01/23 06:35:54
テラワロスwwwwwwwwwwwwwwwwwwwwwwwwwwwwwwwwwwwwwwwwwwwww

614 名前:132人目の素数さん:05/01/23 10:22:01
>>606 = >>608
大学1年にもなって、なんでこうも厨房丸出しなんだろ、ほんと情けない。
とりあえず氏んどけよ。


615 名前:132人目の素数さん:05/01/23 10:27:47
大学4年かも奈

616 名前:すぱいらす ◆spirusLFRk :05/01/23 10:41:00
>>615
大学4年にもなって、線形代数でつまづいてたら終わってるよ。


617 名前:132人目の素数さん:05/01/23 14:31:28
てめ=ら志ね!!きもいぞこら!!

618 名前:132人目の素数さん:05/01/23 14:44:02
>>617
http://science3.2ch.net/test/read.cgi/math/1105688404/
http://science3.2ch.net/test/read.cgi/math/1106442829/


619 名前:132人目の素数さん:05/01/23 14:48:33
>>617=>>606=>>608
マルチ厨=ポリゴンヲタ 資ね


620 名前:冬厨火葬:05/01/23 14:52:30
ポリゴンがまた暴れているのか?

621 名前:132人目の素数さん:05/01/23 14:54:13
ttp://www.sexstump.com/streetscene.jpg
(´Д`)ハアハア

622 名前:132人目の素数さん:05/01/23 15:07:42
>>621
精神的部落ら


623 名前:132人目の素数さん:05/01/23 20:32:26
>>621
(ノд`) アチャー
今夜は魘されそうだ…

624 名前:132人目の素数さん:05/01/23 21:33:45
>>620
そのようです。
マルチで荒らして、みんなに文句言われたら逆ギレしてブラクラ貼る始末。
性根まで腐ったクソガキのようです。


625 名前:132人目の素数さん:05/01/23 23:36:52
>>624
自分ではったのを他人のせいにするな

626 名前:132人目の素数さん:05/01/24 12:35:19
625=606?

627 名前:132人目の素数さん:05/01/24 12:59:16
>>626
自分ではったのを他人のせいにするな


628 名前:132人目の素数さん:05/01/27 16:29:02
627=606?

629 名前:132人目の素数さん:05/01/27 17:29:02
621=624

630 名前:132人目の素数さん:05/01/27 17:44:16
609=610=611

631 名前:624:05/01/28 00:51:44
>>629
それは違う


632 名前:ゾノネム:05/01/28 13:15:16
a, b, c, d を整数とし、特に、a, c を互いに素な正整数とする。

このとき、ad - bc = 1 となる整数の組 (b, d) が存在することを証明せよ。

yo, yo, yo, yo, nyo, nyo, nyo, nyo,............

633 名前:132人目の素数さん:05/01/28 14:37:04
>>632
そんな基本問題を....

634 名前:132人目の素数さん:05/01/28 15:01:06
>>632
すんません、簡単な質問で恐縮なんですが、
cos(-30°)は、-(√3/2)になるのでしょうか。


635 名前:132人目の素数さん:05/01/28 15:18:54
>>634
naranai

636 名前:132人目の素数さん:05/01/28 18:48:25
>>632
ユークリッドの互除法でぐぐれ

637 名前:エージェントスミス:05/01/29 00:40:51
定義域が実数全体の独立変数xに従属して変化する関数f(x)があるとします。
このとき任意の定数aに対しx=aときf(x)キ0,xキaときf(x)=0となるように
関数f(x)を設計してください。
という問題なのですがどなたか頭のいい方できますでしょうか?

638 名前:132人目の素数さん:05/01/29 02:52:58
>>637
2行目のxの束縛が不明。
∀a∃x〜 なら f(x)=1 とすればよく、
∀a∀x〜 なら構成不能。

639 名前:132人目の素数さん:05/01/29 04:00:47
>>634
cos(-x)=cosx
sin(-x)=-sinx

640 名前:132人目の素数さん:05/01/29 06:10:53
xキaときってなんだよ。。。

x ≠ a のとき、だろ。答案の書き方から勉強しなおせ

641 名前:132人目の素数さん:05/01/29 08:28:10
キって初めて見たな(笑
信者=儲みたいで目から鱗だ。

642 名前:132人目の素数さん:05/01/29 10:40:35
>>637は左利きなのだと好意的に解釈してみる

643 名前:132人目の素数さん:05/01/30 21:57:17
男は金で仮想恋愛を得る
女は仮想恋愛で金を得る

lim男(金→0)
lim女(仮想恋愛→0)
を求めよ。
真実の愛へ収束するのか、恋愛は成立しないのか。

644 名前:132人目の素数さん:05/01/31 03:53:43
俺、右利きやけど ≠じゃなくてキを書いてしまう。
理由は文章・数式は左から右へ書くので、キのほうが次の字が書きやすい。

645 名前:132人目の素数さん:05/01/31 04:03:13
別に紙に書くときはいいんだけど、
掲示板に数式書くときに「き」を変換してまで
キを使うのは狂ってますよ

646 名前:132人目の素数さん:05/01/31 15:28:55
くるってる、までは・・・
自分は、「きごう」で変換して膨大な記号の中から探すんだけど。
そんな頻繁に使う文字じゃないからすぐ埋もれて行く。
面白いから今度からはキでいこうかと思った。楽だし。


647 名前:132人目の素数さん:05/01/31 19:54:15
=を変換すれば出てくる。
俺はね。

648 名前:132人目の素数さん:05/01/31 19:59:09
「のっといこーる」で出てくるな、≠。wnnの標準の辞書だと思う。

649 名前:132人目の素数さん:05/01/31 20:57:48
>>647
おー、ちょっと感動した。

650 名前:132人目の素数さん:05/01/31 22:36:32
敢えてキを用いたハイセンスな637に萌えた

651 名前:132人目の素数さん:05/02/01 02:42:52
IME2000の場合
"のっといこーる"だとでない
"ふとうごう"だとでる

652 名前:132人目の素数さん:05/02/01 03:56:32
>>645
>狂ってますよ
キじるしというわけか

653 名前:BlackLightOfStar ◆ifsBJ/KedU :05/02/01 14:00:17
&#8800;


654 名前:132人目の素数さん:05/02/01 17:30:45
(問)x+y+z=白は、条件を変えるとx+y+z=黒になる。
x,y,zに入る色と条件の違いについて述べなさい。



655 名前:132人目の素数さん:05/02/01 18:02:42
>>654
赤、緑、青
条件の違いは光の強度

656 名前:132人目の素数さん:05/02/02 02:32:13

>>655
ソラリゼーションすか

>>654
加法混色と減法混色
「+の定義」や「書いてある関数の形・意味」を変えることを
条件を変えるって言うのには、なんか抵抗があるなあ。

657 名前:132人目の素数さん:05/02/02 03:22:26
RGB/CMYKってことね
数学の素養のなさそうな問題だなあ

658 名前:132人目の素数さん:05/02/04 07:31:42
で、x,y,zに入る色って何よ?

659 名前:132人目の素数さん:05/02/04 19:29:38
文学的な表現に酔うのが好きで
ちょっとかじっただけの理系的知識をヒケラカス痛い香具師って多いよね
「不機嫌なジーン」の脚本家とかもその類だろうな

>>654 = >>658


660 名前:132人目の素数さん:05/02/04 20:16:39
平面上の各点に、白または黒の色をつける。
白点と黒点の集合をそれぞれW、Bとする。

このとき、どのような塗り方に対しても、
WかBの少なくとも一方は、任意の実数dに対し
距離dだけ離れた2点を含むことを示せ。

661 名前:132人目の素数さん:05/02/06 02:17:53
>>660
一辺がdの正三角形の頂点ABCを考えて
ABが同色でなく、ACも同色でなければ
BCが同色になるではありませんか的な論法でいいのかな

662 名前:132人目の素数さん:05/02/06 10:33:04
>>661
おお、そんな簡単だったのか

663 名前:132人目の素数さん:05/02/06 10:50:58
>>656
>ソラリゼーションすか
良くそんな言葉を知っているな。
死語になったかと思っていた。

664 名前:660:05/02/06 12:20:06
>>661
おしい。問題が
「任意の実数dに対し、WかBの少なくとも一方は、
距離dだけ離れた2点を含むことを示せ。」
つまり
∀塗り方 ∀d ∃色 〜
だったらそれで正解。

本問は
∀塗り方 ∃色 ∀d 〜
の形なので、残念ながら不正解ですが、
似たような論法で証明できます。

665 名前:132人目の素数さん:05/02/06 17:41:03
>>660
W=φまたはB=φの時は自明なのでW,B≠φの場合について考える。
まず距離dだけ離れた2点を持つ集合の族をA(d)で表す。
あるd1に対してはW,Bのどちらか一方がA(d1)の元とならない場合
もう一方が任意のdに対してA(d)の元となる事を確かめればよい。

W∈/A(d1)の時、B∈A(d1)は自明(>>661の方法より)。
また、B∈/A(d2) (d1≠d2) となるd2が存在すると仮定すると、
d1>d2であれば、ある白点からd1離れた黒点を考え、白点からd1、
黒点からd2離れた点を考える事ができるが、この点は白点としても
黒点としても矛盾。d1<d2の場合も同様にして矛盾が導ける。
したがってW∈/A(d1)の時はB∈/A(d2)を満たすd2が存在せず、
任意のdに対してB∈A(d)となる。

φは空集合、∈/ は「属さない」って記号のつもり…

666 名前:660:05/02/07 00:38:16
>>665
正解です。
ポイントは、白がd1を含まず、かつ黒がd2を含まないと
仮定すると、三辺が d1,d1,d2 (または d1,d2,d2) の
二等辺三角形を考えたときに矛盾が生じるという点です。

667 名前:660:05/02/07 00:54:46
それでは次の問題。

平面上の各点に、赤または黄または青の色をつける。
赤点、黄点、青点の集合をそれぞれR、Y、Bとする。

このとき、どのような塗り方をしても、任意の実数dに対し、
R,Y,Bのうち少なくとも一つは、距離dだけ離れた
2点を含むことを示せ。

※今回は 「∀塗り方 ∀d ∃色 〜」 です。

668 名前:132人目の素数さん:05/02/07 01:20:00
AB=AC=BC=BD=CD=AE=AF=EF=EG=FG=DGと
なるように点をとる。


669 名前:658:05/02/07 01:34:27
>>659
残念! 654とは他人です。
で、加色混合で白、減色混合で黒になる3色って
解が単一ではないようだが、3色がどのような関係だと
条件に合致するのか興味があったので。
実際にはRGBとCMYでは色空間が異なるけど
ぴったり重なるモデルで考えればどうなる?

670 名前:132人目の素数さん:05/02/07 13:52:41
>>667
背理法により示す。R,Y,Bのどれもdだけ離れた二点を含まないと仮定する。
一辺の長さがdの正六角形を仮定を崩さないように描けば、正六角形に内接する一辺√3dの正三角形は
全ての頂点が同じ色である。

従って、ある頂点から距離√3dになる点の集合(円周)は全て同じ色になる。
しかし、この円周上に適切な二点を選べば距離をdになるようにできるため仮定に反する。

終わり。

671 名前:132人目の素数さん:05/02/07 14:04:12
平面を白黒二色で塗り分ける。三頂点が同じ色で塗られた正三角形を同色正三角形と呼ぶ。

この時、どのような正数dに対しても、ある塗り分けが存在し、
一辺の長さdの同色正三角形が存在しないようにできることを示せ。

672 名前:132人目の素数さん:05/02/07 16:06:52
>>670
>従って、ある頂点から距離√3dになる点の集合(円周)は全て同じ色になる。
ん?なんで?

673 名前:132人目の素数さん:05/02/07 17:25:28
>>670
>ある頂点から距離√3dになる点の集合(円周)は全て同じ色になる。
と仮定すると、六角形の隣り合う2つの頂点から√3d離れた点は
何色だとしても>>670の説明は矛盾する。したがって>>670は間違い。

>>671
まず2つの白点を考え、それぞれの座標を原点(0,0),(2d,0)とする。
同色正三角形が存在しないと仮定すると(d,d√3)(d,-d√3)は共に黒。
この事から(4d,0)は白、さらに(3d,d√3)は黒でなければならないが
この時(2d,2d√3)は白点としても黒点としても矛盾。
したがって任意のdに対して同色正三角形が存在しないようにはできない。

てか、適当な事言って問題にするなよ…

674 名前:673:05/02/07 17:31:43
あー、>>671へのレスは俺が勘違いしてたっぽい。すまん、、

675 名前:132人目の素数さん:05/02/07 20:02:41
通る時通れなくて、通らない時通れるもの、何〜んだ。

676 名前:132人目の素数さん:05/02/07 20:19:37
ふみきり

677 名前:660:05/02/07 22:45:01
>>668一応正解です。
>>670多分、2段落目頭の「ある頂点」は、「中心」の書き間違いだろうと推測。

距離dの2点を含む色が存在しないと仮定すると、
一辺dの正三角形の頂点は赤黄青となるしかない。
さらに、辺-黄青に関して、赤と対称な点を取ると、
そこも赤にならざるを得ない。
これは、ある赤点を中心とした半径d√3の円周上は
全て赤であることを意味する。従ってその円周上からは
赤赤で距離dのペアがいくつでも取れる。矛盾。

最初に考える赤黄青+赤の菱形をABC-D、
次に描く菱形をAEF-Gとしたときの状況を
端的に表したのが668ですが、さすがにあれは
説明不足でしょう。

678 名前:660:05/02/08 04:26:10
>>671
正三角形の高さに等しい幅のシマシマに塗り分ければいいのかな。
夜中に小便に起き、壁にかかった縞模様のタオルを見てふと思いついた。

簡単にするため d=2/√3 とする(正三角形の高さが1)。
(x,y)を、xの値で塗り分け。
0≦x<1 → 白
1≦x<2 → 黒
2≦x<3 → 白
‥‥

679 名前:132人目の素数さん:05/02/08 05:05:36
>>669
ぴったり重なるモデルって何のことを言ってるかよくわかんないけど
俺のイメージで言わせてもらえば
黒(0,0,0) R(1,0,0) G(0,1,0) B(0,0,1)
C(0,1,1) M(1,0,1) Y(1,1,0) 白(1,1,1)
で、原点が黒か白かの違いじゃないの?

てきとうに3つ単位ベクトルをとってやっても
元の(1,1,1)方向に平行になるように足してやれば
無彩色(黒〜灰〜白)にはなるだろうし、
係数の選び方で明度(黒⇔白)も決められるだろう。
係数にマイナスが許されるかどうか知らんが。

そもそも三原色なんてものは人間の目の感覚に依存した曖昧なもんだし
厳密に加法減法が成立するようなものでもない気がするけど。

680 名前:670,671:05/02/08 14:18:03
あれ? >>670間違い?

距離dになる二点で同じ色に塗られたものは存在しないと仮定する。

ある点の色をRとして、その点を中心とする半径√3dの円周を考える。
仮にこの円周上に含まれるある点がY(またはB)だとすると、二点間の距離が√3dになる
二点が存在する。この二点を頂点にもつ正三角形をABCとして、A(Y)、B(R)とする。
三角形ABCに外接する正六角形をADBECF、その中心をOとし、ADBECF、OをY,R,Bで塗り分けると
二点間の距離がdで同じ色になる二点が存在するため、仮定に反する。

したがって、Rに塗られた点を中心とする半径√3dの円周にある点は全てRである。
ところが、この円周上には距離dとなる二点が存在し・・・以下略。
----
今度は会ってると思うが・・・>>670は単にこれを逆にして書いただけなんだけど・・・

>>671については>>678が正解。

681 名前:訂正してみた:05/02/08 14:47:55
>>667
背理法により示す。R,Y,Bのどれもdだけ離れた二点を含まないと仮定する。
一辺の長さがdの正六角形を仮定を崩さないように描けば、正六角形に内接する一辺√3dの正三角形は
全ての頂点が同じ色である。

この正六角形は平面状のどの位置にでもおけるため、結局√3d離れた二点は全て同じ色でなければいけない。

従って、ある頂点から距離√3dになる点の集合(円周)は全て同じ色になる。
しかし、この円周上に適切な二点を選べば距離をdになるようにできるため仮定に反する。
----
真中に一文だけ追加してみた。

682 名前:132人目の素数さん:05/02/08 14:52:31
平面上の全ての点を白黒で塗り分ける。>>671と同じ意味で同色正三角形を定義する。

任意の正数dに対して、同色正三角形が存在しないように塗り分けることは可能か?

683 名前:132人目の素数さん:05/02/08 14:56:06
間違いなので訂正。


平面上の全ての点を白黒で塗り分ける。>>671と同じ意味で同色正三角形を定義する。
平面上のどの正三角形も、同色正三角形にならないよう塗り分けることは可能か?

684 名前:132人目の素数さん:05/02/08 18:19:37
>>681
ごめん、尽く勘違いしてた。673は露骨に無視して。

685 名前:132人目の素数さん:05/02/08 22:27:26
>>684
俺のほうこそすまん。>>683出題してみたが、
普通に回答済みジャン。 気づかなかったよ。


というわけで再度出題。 (疑問に思っただけで未解決な出題)

平面上のすべての点をR,G,B三色で塗り分ける。
距離d離れた二点のうち、色が同じものに注目する。距離dだけ離れた二点で同色なもの
が二色の場合はf(d)=2、一色の場合はf(d)=1、三色の場合はf(d)=3、となるように
関数f:{x|x>0}→{0,1,2,3}を定義する。

すでに回答済みの問題より任意のdに対してf(d)≠0が成立している。
では、任意の正数dに対してf(d)=1となるような塗りわけは存在するか?

686 名前:132人目の素数さん:05/02/09 02:45:07
>>685
平面全部を一色で塗りゃいいんじゃねーの?

687 名前:132人目の素数さん:05/02/09 03:57:11
ただしRGBはそれぞれ少なくとも2点以上存在ってことなのか?

688 名前:132人目の素数さん:05/02/09 13:39:46
>>686
すまん、その通りだ。 華麗に無視してくれ。

689 名前:132人目の素数さん:05/02/09 16:46:20
 |
 8 < もうだめぽ
 '`
  ̄
数ヲタのクズ 685 が自害しました。
人並みの羞恥心を持ち合わせていたようですね。

ァ '`,、'`,、'`,、'`,、(´▽`) '`,、'`,、'`,、'`,、

690 名前:132人目の素数さん:05/02/09 22:01:03
 ネコちん♪パ〜ンチ
         ─  ─.   -   ─  --  - ___
   ∧∧ ─..    -   -     ─_─/___)   ∧∧ .∴・
  ( =^-゚)  -_ ─   ─ -_  _- ─ / (___∧ガッ(::)Д´)>>688.∴・
  ⊂  つ□XXXXXXXXXXXXXXXX|□| (__< ...>.;/つつ
 〜(つ ノ       _  ̄ ─_ ─ ─ ─ヽ (_____∨ ~て つ
,,,,,,,,, (ノ    _     -  -  ─ _- ─\_(__ノ    (/


691 名前:660:05/02/10 00:40:35
>>680
失礼、670で正解ですた。読み違えていた。

>>688
これだったらどうよ。未解決。

平面上のすべての点をR,G,B三色で塗り分ける。
どの色も、それぞれ、ある距離dだけ離れた2点を含まないように
塗り分けることは可能か?

すなわち、たとえば
Rが距離1の2点を含まず、
Gが距離2の2点を含まず、
Bが距離3の2点を含まない。
とすることは可能か。

※660の三色バージョンです。つまり否定を考えると
「どのような塗り方をしても、RGBの少なくとも一つは、
任意の距離dだけ離れた2点を含む。」と言えるか?
となる。

692 名前:132人目の素数さん:05/02/10 03:09:39
なんかオナニースレになってる

693 名前:132人目の素数さん:05/02/10 03:46:44
板自体がオナヌィ板ですから。

694 名前:132人目の素数さん:05/02/10 04:29:09
納得

695 名前:132人目の素数さん:05/02/13 09:37:32
昨日秋山仁がテレビで
下の4つの○を結ぶ最短の線は

○        ○
  \     /
   >──<
  /     \
○        ○

このようになるって言ってたんだけどなんで?
ちなみに線が交わる所は角度が120゚になるそうです。

696 名前:132人目の素数さん:05/02/13 11:39:27
>>695
条件は?
4つの○を結ぶとは?
最短とは何を持って最短?

697 名前:132人目の素数さん:05/02/13 11:44:08
>>696
おまいのチンコの長さをもって最小と定義する

698 名前:132人目の素数さん:05/02/13 11:47:23
1+√3 < 2√2
(約2.732 < 約2.828)


699 名前:132人目の素数さん:05/02/13 11:47:50
>>697
わたしは、その問に対するすばらしい解法を思いついたのだが
それを図示するには余白が狭すぎる

700 名前:132人目の素数さん:05/02/13 12:27:57
Melzakのアルゴリズムで…

701 名前:132人目の素数さん:05/02/13 12:47:14
>>696
結ぶとは
・全ての○が線で「一つに」つながっている。
・線は何度でも分岐してていいし、曲線でもいいし、複数でもいいと思います。
・その他制限は特にないみたいです。
最短とは
・線分の長さを合計してそれが最も短い。
だと思いますが、これでは曖昧ですか?

またこの問題は○の数、並べ方によらず、
必ず分岐点では角度が120゚になるように結ばれるという事だったのですが
それが何故なのか全く分からなかったもので・・・
数学的にはどのように考えればいいんですか?

702 名前:132人目の素数さん:05/02/13 13:06:37
>>698
もっと短いつなぎ方が存在する可能性を否定できてないわけですが。

703 名前:132人目の素数さん:05/02/13 15:09:00
ユークリッド幾何じゃなくてもいいの?

704 名前:132人目の素数さん:05/02/13 15:12:00
ユークリッド幾何でお願いします。

705 名前:132人目の素数さん:05/02/13 15:24:25
節になる点は4都市を頂点とする正方形の中(周も含む)にあることがわかります。
また、節になる点は2個以下でなければならないことがわかります。
4都市と節になる点を頂点とする連結な道路の組み合わせは(節になる点の位置を無視すれば)有限個しかありません。
この組み合わせのそれぞれにおいて、節になる点(2つか1つ)を正方形の中で動かした場合、道路の長さの和は有開閉集合上の連続関数となりますので最小値を持ちます。
それぞれの組み合わせの最小値になる配置が求めるものです。
もし道路の長さの和が最小なら節になる点からは3本の道路が出ていて120度ずつ角をなしていることもわかりますので候補は絞られます。
結果的にこれになります。

706 名前:132人目の素数さん:05/02/13 15:41:14
705で納得できる人がいるんだろうか。
いや、705はそれで納得してるんだろうか。

707 名前:132人目の素数さん:05/02/13 16:57:48
>>701
それだと、対角線を引いたものの方が
短いと思うが?

条件間違ってないか?


708 名前:132人目の素数さん:05/02/13 17:18:19
分岐は三叉路に限る  かな?


709 名前:132人目の素数さん:05/02/13 17:22:47
>>707
いや、直感的には対角線の方が短いように思えるけど
実際にやると>>695のようにした方が短いみたいです。
それを端的に示しているのが>>698
一辺の長さが1の正方形の頂点を結ぶ場合、
>>695の長さは1+√3、対角線で結んだものは2√2になります。
秋山仁さんのスレでも話が出てたみたいですけど
シュタイナー木といってあまり簡単な問題じゃなさそうですね…。

>>708
分岐にも制限はないようです。

710 名前:132人目の素数さん:05/02/13 17:25:04
>>708
分岐点間の距離が短い方が線分の総和が小さくなるから
120°を成す所というのは、その条件だと最小ではなくなる。
というか最小値が存在しなくなるのでは?

711 名前:132人目の素数さん:05/02/13 17:27:48
あのな烏賊歯科の過去問あさってみろ まったく同じ問題がある

712 名前:132人目の素数さん:05/02/13 17:28:04
>>709
線分の総和が最小になるという条件では明らかに対角線の方が短いぞ


俺が言いたいのはだな、条件を正確に書けってことだよ!

713 名前:132人目の素数さん:05/02/13 17:30:54
>線分の総和が最小になるという条件では明らかに対角線の方が短いぞ
なぜ?

714 名前:132人目の素数さん:05/02/13 17:36:19
>>695
それ間違いじゃない?対角線で結んだほうが短いに決まってる
対角線で結んで、真ん中の横の線との交点見て、三角不等式使えばわかる。

715 名前:132人目の素数さん:05/02/13 17:37:44
>>713
まず"線分の総和"というのは俺の中では"道のりの総和"
とは思えないということを言っておこう

>>695のような図にたいして、4点を左上から時計回りにABCDとする。
分岐点を左からE,Fとする。

おれのいってる"線分の総和"というのはAE+DE+EF+FB+FCのこと
おそらくよくある問題で問われるのはそれではないだろう。

716 名前:132人目の素数さん:05/02/13 17:41:14
ちなみに、道のりと言ってもどういう道のりのことなのかは
>>701からは全く読み取れない
寧ろ誤解を招く

717 名前:132人目の素数さん:05/02/13 17:45:29
最短ネットワーク問題だっけかね

718 名前:132人目の素数さん:05/02/13 17:51:07
>>714-715の内容を読んでちょっと気が楽になった。
>おれのいってる"線分の総和"というのはAE+DE+EF+FB+FCのこと
俺もそういう意味で言ってるよ。
>>714-716
とりあえず一辺の長さが1の正方形の頂点の場合で考えようか。
>>695の場合AE=DE=FB=FC=1/√3、FB=1-(1/√3)ってのは計算できる?

719 名前:132人目の素数さん:05/02/13 17:51:39
高速道路ってのは1メートルつくるのにウン百万円もかかったりするもんだ。
4都市を結ぶもっとも安上がりな結線方法ってのが重要なわけ。
線分の総和ってのはそういう意味で、対角線ではないことは自明だろ?

720 名前:132人目の素数さん:05/02/13 17:54:25
ちゃんと数学的に定義しろっつーことだな

721 名前:718:05/02/13 17:55:08
ごめんごめん。最後の式はEF=1-(1/√3)ね。

722 名前:132人目の素数さん:05/02/13 18:03:43
>>718
ちゃんとABCDの位置関係を定義してからにしてもらおうか

723 名前:132人目の素数さん:05/02/13 18:09:06
>>722
一辺の長さが1の正方形の頂点の場合って書いてるけど?

724 名前:132人目の素数さん:05/02/13 18:16:05
>>723
全てを理解した。俺が間違ってたようだ。すまん。
無駄になってる道があるってことだな。

725 名前:132人目の素数さん:05/02/13 18:17:14
アメリカが全米の拠点都市間を結ぶハイウェイ建設を計画した時に、もっとも経費を
削減できる(総延長の短い)計画の作成を数学者は依頼された。
想像以上の複雑で困難な計算に数学者は苦戦した。

結局、針金で各拠点の位置を模した模型を石鹸水に浸して持ち上げ、針金間に張った
シャボンの膜をもとにハイウェイ網は計画されたという。
表面張力によりシャボンの膜は総延長を最短にしようとする。各地に120度で
交差する三叉のインターチェンジが建設された背景には石鹸水の秘密があった。

民明ry

726 名前:132人目の素数さん:05/02/13 18:23:31
>>724
いや、気にしなくていいよ。勘違いする事ぐらい誰でもあるさ。
それにしてもいかに自分を抑えるか、ちょっとした修行になるな‥。

727 名前:132人目の素数さん:05/02/13 18:23:59
この論文を読んでいるとどんな位置にあっても120°になるようだ
また一般のn(≧3)についても書いてあるが、長!

728 名前:132人目の素数さん:05/02/13 18:40:13
地球が丸いことを考慮していないな。。。

729 名前:132人目の素数さん:05/02/13 18:46:18
おまえらが明日チョコレートもらえる確率は?

730 名前:132人目の素数さん:05/02/13 18:47:11
バレンタインのHはギリマン?

731 名前:132人目の素数さん:05/02/13 18:49:06
>>728
それは考慮したくなかったがために政府には説明しなかったって背景もありそう。

732 名前:132人目の素数さん:05/02/13 18:53:15
長野のトンネルは長距離でアップダウンが多くて危険だね。

733 名前:132人目の素数さん:05/02/13 20:06:35
サイコロを3つ転がしたとき、出た目が連続,またはゾロ目になる確率を求めよ
但しサイコロのどの目も出る確率は等しいものとする

○ 1.2.3 2.2.2 など
× 2.2.3 5.6.1 1.3.5など

734 名前:132人目の素数さん:05/02/13 21:03:38
(i,i,i) (i=1〜6)は各1通り、計6通り
(i,i+1,i+2) (i=1〜4) は各6通り、計24通り
よって(6+24)/6^3=5/36

735 名前:132人目の素数さん:05/02/13 22:23:21
更新
http://www3.tokai.or.jp/meta/gokudo-/omoshi-log/

736 名前:132人目の素数さん:05/02/13 23:20:27
ルート5をaとおいたとき、ルート0.45をaを使って表せ

737 名前:132人目の素数さん:05/02/13 23:38:41
(a/a+a/a+a/a)/(a+a)

738 名前:132人目の素数さん:05/02/14 01:43:24
√0.45=√(45/100)=√45/√100=3*√5/10=(3/10)*√5=(3/10)*a

739 名前:132人目の素数さん:05/02/14 22:19:00
平面上に3点A,B,Cがあるとき、
AP + BP + CP となる 点P は
1) ∠CAB >= 120度の時 P = A
2) ∠ABC >= 120度の時 P = B
3) ∠BCA >= 120度の時 P = C
4) ∠CAB < 120度 & ∠ABC < 120度 & ∠BCA < 120度のとき
は∠APB = ∠BPC = ∠CPA = 120度なる点P
で与えられる。

740 名前:132人目の素数さん:05/02/14 23:15:01
平面上にn点ある場合
適当に接続点になる点をいくつか加えて、
これらの点を頂点とする連結なグラフ
(グラフ上をたどってどの頂点からも他のどの頂点へも行ける)
のうちで辺の長さの総和が最小となるものはどのようなグラフか。

>>739を証明済みとする
長さの総和が最小となっていると仮定する。
a) 接続点から1本だけの辺が出ていることはない。
∵1本だけの辺が出ていればその点と辺を削除した方が総和は小さい。
b) 接続点から2本だけの辺が出ていることはない。
∵2本だけの辺が出ていればばその点と辺を削除して、その辺がつながっていた2つの頂点を直接結んで辺とした方が総和は小さい。
c) 接続点Cから4本以上の辺が出ていることはない。
∵4本以上の辺が出ているならば接続されている隣り合う2辺のうちの少なくとも一組は120度より小さい。
それらの辺のもう一端をそれぞれA,Bとして>>739を適用する。
>>739の1)の時は辺CBを削除して辺ABを結んだグラフのほうが総和が小さいので仮定に反する。
>>739の2)の時は辺CAを削除して辺ABを結んだグラフのほうが総和が小さいので仮定に反する。
>>739の4)の時は得られたPを用いて辺CAと辺CBを削除して、
辺AP , 辺BP , 辺CP を加えたグラフのほうが総和が小さいので仮定に反する。
結果的にCに接続される辺は1本減ることになってしまうので仮定に反する。
結局、接続点からは辺はちょうど3本出ていることになる。
しかも隣り合う辺のなす角度は120度である。

また同様にもとのn点に対しては接続される辺の数は3本以下であることもわかる。


741 名前:132人目の素数さん:05/02/15 01:00:00
>>739
が最小。


742 名前:132人目の素数さん:05/02/15 02:17:13
>>739が最小?
>>739AP + BP + CPが最小

743 名前:132人目の素数さん:05/02/15 11:34:32
>>740
おみごと!

744 名前:132人目の素数さん:05/02/15 21:46:02
>>729
漏れ婚約中だからトリビアルに1以上


745 名前:132人目の素数さん:05/02/15 22:34:04
>>744
non-trivial

746 名前:132人目の素数さん:05/02/16 00:42:40
>>740のつづき
もとの点の数を n 点
接続点の数を m 点とする
辺の数を l 本とする

やはりこの連結グラフの辺の長さの総和が最小となっていると仮定する。
d) サイクル(閉じた道)を含んでいない。
∵含んでいればサイクルのうちの一辺を削除しても連結であり総和が減ってしまい仮定に反する。

e) dのようなグラフでは 辺の数 は 頂点の数-1 である。
∵有限個の頂点なのでであるので1辺しかつながっていない頂点がある。
この頂点1辺を削除しても 頂点の数 - 辺の数は変わらない。
次々に削除していくと、1点になる 頂点の数 - 辺の数 = 1- 0 = 1
もとのグラフでも 辺の数 = 頂点の数 - 1が成り立っていたことになる。

>>740のグラフにおいて
f) m <= n - 2 がなりたつ
n点から1辺以上、m点から3辺出ているので、
辺の数 l は( n + 3 m) / 2 本以上ある。
一方e)より l = n + m - 1
すなわち n + 3 m <= 2 (n + m - 1)
m <= n - 2である

g) 接続点はもとのn点を含む最小の凸多角形のの内部か周上にある。
接続点を消して凸多角形に一番近い点を接続点に動かせば(接続辺も消さずに動かせば)総和はより小さくなる。もともと最小だったという仮定に反する。
とくに>>705の前半が言えたことになる。


747 名前:132人目の素数さん:05/02/16 03:03:48
連日出題・解答が続いてた塗り分け問題が途絶えた理由

748 名前:132人目の素数さん:05/02/16 07:13:46
>>747
単純に秋田から

749 名前:132人目の素数さん:05/02/17 00:47:27
akippoiyatsume

750 名前:132人目の素数さん:05/02/17 22:52:57
トーラス上の地図で塗り分けるのに7色必要なものを作れ
(答えを掲示板上でどう書くんだ、と思うでしょうがそれも問題のうち)

751 名前:132人目の素数さん:05/02/18 00:00:34
明らか

752 名前:132人目の素数さん:05/02/18 03:23:41
>>750
          ...,、 -  、 
      ,、 '  ヾ 、    丶,、 -、 
     /    ヽ ヽ  \\:::::ゝ 
 /ヽ/   i  i    ヽ .__.ヽ ヽ::::ヽ 
 ヽ:::::l i.  l  ト  ヽ  ヽ .___..ヽ 丶::ゝ 
 r:::::イ/ l  l.  i ヽ  \ \/ノノハ  ヽ 
 l:/ /l l.  l  i  ヽ'"´__ヽ_ヽリ }. ',  ', 
 'l. i ト l  レ'__    '"i:::::i゙〉l^ヾ  |.i. l    ____________ 
. l l lミ l /r'!:::ヽ    '‐┘ .} /  i l l  /想像力働かせましょう。 
  l l l.ヾlヽ ゝヾ:ノ   ,     !'"   i i/ i< その程度自分でやりましょう。 
  iハ l  (.´ヽ     _   ./    ,' ,' '  | 脳味噌ありますか? 
   |l. l  ` ''丶  .. __  イ         |無いんですか? 
   ヾ!        l.   ├ァ 、        \それなら人間辞めましょうよ。 
          /ノ!   /  ` ‐- 、      ̄ ̄ ̄ ̄ ̄ ̄ ̄ ̄ ̄ ̄ ̄ ̄ ̄ 
         / ヾ_   /     ,,;'' /:i 
        /,,  ',. `  /    ,,;'''/:.:.i 

753 名前:132人目の素数さん:05/02/18 16:59:11
5分間でトラックを5周する馬、5分間で10周する馬、5分間で15周する馬がいます。
スタートを同時に出発したとき、次にスタートを同時に通るのは何分後でしょうか?

754 名前:132人目の素数さん:05/02/18 17:15:05
↑最初にこの問題を見たときはまんまと引っかかった。

755 名前:132人目の素数さん:05/02/18 17:18:59
遅くとも5分後には。

756 名前:132人目の素数さん:05/02/18 23:18:18
どこか引っかかる所あるか?

757 名前:132人目の素数さん:05/02/18 23:50:24
何分後に揃うかよく考えろ

758 名前:756:05/02/18 23:52:11
等速なら1分後、でどこか間違ってる?

759 名前:132人目の素数さん:05/02/18 23:52:43
ってメェル欄に書いてありましたな
どう間違うのか分からん

760 名前:754(753とは別人):05/02/19 00:18:53
最初に出された問題は、1分で3周、5周、7周みたいな形だった。
唐突に問題を見せられ、10秒で解けと言われて、1周を3分5分7分だと
「まんまと」誤読して105分後と即答。相手は大喜び。

頭の良い皆さんには想像もつかないことかもしれないがね。

761 名前:132人目の素数さん:05/02/19 00:23:08
いや10秒で、とか言われたら落ち着いて解けないから間違うでしょう

762 名前:132人目の素数さん:05/02/19 08:13:54
◎1分後
○遅くとも5分後

△5分後
×15分後、10分後、など それ以外


763 名前:132人目の素数さん:05/02/19 16:58:53
「A地点からB地点まで行きは時速40km、帰りは時速60kmでした。
 往復の平均時速は何kmでしょうか。」
私は、この問題に即答で50kmと答えてしまいました。
それも、友人に指摘されるまで間違いに気がつきませんでした。


764 名前:132人目の素数さん:05/02/19 20:37:40
x/40+x/60=3x/120+2x/120=5x/120=x/24
x/24/2/1=x/48
時速48km?

765 名前:132人目の素数さん:05/02/19 21:14:55
0でしょ。

766 名前:132人目の素数さん:05/02/19 22:17:05
>>764
正解!

767 名前:132人目の素数さん:05/02/20 03:42:30
Harmonic mean ですな ( ゚∀゚) テヘッ

768 名前:132人目の素数さん:05/02/20 14:36:35
>>763
平均の定義による。
時間の平均なら時速48kmだが、距離の平均なら時速50km。


769 名前:132人目の素数さん:05/02/21 13:07:30
age

770 名前:132人目の素数さん:05/02/23 03:04:25
結局A地点から動いていないのだから0km

771 名前:132人目の素数さん:05/02/23 14:18:10
どんな4桁の数字でも、↓を繰り返すと6174になる
      (4つバラして大きい順に並べた数)−(4つバラして小さい順に並べた数)

 例:1234 → 4321−1234=3087 
.        → 8730−0378=8352
.        → 8532−2358=6174
.        → 7641−1467=6174

772 名前:132人目の素数さん:05/02/23 15:33:05
1111

773 名前:132人目の素数さん:05/02/23 23:25:09
カプリカ数って奴ですか。

774 名前:132人目の素数さん:05/02/23 23:31:16
名前ついてたんか。
よく知ってるね。

775 名前:132人目の素数さん:05/02/24 00:56:22
へーこりゃ面白いな

776 名前:132人目の素数さん:05/02/24 01:10:53
【トリビアの泉】
カプレカ数で検索すると、自分の名前を付けてるイタイHPがヒットする。

777 名前:132人目の素数さん:05/02/24 11:50:20
6174になるのに
8回以上かかる数はなく
7回かかる数は1980もある
ってのがなんか不思議。

778 名前:132人目の素数さん:05/02/28 17:42:27
どうやって証明すればいいんだろう。
ぞろ目ではできないんですよね?

779 名前:132人目の素数さん:05/03/01 12:02:59
age

780 名前:132人目の素数さん:05/03/02 00:23:03
隣の家の夫婦には子供が3人いる。この時次の問に答えよ
1)子供が男の子1人と女の子2人である確率を求めよ
2)子供のうち1人は女の子であった。この時男の子1人と女の子2人になる確率を求めよ。
3)女の子の子供は2番目の子供だった。この時男の子1人と女の子2人になる確率を求めよ。

781 名前:132人目の素数さん:05/03/02 03:09:28
1)3/8
2)1/2
 女一人をどけて、残り2人が男女1人ずつの確率を出しても良いし
 女3人 (1/8)(3/3)、女2男1(3/8)(2/3)、女1男2(3/8)(1/3)、男3(1/8)(0/3)
 の合計を分母とし、女2男1を分子として求めてもよい
3)1/2
 2)と同じで2番目の子供をどけて考えてもいいし
 何番目の子か考慮してもしなくても同じ。

782 名前:132人目の素数さん:05/03/02 04:40:25
2)は 3/7ではなかろうか?

783 名前:132人目の素数さん:05/03/02 06:10:21
>>782
(1/8)*(3/3) (3/8)*(2/3) (3/8)*(1/3) (1/8)*(0/3) を
(1/8) (3/8) (3/8) (1/8)*0 にしている

784 名前:132人目の素数さん:05/03/02 10:55:13
その数字の羅列の意味はわからんが
答えは3/7だぞ

785 名前:132人目の素数さん:05/03/02 14:08:33
この手の問題は、「隣家の夫婦」というような現実的な設定を用いることにより、
解答者が自発的に余計なことを考え始めて混乱するような書き方に(意図的に)
してあるところがミソ。

すなわち問題文が無駄な情報を含まず、余計なことを考慮する余地のないように
書かれていれば、何の芸もない問題となる。

786 名前:変形版:05/03/02 14:16:30
3人の子供を持つ日本の家庭を全て集めて全体集合Uとする。
3人の子供には、誕生時刻により上中下と順序を入れておく。

このとき子供の男女の組合せは8通りあるが、それらのいずれも
全体に対し占める割合が等しく1/8ずつであると仮定する。
つまり世の中には3人の子供を持つ家庭が8戸しかなく、
全て上中下の男女の組合せが異なっているものとみなしてよい。

ここで
U = 全体集合(8戸)
A = 少なくとも1人の女の子がいる家庭の集合
B = 中が女の子である家庭の集合
C = 男1人、女2人である家庭の集合(順序は問わない)
とするとき、

(1)|C|/|U|
(2)|C|/|A|
(3)|C|/|B|

をそれぞれ求めよ。

787 名前:132人目の素数さん:05/03/02 15:23:08
|U|=8,|A|=7,|B|=4,|C|=3

(4)|C|/|B∪C|
を求めよ。

788 名前:132人目の素数さん:05/03/02 20:01:53
「子供のうち1人は女の子であった。」を単純に「男3兄弟でない家庭」と考えてもいいのだろうか?
「子供のうち1人は女の子であった。」とわかった過程によっては確率が変わると思う。
たとえば、「たまたま子供の一人が出かけるところを見かけた。」だと
その家庭は、「男2女1」よりも「女3」である家庭である確率のほうが高いと思われるのだが。

789 名前:132人目の素数さん:05/03/02 21:39:16
ん?3〜4行目は確かに正しいけど
1行目は数学以前の問題で日本語的に疑う余地がないでしょ。
女の子がいる時は3〜4行目の条件付確率は等しくならないけど
そもそも女の子がいない時は関係ない。

790 名前:132人目の素数さん:05/03/02 23:39:54
>>780
前提条件として、ある子供が男である確率と女である確率は等しいというのを仮定する必要がある。
2)は、「子供のうち少なくとも1人は女の子であった。」とするか、「1人は女の子であとの2人についてはわからない。」としたほうがより正確になる。


791 名前:132人目の素数さん:05/03/03 01:20:40
>>786
変形の過程で(2)の問題が変わっている。
答えを3/7にしている>>782>>784の間違いの原因もそこ。

子供2人の問題で
少なくとも一人が女と分かっているとき
男・女である確率を2/3にしちゃうのと同じ間違いだな。

792 名前:132人目の素数さん:05/03/03 05:04:43
・ジョーカーを除くトランプ52枚の中から1枚のカードを抜き出し
表を見ずに箱の中にしまった。
残り51枚のカードをよくシャッフルしてから3枚抜き出したところ
3枚ともダイヤであった。このとき、箱の中のカードがダイヤである確率は
  A)10/49 B)1/4

・3枚のカードがあり、それぞれ両面赤、両面黄、片面赤・片面黄である。
この3枚のうちの1枚がテーブルの上に置いてあり、見えている面が赤のとき
その裏が赤である確率は
  A)2/3 B)1/2

・隣家には子供が2人いることがわかっている。性別はわかっていない。
聞こえてくる声から、一人は女の子だと分かった。
このとき、隣家の子供が男女1人ずつである確率
  A)1/2 B)2/3

・隣家には子供が2人いることがわかっている。性別はわかっていない。
そのうちの一人が訪ねてきた。女の子だった。
このとき、隣家の子供が男女一人ずつである確率
  A)1/2 B)2/3

793 名前:132人目の素数さん:05/03/03 05:15:47
>>792
3番目と4番目は同値な問題じゃん!
単なる条件つき確率だから、 答え 2/3

794 名前:132人目の素数さん:05/03/03 07:08:45
普通はa,bという二人を考えた時、aの性別はbの性別に依存しない。
考えられる組み合わせが全て同確率という仮定があるわけではないから。

795 名前:132人目の素数さん:05/03/03 07:22:59
隣家には子供が2人いることがわかっているが性別はわかっていない。
このとき、隣家の子供が男女1人ずつである確率は 1/2

隣家には子供が2人いることがわかっているが性別はわかっていない。
聞こえてくる声から、一人は女の子だと分かった。
あるいは、そのうちの一人が訪ねてきたが、女の子だった。
このとき、隣家の子供が男女1人ずつである確率は 2/3

でOK?

796 名前:1/2:05/03/03 11:22:53
>>795
なんも分かってない。それで何かを説明してるつもりなの?そもそもどういう
方法で2/3を出してるのか書かれてないから何とも言えないけど、それは多分
「ある人がコインを10回投げました。ビデオで確認すると1〜9回目までは
全て表でした。では10回目が裏の確率aは?」という問題でa>1/2という答えに
なってるのと同じ間違いをしてると思う。>>794でも書いたけどそれぞれの性別が
男か女かって言うのは独立事象だとは思わないの?子供の性別の組み合わせ
(4通り)がそれぞれ等しく1/4である事は仮定ではなく、何も条件がない状態で
ある一人が男である確率が1/2(これが前提)から導かれる事。
つまり条件付なら「それぞれの組み合わせが等確率」という所が変わる。
具体的に言うと、
━━━━━━面倒ならここから読め。━━━━━━
兄弟が(男、男)、(男、女)、(女、男)、(女、女)である事象をそれぞれ
MM, MW, WM, WW とするとそれぞれの確率は、
 P(MM)=P(MW)=P(WM)=P(WW)=(1/2)x(1/2)=1/4
また、訪ねて来た一人が女であるという事象をAとするとその確率P(A)は
 P(A)=P(MM)x0+P(MW)x(1/2)+P(WM)x(1/2)+P(WW)x1
    =(1/4)x0+(1/4)x(1/2)+(1/4)x(1/2)+(1/4)x1 =1/2

797 名前:2/2:05/03/03 11:23:47
ここで、訪ねて来た一人が女であり、かつ兄弟が男一人女一人である確率は
 P(A∩(MW∪WM))
  =P((MW∩A)∪(WM∩A))
  =P(MW∩A)+P(WM∩A) *1
  =P(MW)P(A|MW)+P(WM)P(A|WM)
  =(1/4)x(1/2)+(1/4)x(1/2) =1/4 …(1)
また、
 P(A∩(MW∪WM))
  =P((A∩MW)∪(A∩WM))
  =P(A∩MW)+P(A∩WM) *1
  =P(A)P(MW|A)+P(A)P(WM|A)
  =(1/2)(P(MW|A)+P(WM|A))
  =(1/2)P((MW∪WM)|A) *1
  =1/4 …(1)より
したがって訪ねて来た一人が女であったという条件下での
兄弟が男一人女一人であるという確率は
P((MW∪WM)|A)=1/2,,

*1ではMWとWMが排反である事を使った。

こうやって説明汁。

798 名前:132人目の素数さん:05/03/03 11:35:59
・隣家には子供が2人いることがわかっている。性別はわかっていない。
そのうちの一人が訪ねてきた。女の子だった。
このとき、隣家の子供が男女一人ずつである確率

これは1/2だね。
こう書き換えればより直観的に分かる。

・隣家には子供が2人いることがわかっている。性別はわかっていない。
そのうちの一人が訪ねてきた。性別を確認した。
このとき、隣家のもう一人の子供が今の子供と性別が異なっている確率

1/2に決まってる。

799 名前:132人目の素数さん:05/03/03 11:43:00
釣りか?マジで言ってるのか?

800 名前:132人目の素数さん:05/03/03 11:51:40
狐につままれるような感覚は理解できるけどな。俺にも経験あるから。

801 名前:132人目の素数さん:05/03/03 12:02:48
親御さんに「2人の子供のうち1人を紹介して下さい」と言って
出てきた子の顔を見たら女の子だった。
このときもう一方が男の子である確率は?答え1/2

親御さんに「2人の子供のうち少なくとも一人は女の子ですか?」
と聞いたら「はい」と答えた。
このときもう一方が男の子である確率は?答え2/3

女の人の子が尋ねてきた、とか、女の子の声が聞こえた、だと
どっちの場合なのか意見が分かれるので良くない

802 名前:132人目の素数さん:05/03/03 12:17:08
は?ちょっと待て

803 名前:132人目の素数さん:05/03/03 12:35:34
どうした

804 名前:132人目の素数さん:05/03/03 12:46:13
>>801
「ある一人が女であるか」という事と「少なくとも一人は女であるか」という事の
違いは、前者は『偶然』選ばれた一人が女であるというだけYESであり、もう一人が
男か女かは関係ない。しかし後者はどちらかさえ女ならYESとなる。
これは例えばモンティホールジレンマで残りの箱を適当に開けられる場合と
外れを知っている司会者が外れを開けてみせる場合に相当する。
一人訪ねてきてそれが女であった場合、女を選んで確認できたわけではないので、
これは明らかに前者であり、意見の分かれる余地はない。
つまりあの問題で2/3と答えるのは間違い。

805 名前:804:05/03/03 12:48:06
>前者は『偶然』選ばれた一人が女であるというだけYESであり、
 ↓
>前者は『偶然』選ばれた一人が女であるという時だけYESであり、

806 名前:798:05/03/03 14:11:26
>>799
マジで言ってるけど、何か問題ある?

807 名前:795:05/03/03 14:41:59
795 : >>796-797へ、いま返事書いてます

796-797 : うるせー、ゴルァ!

795 : 条件つき確率を知ってますか?

796-797 : 馬鹿にすんか、ゴルァ!

795 : あなたの説明の中では、条件つき確率を求めていませんよ

796-797 : ちっ、うまく誤魔化したんだが、バレちまったか

795 : いま、あなたの導入した記号で計算していますから、もう少し待ってね。

808 名前:132人目の素数さん:05/03/03 14:50:32
隣家に2人の子供がいることが分かっているとする。

A : 隣家に男女一人ずつの子供がいる事象
B : 隣家から女の子が尋ねてくる or 女の子の声がする事象

隣家の子供が、年齢の高い順から 男男、男女、女男、女女 である事象を
MM、MW、WM、WW

隣家に男女一人ずついる確率
P(A) = P(MW)+P(WM) = 1/4 + 1/4 = 1/2

隣家から女の子が尋ねてくる確率
P(B) = P(MM)*0 + P(MW)*(1/2) + P(WM)*(1/2) + P(WW)*1 = 3/4

隣家に男女一人ずつの子供がいて、女の子が尋ねてくる確率
P(A∩B) = P(MW)*(1/2) + P(WM)*(1/2) =1/4 + 1/4 = 1/2

隣家から女の子が尋ねて来たときに、隣家が男女一人ずつの子供がいる条件つき確率 P_B(A)
高校生でも知っている P(A∩B) = P(B)*P_B(A) より、1/2 = (3/4)*P_B(A) を解いて P_B(A)=2/3 (終)


>>796-797
> なんも分かってない。それで何かを説明してるつもりなの?

あのな、何かを説明するとはこう言うことだよ。オービー君!
まずは、条件つき確率を勉強してから、えらそうなことを言いたまえ!
目立つように晒し上げておく。

809 名前:132人目の素数さん:05/03/03 14:58:28
>807-988
(゚∀゚) イイッ!
よくある質問に入れておくといいかも・・・

810 名前:BlackLightOfStar ◆ifsBJ/KedU :05/03/03 15:00:02
Re:>808 どんな計算してるんだよ?P(B)=1/2,P(A∩B)=1/4だろが。

811 名前:132人目の素数さん:05/03/03 15:00:46
>隣家から女の子が尋ねてくる確率
>P(B) = P(MM)*0 + P(MW)*(1/2) + P(WM)*(1/2) + P(WW)*1 = 3/4

>P(B) = P(MM)*0 + P(MW)*(1/2) + P(WM)*(1/2) + P(WW)*1 = 3/4
>P(B) = P(MM)*0 + P(MW)*(1/2) + P(WM)*(1/2) + P(WW)*1 = 3/4
>P(B) = P(MM)*0 + P(MW)*(1/2) + P(WM)*(1/2) + P(WW)*1 = 3/4

晒し上げ。

812 名前:132人目の素数さん:05/03/03 15:02:56
先生!>>807の意味が分かりません!

813 名前:132人目の素数さん:05/03/03 15:04:00
謝罪まだー

814 名前:132人目の素数さん:05/03/03 15:06:23
スコココバシッスコバドドトスコココバシッスコバドドトスコココバシッスコバドドトスコココバシッスコバドドトスコココ
スコココバシッスコバドドドンスコバンスコスコココバシッスコバドト _∧_∧_∧_∧_∧_∧_
スコココバシッスコバドドト从 `ヾ/゛/'  "\' /".    |                    |
スコココバシッスコハ≡≪≡ゞシ彡 ∧_∧ 〃ミ≡从≡=< 謝罪と切腹まだーーー!!!!!  >
スットコドッコイスコココ'=巛≡从ミ.(・∀・# )彡/ノ≡》〉≡.|_ _  _ _ _ _ ___|
ドッコイショドスドスドス=!|l|》リnl⌒!I⌒I⌒I⌒Iツ从=≡|l≫,゙   ∨  ∨ ∨ ∨ ∨ ∨ ∨
スコココバシッスコバドト《l|!|!l!'~'⌒^⌒(⌒)⌒^~~~ヾ!|l!|l;"スコココバシッスコバドドドンスコバンスコスコココ
スコココバシッスコバドドl|l|(( (〇) ))(( (〇) ))|l|》;スコココバシッスコバドドドンスコバンスコスコココ
スコココバシッスコバドド`へヾ―-―    ―-― .へヾスコココバシッスコバドドドンスコバンスコスコココ

815 名前:132人目の素数さん:05/03/03 15:08:10
>>813-814
誰に謝罪要求してるのかはっきりしろよ。

816 名前:132人目の素数さん:05/03/03 15:10:08
                _
              //.|
             //./|
           //./| |
          //./ /|. |
        //./|/::/| |          _______________
        □/ / // | |.          |
        | |/.;;;;//.  | ||.         | じゃあ、>>807は死刑という事で・・・。
        | | ;;;;;;//   | |||         |_
        | |.;;;//    | |.||     ∧ ∧  |/ ̄ ̄ ̄ ̄ ̄ ̄ ̄ ̄ ̄ ̄ ̄ ̄ ̄
        | |//..    | | ||.    ( ・∀・)
        | |/.     | |. ||    (    )           ワイワイ  ガヤガヤ
 ______.| |___//| ||__ / | | |__
        | |   //  |. ̄∠/(__(__) /.|          ∧_∧ ∧_∧ ∧ ∧.
..∧_∧   (| |⌒/. ∧ ∧⊃イヤァァァ.     //|         (´-`;)(@・ )(;´∀)(
( ・∀・).(⌒| |//(;´Д`) ←>>1   //  |        ∧∧ ∧ ∧ ∧_∧. ∧∧
(    )  ̄| |/ (⊃ /  ⊂.⊃.   //   |       (∀・ )(ー゚* )(   )(´∀`
| | |.   | |    /   └─┘ //   /.      ∧_∧ ∧ ∧ ∧ ∧. ∧_∧
(__)_)   | |  /         //   /       <_`  )(´・ω)(д゚` )(
        | |/         //   /.       ∧_∧ ∧ ∧ ∧_∧. ∧_∧ ∧
        ~~         //   /        (   )( ゚∀゚)(`   )(   )(゚д
.                //   /        ∧_∧ ∧_∧  ∧_∧ ∧_∧ ∧_∧
.               //   /         (д- )(   )( ´,_ゝ)(TдT)(∀` )

817 名前:132人目の素数さん:05/03/03 15:10:49
スコココバシッスコバドドトスコココバシッスコバドドトスコココバシッスコバドドトスコココバシッスコバドドトスコココ
スコココバシッスコバドドドンスコバンスコスコココバシッスコバドト _∧_∧_∧_∧_∧_∧_
スコココバシッスコバドドト从 `ヾ/゛/'  "\' /".    |                    |
スコココバシッスコハ≡≪≡ゞシ彡 ∧_∧ 〃ミ≡从≡=< 807の謝罪と切腹まだーーー!!!!!  >
スットコドッコイスコココ'=巛≡从ミ.(・∀・# )彡/ノ≡》〉≡.|_ _  _ _ _ _ ___|
ドッコイショドスドスドス=!|l|》リnl⌒!I⌒I⌒I⌒Iツ从=≡|l≫,゙   ∨  ∨ ∨ ∨ ∨ ∨ ∨
スコココバシッスコバドト《l|!|!l!'~'⌒^⌒(⌒)⌒^~~~ヾ!|l!|l;"スコココバシッスコバドドドンスコバンスコスコココ
スコココバシッスコバドドl|l|(( (〇) ))(( (〇) ))|l|》;スコココバシッスコバドドドンスコバンスコスコココ
スコココバシッスコバドド`へヾ―-―    ―-― .へヾスコココバシッスコバドドドンスコバンスコスコココ

818 名前:132人目の素数さん:05/03/03 15:11:02
なら同意。

819 名前:132人目の素数さん:05/03/03 15:14:08
このヘタレ! ___ オラッ!      
    ドッカン |   | でてこい、807!
    ∩∩  |   |   |  ∩∩    
   | | | |  |   |   |  | | | |    
  ..(  ,,) .|   |   | (・x・ )  
  /  .つ━━ロ|ロ ドカン l   |U 
〜(   /   |   |   |⊂_ |〜
  し'∪   |   |   |   ∪ 

820 名前:132人目の素数さん:05/03/03 15:16:40
/|         |  |_____ΦΦΦΦΦΦΦΦΦΦΦ||ΦΦΦ
  |         |  | ̄ ̄ ̄ /|                    ||
  |         |  |   / /|TTTTTT   TTTTTTTTTT||TTTTT
  |        /\ |  /|/|/|^^^^^^ |三三| ^^^^^^^^^^^||^^^^^^^
  |      /  / |// / /|
  |   /  / |_|/|/|/|/|
  |  /  /  |文|/ // /
  |/  /.  _.| ̄|/|/|/         Λ_Λ
/|\/  / /  |/ /           (___) おいおい、何が始まったんだ
/|    / /  /ヽ            /〔 祭 〕〕つ
  |   | ̄|  | |ヽ/l            `/二二ヽ
  |   |  |/| |__|/   Λ_Λ     / /(_)
  |   |/|  |/      ( ´∀`)   (_)    Λ_Λ
  |   |  |/      // /  ^ ̄]゚        (`   )
  |   |/        ゚/ ̄ ̄_ヽ         ⊂〔〔 祭 〕
  |  /         /_ノ(_)          ┌|___|
  |/          (__)             (_ノ ヽ ヽ
/                                (_)

821 名前:132人目の素数さん:05/03/03 15:36:11
>807
これも計算してみ!

電話して、女の子がいると聞いたら 「いる」 と言われたとき、2人とも ょぅι゜ょ である確率

822 名前:132人目の素数さん:05/03/03 15:36:42
>>807は多分条件付確率の表記法を知らなかったんだろうな。
だから条件つき確率が求められてないとか言ったんだろう。

823 名前:132人目の素数さん:05/03/03 16:06:56
        r;;;;;ノヾ           >>807
        ヒ‐=r=;'      ∬    君も男なら聞き分けたまえ!
        'ヽニ/  っ━~~   
      _と~,,  ~,,,ノ_.  ∀   
          ミ,,,,/~), │ ┷┳━ 
       ̄ ̄ ̄ .じ'J ̄ ̄| ┃
       ̄ ̄ ̄ ̄ ̄ ̄ ̄  ┻

824 名前:132人目の素数さん:05/03/03 16:08:27
この話題終了。

825 名前:132人目の素数さん:05/03/03 16:12:15
Σ(゚Д゚ エーッ!!
まだ 807 の謝罪と切腹を見てませんっ!

826 名前:132人目の素数さん:05/03/03 16:13:39
いや、俺がもういいって言ってるからいいのw
これ以上発展性ないし。

827 名前:826:05/03/03 16:19:17
        r;;;;;ノヾ           >>825
        ヒ‐=r=;'      ∬    この板では私が神だ!
        'ヽニ/  っ━~~    私がもういいと言ったら、そこで終わりだ!
      _と~,,  ~,,,ノ_.  ∀    君も男なら聞き分けたまえ!
          ミ,,,,/~), │ ┷┳━ 
       ̄ ̄ ̄ .じ'J ̄ ̄| ┃
       ̄ ̄ ̄ ̄ ̄ ̄ ̄  ┻

828 名前:132人目の素数さん:05/03/03 16:20:07
騙るなボケ。

829 名前:132人目の素数さん:05/03/03 16:23:22
実は 826 = 807。

830 名前:132人目の素数さん:05/03/03 16:26:33
>>807=>>786?
まあ根本が間違ってたら
正解が間違いに見えてしまうといういい例だったな。
こういう人材が居るとスレの保養になる。

831 名前:132人目の素数さん:05/03/03 16:28:54
失敗から学ぶ。
転んでも、何かを拾って起き上がる。
勉強になりますね。

( ゚∀゚) テヘッ

832 名前:132人目の素数さん:05/03/03 16:32:57
794=796=797=800=804=805=811=815=818=822=824=826=828
このスレに張り付いてました。

833 名前:132人目の素数さん:05/03/03 16:36:48
新ネタキボンヌ!

834 名前:132人目の素数さん:05/03/03 16:46:15
横から煽るだけの奴ウザい。

835 名前:788:05/03/04 02:28:39
>>789
すまん、書き方が悪かった。

>「子供のうち1人は女の子であった。」を単純に
>「男3兄弟でない家庭」と考えてもいいのだろうか?

「子供のうち1人は女の子であった。」を単純に
「男3兄弟でない家庭が等確率で存在」と考えてもいいのだろうか?

と書きたかった。

先の例の「たまたま子供の一人が出かけるところを見かけた。」だと
その家庭は、「男2女1」よりも「女3」である家庭である確率のほうが高いと思われるが
「女児のいる家庭のみのくじ引き分譲住宅」だと「男3兄弟でない家庭が等確率で存在」となりそうだ。



836 名前:132人目の素数さん:05/03/04 02:34:57
いったい何が等確率に起こるのかが明らかにされていないと
正解は求められないというだけのことではないのか?

837 名前:132人目の素数さん:05/03/04 03:11:20
>>835
>>789じゃないが流れから見て
>>783の2列目と3列目の違いのことを言ってるってことは分かるよ。

>>836
男女の存在確率は特に断ってない場合
半々ってのが暗黙の了承みたいだね。
コインの裏表やサイコロの目と同じで。

838 名前:132人目の素数さん:05/03/04 03:12:10
ジョーカーを除く52枚のトランプからランダムに1枚ずつ3回引いた。
1)引いた3枚が赤(ハート又はダイヤ)2枚・黒(スペードまたはクラブ)1枚
である確率は?
2)3枚の中からランダムに1枚取り出して見たら赤だった。このとき
引いた3枚が赤2枚黒1枚である確率は? 
3)3枚のうち2枚目に引いたカードが赤だった。このとき
引いた3枚が赤2枚黒1枚である確率は?

839 名前:132人目の素数さん:05/03/04 11:54:20
>>836
それと条件もはっきりさせないといけないって事だね。ただその条件が十分厳密に
書かれてると言えない場合には必ず明白な理由があるからあまり疑心暗鬼になる
事もない。今回の場合では条件の範囲が曖昧にされてたりとか。
つまり一見「女の子は少なくともx人」(今回はx=1)っていう同値な命題に変えられ
そうな二つの条件の場合では、そう書き換えることで、全体の中に「少なくともx人」
なのか、ある特定の部分集合(今回は特定の一人)の中に「少なくともx人」なのかと
いう重要な情報が消えてしまってるって話でした。蛇足だけどそういう条件の範囲と
なる部分集合が複数あってさらに排反でない場合もあるから(数学科の人がそんな
面倒なだけの問題に出くわすかどうかは知らんが)、それがいい加減だと一見正解
のようにも見える別の答えに惑わされる事になるよと。あと煽ラーには本気で氏んで
ほしいと思いました。

840 名前:132人目の素数さん:05/03/04 12:23:14
1つのビンに入っているジュースを2人が2つのコップに分けて
平等に飲もうとしている。計測するものが何もない場合、2人がジュースの量に
納得して飲むにはどうしたらいいか。なお、2人のどちらともジュースは出来る限り
多く飲みたいと思っているとする。

答えは、1人がジュースを2つのコップに目分量で半分ずつに分け、もう1人がその2つの
コップを見比べて、飲みたい方(量が多く見える方)を取ればお互いに納得する。

では、これを3人で行う場合、及び4人で行う場合において誰もが納得してジュースが飲める方法を述べよ。


簡単ですかね。




841 名前:132人目の素数さん:05/03/04 12:51:17
>>839
たて読みする気も起こらない

842 名前:132人目の素数さん:05/03/04 13:22:08
たいした事は何も書かれてないから気にしなくていいよ煽ラー。

843 名前:132人目の素数さん:05/03/04 13:44:53
>>840
簡単ではないが既出

844 名前:132人目の素数さん:05/03/05 17:55:12
>>840の答えがわかりませんorz

2人の場合はわかったけど、3人?、4人!?の場合なんて
まったくわかりません・・・。

どなたか教えてください><;

845 名前:132人目の素数さん:05/03/05 19:30:50
一般化してN人で考えてみればいいよ
因みに、要するに1/Nだけ飲めなかったら自分の責任、という
ような方法を求めよ、ということかと

846 名前:BlackLightOfStar ◆ifsBJ/KedU :05/03/05 20:19:23
Re:>840 実は同じような問題が昔平成■■■■会で出たんだよ。

847 名前:132人目の素数さん:05/03/05 20:38:19
アミダラ女王

848 名前:132人目の素数さん:05/03/05 21:15:47
平成教育委員会は見てないから、この解答で正しいかどうかの自信は無いんだけど、

STEP1
 1人目が、与えられた飲み物を自分に納得のいくように、1:(n-1)に分ける。

STEP2
 2人目が、分割された飲み物のうち(n-1)側を自分に納得のいくように、1:(n-2)に分ける。

STEP1〜STEP2を、n-1回繰り返す。

STEP3
 n個に分かれた飲み物を、n人目から、n-1人目、n-2人目・・・、という順番で
 とっていく。


849 名前:132人目の素数さん:05/03/05 21:40:00
一人目が1を1/3と2/3に分ける。
二人目が2/3を1/2と1/6に分ける。
三人目が1/2をとる。
二人目が1/3をとる。
一人目が1/6をとる。


850 名前:BlackLightOfStar ◆ifsBJ/KedU :05/03/05 21:53:56
Re:>848 [>849]が言うのは、最初に分ける人が不利だからこれでは駄目だということ。

851 名前:132人目の素数さん:05/03/05 21:57:27
何故不利?
そんな事言ったら二人の場合も最初のヤシが不利だよ。
この問題は元々数学的ではない。


852 名前:132人目の素数さん:05/03/05 22:10:56
コップ3つあったら
なんか本当に均等に分ける方法がありそうだけど、ないかな。

853 名前:BlackLightOfStar ◆ifsBJ/KedU :05/03/05 22:13:14
Re:>851 二人の場合は最初の人が均等に分ければいいだろう。ところが三人の場合は一人が1/3を切り分けても二人目が変な切り方をすると一人目のとりまえが少なくなる。

854 名前:BlackLightOfStar ◆ifsBJ/KedU :05/03/05 22:15:21
「切り方」っておかしいな。
平成■■■■会ではケーキを三人で分ける問題だった。

855 名前:BlackLightOfStar ◆ifsBJ/KedU :05/03/05 22:21:39
Re:>840 ジュースを少しずつコップについでいく。適当につぎ終わったら自分の取り前にする。ただし、ついでいる途中で他の人がジュースを取る権利がある。これでとりあえず平穏になるだろう。

856 名前:132人目の素数さん:05/03/05 23:38:17
>>855
それが一つの解だな。

ただその「注いでる途中に声をかける」っていう連続的(?)な操作を認めない場合はどうだ?
(昔どこかのスレでこの解答を見た)

857 名前:848:05/03/05 23:54:50
>>848 自己レス。
よく考えたら、漏れの解答だと確かに序盤の分割者が不利ですな。

自分が分けた 1:(n-1) の飲み物のうち、(n-1)側を他人に恣意的に分けられて
それしか選べない状況になったら、確かに納得できない。

>>849
それも三人目の人に選択権が全くないという点でまずいように思う。
というかいまいち説明がわからない。


858 名前:132人目の素数さん:05/03/06 01:52:31
何か昔ピーターフランクルの問題集にあったよ

 まず順番を決めて
STEP1
 1人目が、与えられた飲み物を自分に納得のいくように、1:(n-1)に分ける。
STEP2
 1人目が分けた 1/n のジュースを、2人目以降に順に回していく。
 それが多すぎると思う人は適正と思われる量まで減らし、
 少ない or 丁度いいと思う人は手を付けない。
 n人目まで回して、最後に手をつけた人がその 1/n を貰う。

 nにn-1を代入して最初に戻る

他人のミスで自分のジュースが少なくなるのは不味いですね

859 名前:132人目の素数さん:05/03/06 05:58:19
>>858 と本質的に同じだけど、
高校の物理の先生に教わった解答
(そのときはカステラの分け方だった)

カステラの左端から右へ包丁を動かしていって、最初に「ストップ」と言った人が、
その時点で包丁のある位置から左端までのカステラをもらう。
これで一人減るので、また同じことを繰り返す。

860 名前:132人目の素数さん:05/03/06 07:59:42
>>858-859
なるほど逆オークションみたいな感じだな。勉強になりますた。


861 名前:132人目の素数さん:05/03/06 16:59:20
次の□に当てはまる数字は?
またその理由も答えなさい 

1 3 5 □ 11 25 35


862 名前:132人目の素数さん:05/03/06 17:36:01
>>861
名古屋の人?

863 名前:132人目の素数さん:05/03/06 21:20:06
名古屋ローカルで通用するネタなの、この問題?知らんけど。
だとすれば全然数学関係ないな。時間の無駄だった。


864 名前:132人目の素数さん:05/03/06 21:29:45
>>863
名古屋ネタかどうか知らないけど、
数学ネタでないことは間違いない。

865 名前:132人目の素数さん:05/03/06 21:32:05
うち名古屋だけど33も映るよ

866 名前:132人目の素数さん:05/03/09 07:05:39
f:Z^2→N は、任意の(n,m)∈Z^2に対して
f(n,m)={f(n+1,m)+f(n,m+1)+f(n-1,m)+f(n,m-1)}/4 を満たすとする。
このようなfを全て求めよ。

867 名前:BlackLightOfStar ◆ifsBJ/KedU :05/03/09 13:53:21
Re:>866
fの終域はNなので、fは最小値をとる。
Z^2の点(x,y)で最小値をとると仮定すると、
f(x+1,y),f(x-1,y),f(x,y+1),f(x,y-1)はいずれもf(x,y)以上であり、
この四つのうち少なくとも一つがf(x,y)より大きいとすると、
f(x,y)=(f(x+1,y)+f(x,y+1)+f(x-1,y)+f(x,y-1))/4>f(x,y)となる。
よって、f(x+1,y)=f(x-1,y)=f(x,y+1)=f(x,y-1)=f(x,y)が成り立つ。
fが定数になることはここから容易に分かる。

868 名前:132人目の素数さん:05/03/09 15:45:54
>>861
次の□に当てはまる数字は?
またその理由も答えなさい 

1 3 5 □ 11 25 35

答:□=27/5

869 名前:132人目の素数さん:05/03/09 15:47:30
>866
どこかで見た問題だけど、誰か知ってる?

870 名前:132人目の素数さん:05/03/09 16:46:16
>>869
実数値、複素数値なら見たことあるな。

871 名前:132人目の素数さん:05/03/10 00:37:30
鉄球がたくさん入っている袋が10個あります。
鉄球には10g , 11g , 12gの三種類があり、
一つの袋には同じ種類の鉄球しか入ってません。
グラム単位の計量が可能な台秤を一回だけ使って
どの袋にどの種類の鉄球が入っているか調べる方法を考えてください。

872 名前:132人目の素数さん:05/03/10 01:08:07
無理です。

873 名前:132人目の素数さん:05/03/10 01:25:00
(3^10−1)/2=29524。


874 名前:132人目の素数さん:05/03/10 01:42:41
>>871
あきれた。
一つの袋に同数の鉄球が入っているのかすら定義されていないのだから救いようがない

875 名前:132人目の素数さん:05/03/10 02:35:36
>>871
1つめの袋から順に
1個・3個・9個…を取り出して台ばかりにのっける
とかいう感じの解答

>>874
一つの袋に同数???

876 名前:BlackLightOfStar ◆ifsBJ/KedU :05/03/10 14:11:11
Re:>871 一つずつ鉄球を台ばかりに載せてはかりの変化を見る。

877 名前:132人目の素数さん:05/03/10 14:55:08
>>876◆ifsBJ/KedU 頭悪そう

878 名前:132人目の素数さん:05/03/10 17:17:04
算数の先生こんな問題だしてきた…

@ABCDEFGHまで各1個あります。
これらを全て使って、以下の○を埋めて式を組み立てなさい。

○○○○○
− ○○○○
BBBBB

正解わかりません…

879 名前:BlackLightOfStar ◆ifsBJ/KedU :05/03/10 21:27:15
Re:>877 お前に何が分かるというのか?
Re:>878
5個の2,3,4で和を9で割ったあまりが6になるようにするには、和が15でないといけない。
一方、各桁の上段から下段を引いた値が3になるようにすることは出来ないので、
一箇所または二箇所で繰り下がりがある。
二箇所とも、右の桁で繰り下がりがあるような繰り下がりがあるようにすることはできない。
よって、一つの列にのみ、1-8または2-9の組が現れる。
1-8の組が現れるとすると、7-4の組もできて、3は左に入らねば成らず、残り四つの数は入れられない。
よって、2-9の組があって1-8の組はない。
さらに左に3が入るとすると、6はどこにも入れられないので、
左には4が入る。
次に、1,3,5,6,7,8から、二数ずつ選んで、その差が3または6になるようにすると、
1-7,6-3,8-5の組み合わせのみができる。あとは答えを出すだけ。
41268-7935と41286-7953のみが答え。

880 名前:132人目の素数さん:05/03/10 21:55:48
>879
普段は毒電波ゆんゆんの板荒らしなのに、
薬で頭がシャキッとしてる時はいいこと書くんだな。

881 名前:132人目の素数さん:05/03/10 22:07:28
>>871
それぞれの袋から1個ずつ取り出して、台秤に載せるんだ。
それから、よく目を凝らして大きさを見るんだ。
いちばん大きいのが 12g で、いちばん小さいのが 10g、
真ん中の大きさが 11g だ。

882 名前:132人目の素数さん:05/03/10 22:12:05
>>881
中に空洞があって、大きさと重さは比例しないかもな…

883 名前:881:05/03/10 23:04:58
>>882
しまった。

じゃ、順当に3つの袋 A, B, C から1個, 2個, 4個 を取り出し、
7個を秤に載せる。
74g なら、袋 A, B, C は、それぞれ 12g, 11g, 10g
75g なら、袋 A, B, C は、それぞれ 11g, 12g, 10g
76g なら、袋 A, B, C は、それぞれ 12g, 10g, 11g
78g なら、袋 A, B, C は、それぞれ 10g, 12g, 11g
79g なら、袋 A, B, C は、それぞれ 11g, 10g, 12g
80g なら、袋 A, B, C は、それぞれ 10g, 11g, 12g


884 名前:132人目の素数さん:05/03/10 23:46:33
もう一度問題を読んでみよう

885 名前:132人目の素数さん:05/03/11 12:11:53
基地外だらけ

886 名前:132人目の素数さん:05/03/11 15:58:50
最初の袋からは1個 → 10g、11g、12gのいずれか
次の袋からは10個 → 100g、110g、120gのいずれか
その次の袋からは100個 → 1000g、1100g、1200gのいずれか
以下同じように10倍ずつ取り出し量る。
合計の重量の下一桁に注目
これが0なら最初の袋は10g、1なら11g、2なら12g
最初の袋から取り出した分の重量を引き、下二桁目に注目
これが0なら2番目の袋は10g、1なら11g、2なら12g
二番目の袋から取り出した分の重量を引き、下三桁目に注目
以下同様。

取り出す個数をもっと少なくしたいなら、最初の袋からは1個
次からは3倍ずつにして、3進数で考えればよい。


887 名前:132人目の素数さん:05/03/11 16:03:48
>>875でもう出てんだけど解答

888 名前:132人目の素数さん:05/03/11 16:08:04
100倍ずつ取り出せば計算はもっと簡単になる

889 名前:132人目の素数さん:05/03/11 16:08:41
>>887
「とかゆうかんじ」を解答と認めるならな

890 名前:132人目の素数さん:05/03/11 16:43:18
0個1個3個でいいじゃん。

891 名前:132人目の素数さん:05/03/11 21:05:35
また問題読んでないやつか?

892 名前:132人目の素数さん:05/03/11 21:53:56
レベル下がったなこのスレ

893 名前:132人目の素数さん:05/03/11 22:02:31
なんだお前は?

894 名前:132人目の素数さん:05/03/11 22:31:28
さらしあげ

895 名前:132人目の素数さん:05/03/11 22:42:56
疑問に思ったこと。
自然数列{an}は、納i=1〜∞]1/ai が収束しているとする。A={ai|i∈N}と
するとき、N−Aには 無限項から成る等差数列が含まれると言えるだろうか。

言えない気がするのだが、分からない。

896 名前:132人目の素数さん:05/03/11 22:56:11
うわわかんね意外とむず

897 名前:BlackLightOfStar ◆ifsBJ/KedU :05/03/11 22:59:40
どんな自然数nでもn(n+1)/2と表せない自然数から無限項からなる等差数列を作れるのかどうか?

898 名前:BlackLightOfStar ◆ifsBJ/KedU :05/03/11 23:02:12
5n+4で終わりか。難しいな。

899 名前:132人目の素数さん:05/03/12 01:51:20
>>895
A={(n^2)*(2^m)|n,m∈N}なんかで反例になってない?

900 名前:899:05/03/12 02:17:08
>>899
ごめん、これじゃダメだ。
任意のn,m∈Nに対して、n+mk∈Aなるk∈Nが無限個あればN-Aは無限等差列を含まない。
だから、例えばk_(n,m,j)を
Σ[j=1 to ∞] (1/(n+m*k(n,m,j)))<1/((n^2)*(2^m))
となるように選んで(任意のn,mについてこれは可能)、
A={n+m*k(n,m,j)|n,m,j∈N}
とすりゃ反例。であってるかな?


901 名前:132人目の素数さん:05/03/12 03:48:11
>874 →あきれた。ここまでの見当違いは救いようがない。
>875 →>>889
>876 →一回だけ使って。>>877
>881 →>>882
>883 →10袋。>>884
>886 →>>887
>890 →0個…>>891

なかなかつわもの揃いの解答者陣であった。

902 名前:132人目の素数さん:05/03/12 11:25:30
>>901
>>888の評価がないぞ

903 名前:BlackLightOfStar ◆ifsBJ/KedU :05/03/12 18:22:45
Re:>901 一度のせた球は全ての測定が終わるまでのせたままにするんだよ。それでも一回ではないというのか?

904 名前:132人目の素数さん:05/03/13 01:06:13
はかりの目盛りを読むのが一回。
幾ら乗せても目盛りを読まずに下ろせば0回。

905 名前:895:05/03/13 01:56:18
>>900
すげー!k(n,m,j)の例としては、k(n,m,j)={(n^2)*2^(m+1)}^jが
あるな(計算ミスしてなければ)。ありがとう!すっきりしたよ。

906 名前:132人目の素数さん:05/03/13 03:43:56
辺の長さが3,5,7の透明な直方体がある。
底面(5×7)の中心からななめに直方体内部に向かって
(投影図でいうと正面からても側面から見ても真上から見ても
45°になる方向に)光線が出ている、光線は直方体の面に当たると
反射する(入射角=反射角)。
1)光線が元の点に戻ってくるまでに何回反射するでしょう
2)光線の軌跡の交点はいくつあるでしょう

光線や透明だと矛盾があるかもしれないが
ビリヤードのような反射の問題の立体版ということでよろしく

907 名前:132人目の素数さん:05/03/13 03:48:11
3)辺の長さA,B,C(A,B,Cは自然数、A≠B≠C)で
  1)2)を一般化できるでしょうか

908 名前:132人目の素数さん:05/03/13 10:10:06
平面版と同じ

909 名前:BlackLightOfStar ◆ifsBJ/KedU :05/03/13 18:29:49
Re:>904 じゃあはかりから目を離さなければいいのだな。

そもそも、何千個もおもりをのせるという方がナンセンスなんだよ。

910 名前:期末テスト:05/03/13 20:28:22
aを正の整数とする。放物線y=ax2(−1<=x=>1)上の動点Pとy軸上の
定点Q(0、q)との距離の二乗PQ2は、点Pが原点0に一致するとき最大である
このとき、次の問いに答えよ。
(1)qの最小値をaの式で表せ。
(2) (1)のとき、中心がQで原点を通る円のうち、半径が最小となる円の
方程式を求めよ。

911 名前:132人目の素数さん:05/03/13 20:55:02
>910
(1)三角形を考えたら、QY(+/-1)=QOぐらいじゃない?
(2)r=OQ,c=OQ/2ぐらい?

912 名前:132人目の素数さん:05/03/13 21:41:35
>>907
A = C でもいいのね?

913 名前:BlackLightOfStar ◆ifsBJ/KedU :05/03/15 20:44:00
Re:>895
とりあえず、数列を1,2,4,8,16と指数的に増大するようにして、さらに、
第1項は適当に数を足して2k(kは自然数,以下同様)となるようにし、
第2項は適当に数を足して2kになるようにし、
第3〜7項は適当に数を足して2k,2k+1,3k,3k+1,3k+2が全て現れるようにする。
第8〜16項は適当に数を足して2k,2k+1,3k,3k+1,3k+2,4k,4k+1,4k+2k4k+3が全て現れるようにする。
第17〜30項は適当に数を足して、…
と繰り返していけば反例になるのではないか?

914 名前:132人目の素数さん:05/03/17 04:27:41
>>792
> ・隣家には子供が2人いることがわかっている。性別はわかっていない。
> 聞こえてくる声から、一人は女の子だと分かった。
> このとき、隣家の子供が男女1人ずつである確率
>   A)1/2 B)2/3

こっちの答えは、結局なに?
1/2なの?

915 名前:132人目の素数さん:05/03/17 12:57:03
蒸し返すな。ログ読んで判断しる。

916 名前:132人目の素数さん:05/03/17 14:38:19
>>915
> ・隣家には子供が2人いることがわかっている。性別はわかっていない。
> 聞こえてくる声から、一人は女の子だと分かった。
> このとき、隣家の子供が男女1人ずつである確率
>   A)1/2 B)2/3

こっちの答えは、結局なに?
1/2なの?

917 名前:132人目の素数さん:05/03/17 23:22:00
>>914
この問題の条件だけからは答は定まらない。

まず2人の子供の組み合わせが全て等しいと仮定。
この段階では男男・男女・女男・女女のどれであるかは同じ確率。


さらに追加の条件として、2人の子供のうちどちらが話した声を聞いたとして、どちらが話すかを同じ確率と仮定する。
そうすると、女の子の声を聞いた場合は以下の4通り。
男・女の組み合わせで女が話した。
女・男の組み合わせで女が話した。
女・女の組み合わせで始めの女が話した。
女・女の組み合わせで後の女が話した。
この4つが同じ確率で起る。

また、2人の子供の両方とも話して、女の子の声のみが聞こえてくると仮定した場合は以下の3通り。
男・女の組み合わせで女の声が聞こえた。
女・男の組み合わせで女の声が聞こえた。
女・女の組み合わせで女の声が聞こえた。
この3つが同じ確率で起る。

それ以外の条件も考えられる。男と女の組み合わせの場合は話をしないが、男・男の組み合わせと女・女の組み合わせの場合は話をすると仮定する。
その場合は女・女の組み合わせで女の声が聞こえたという1通りになる。

つまり追加の条件を変える事によって、0から1までの任意の確率をとることができる。


918 名前:132人目の素数さん:05/03/18 03:00:56
>また、2人の子供の両方とも話して、女の子の声のみが聞こえてくると仮定した場合は以下の3通り。
>男・女の組み合わせで女の声が聞こえた。
>女・男の組み合わせで女の声が聞こえた。
>女・女の組み合わせで女の声が聞こえた。
>この3つが同じ確率で起る。

これはなんか変。

>それ以外の条件も考えられる。
>男と女の組み合わせの場合は話をしないが、
>男・男の組み合わせと女・女の組み合わせの場合は話をすると仮定する。
>その場合は女・女の組み合わせで女の声が聞こえたという1通りになる。

これは追加条件つけすぎ。言わんとするところは分かるが。

>つまり追加の条件を変える事によって、0から1までの任意の確率をとることができる

まあその通り。そもそも男女の存在確率や兄弟の性別の相関には言及されて
ないわけだから、追加条件でどうとでもなる(w

919 名前:132人目の素数さん:2005/03/26(土) 23:32:45
3次元空間にn(>=3)個の点がありこの中から任意の3点を
選ぶと二等辺三角形になる。nの最大値を求めよ。


920 名前:132人目の素数さん:2005/03/27(日) 00:22:21
>>919
宿題ぽいなそれ


921 名前:BlackLightOfStar ◆FGVY03TaBA :2005/03/27(日) 01:32:20
>>913
人の名前真似すんな

922 名前:132人目の素数さん:2005/03/27(日) 04:48:08
>>914
それぞれの事象を次のように設定し条件付確率で求める。
全事象:2人の性別。4通り
事象A:女がいる。3通り
事象B:男がいる。3通り
事象(A∩B):男1人女1人である。2通り

問いの、1人が女と分かっているときもう1人が男という事象は(B|A)で表せる。
それぞれの確率P(A)=3/4、P(B)=3/4、P(A∩B)=1/2を用いて、P(B|A)を導くと答え。
P(B|A)=P(A∩B)/P(A)
P(B|A)=(1/2)/(3/4)
∴P(B|A)=2/3

答えは2/3とみた――――!!(^▽^)y-゜゜゜

923 名前:132人目の素数さん:2005/03/27(日) 04:58:24
おもしろくねーな。

924 名前:BlackLightOfStar ◆ifsBJ/KedU :2005/03/27(日) 06:44:18
Re:>921 お前誰だよ?

925 名前:132人目の素数さん:2005/03/27(日) 12:19:39
>>919
7点はいけた。8点でできるのかな‥

926 名前:132人目の素数さん:2005/03/27(日) 13:50:43
>>922
ログ嫁

927 名前:132人目の素数さん:2005/03/27(日) 18:49:09
>>926
カス野郎!

928 名前:132人目の素数さん:2005/03/27(日) 22:35:34
922だが、何かこのスレでしてはならん事をしたようだ。>>926、何かはわからんがスマン‥
>>927
あともうちょっとで1000だから楽しく行こう。お願いm(_ _)m

929 名前:132人目の素数さん:2005/03/27(日) 22:42:36
殺伐がデフォなので、大丈夫です。

930 名前:132人目の素数さん:2005/03/27(日) 23:01:41
>>928
ここは狂人の世界だから気にするな。

931 名前:132人目の素数さん:2005/03/28(月) 00:33:55
>>929 >>930そ、そうなのか。この世界ではそれが楽しいのか‥知らんかった。ありがと
ちと私には荷が重いようだ。修業してくる。

932 名前:132人目の素数さん:2005/03/31(木) 23:17:50
>>919
解答マダー?  チンチン

933 名前:132人目の素数さん:2005/03/31(木) 23:35:26
自分で考えた問題で、答えにあんまり自信がないんですが・・・

dは実数とする。
点(d , 0)と点(0 , 6-d)を通る直線を、(0≦d≦6)を満たすすべてのdについて引いたとする。
そのときできる曲線?の式を求めよ。

言葉で説明できないんで図を描きました。
赤で描いたのが、おおまかですが、求めたい曲線です。
http://www.geocities.jp/akira2403/qu.png

934 名前:132人目の素数さん:2005/03/31(木) 23:40:20
方物線だろうな。微分方程式たてればよろし。

935 名前:132人目の素数さん:2005/03/31(木) 23:47:21
ウソ吐くな

936 名前:132人目の素数さん:2005/03/31(木) 23:51:06
age

937 名前:132人目の素数さん:皇紀2665/04/01(金) 00:49:37
>>933
直線の式 x/d + y/(6-d) = 1 を満たすdが取れるような
(x,y)の範囲を考えたとき、その境界が求める曲線になる。
上の式を展開整理すると d^2+(y-x-6)d+6x = 0
dの実数条件から (y-x-6)^2-24x≧0
求める曲線はこの不等号を等号で置き換えたもの。

938 名前:132人目の素数さん:皇紀2665/04/01(金) 08:54:25
面白い問題
(x-a)(x-b)(x-c)・・・・・(x-z)=

939 名前:132人目の素数さん:皇紀2665/04/01(金) 09:13:51
プッ

940 名前:132人目の素数さん:皇紀2665/04/01(金) 10:04:01
>>935
ほ〜ら苦戦。ちゃんと解けやw。

941 名前:132人目の素数さん:皇紀2665/04/01(金) 12:09:37
>>937
正解です

つまらない問題でしたね(ノ∀`)

942 名前:BlackLightOfStar ◆ifsBJ/KedU :皇紀2665/04/01(金) 13:45:06
Re:>934 「方物線」ってどうやって書いた?

943 名前:132人目の素数さん:2005/04/03(日) 13:06:03
簡単な問題だが、結果に感動した。

(x^2^n)−1=(x−1)Π[i=0〜n−1](1+x^2^i) であることを示し、これを
用いて納i=0〜(2^n)−1]x^i=Π[i=0〜n−1](1+x^2^i)であることを示し、
さらに、この式の右辺を展開して左辺と比較することで 任意の自然数mが
m=納i=0〜∞](2^i)*ai (ai=0 or 1 (∀i∈N∪{0}),ほとんど全てのiに
対してai=0) という形に一意的に表せることを示せ。

944 名前:132人目の素数さん:2005/04/04(月) 05:43:52
age

945 名前:132人目の素数さん:2005/04/04(月) 21:07:24
周長の等しい閉曲線の内、面積最大のものは円である事を証明せよ。

946 名前:132人目の素数さん:2005/04/04(月) 23:12:44
有名問題

947 名前:132人目の素数さん:2005/04/05(火) 01:03:54
>>945
微分幾何の宿題を貼るな。


948 名前:132人目の素数さん:2005/04/20(水) 03:28:32
あげ

949 名前:132人目の素数さん:2005/04/20(水) 16:20:17
12枚のコインの中に1枚だけ偽物が混じってます。
偽物のコインは重いか軽いか判りません。
天秤を使って最低、〈何回で〉…? 又〈どうやって〉…?

950 名前:132人目の素数さん:2005/04/20(水) 16:54:34
>>949
13枚の場合の3回ではできない証明がおもしろかった

951 名前:132人目の素数さん:2005/04/20(水) 22:36:49
>>950
できるんじゃないか? 方法は12枚の場合と同じ。偽物がどれかはわかるけど、重いか軽いかはわからない場合がある。


952 名前:132人目の素数さん:2005/04/21(木) 01:28:17
>>951
その重いか軽いか特定できないのは「できない」とする証明だった。

953 名前:132人目の素数さん:2005/04/23(土) 02:27:16
>>949
http://www.geocities.co.jp/CollegeLife-Club/7442/math/index.html

954 名前:132人目の素数さん:2005/04/26(火) 01:05:14
age

955 名前:132人目の素数さん:2005/05/04(水) 14:49:36
http://www.geocities.co.jp/CollegeLife-Club/7442/math/index.html#coin2

956 名前:132人目の素数さん:2005/05/04(水) 18:52:45
(=゚ω゚)ノ ていっ

問1【現代数学は、いわゆる「不可能性の証明」にみられるように経験的・実証的な議論の蓄積の上に成立している。】
問2【数学の根本理念は不変であり、古代の数学も現代の数学も、その証明観に何の変わりもない。】

それぞれ正か否か具体例をあげて証明せよ!


( ´,_ゝ`) 数学というより哲学の問題。できるネ申はいるかな、フフフ♪

957 名前:BlackLightOfStar ◆vkn9fRJn.s :2005/05/05(木) 23:49:14
さーて久しぶりに暴れることにしてみるか

958 名前:132人目の素数さん:2005/05/05(木) 23:50:52
暴れろバカ

959 名前:132人目の素数さん:2005/05/05(木) 23:55:17
>>957
おまえ誰だよ

960 名前:BlackLightOfStar ◆vkn9fRJn.s :2005/05/06(金) 00:03:55
>>959
貴様に聞かれる筋合いもなければ答える義務もない。
だいたい、名前欄にBlackLightOfStar ◆vkn9fRJn.sと書いただろうが。

>>958
従ふ

961 名前:BlackLightOfStar ◆ifsBJ/KedU :2005/05/06(金) 07:25:22
Re:>957,960 お前誰だよ?

962 名前:132人目の素数さん:2005/05/06(金) 16:59:38
自然数の集合NをS1〜Smに分割したら、どれもx+y=zを満たすようなx,y,zのペアを含まなかった。
そんな風に分割出来る自然数mは存在するか?

963 名前:132人目の素数さん:2005/05/06(金) 17:17:34
直角三角形がある。
斜辺の長さは1cmで一番短い辺の長さはxcm
それより長いもう一辺の長さはycmとする。

それぞれの比が
y:x-y = x:1-x = 1:x
である。

yが1cmの時のその直角三角形の面積を求めよ。

964 名前:132人目の素数さん:2005/05/06(金) 17:32:25
x^2+x-1=0、y=x^2を解けばx,yが出る。
面積は(1*x/y)/2

965 名前:132人目の素数さん:2005/05/06(金) 19:24:26
>>962
できる。
証明)
十分大きな自然数mを持ってきて、
分割後の集合に要素が3つ以上含まないように分割する。
この場合、当然ながらどれもx+y=zを満たすようなx,y,zのペアを含まない。
Q.E.D.

966 名前:132人目の素数さん:2005/05/06(金) 19:30:07
>>965
自然数の集合Nは全ての自然数の集合Nで
だから全部有限有限になるようには分割出来ないと思って下され。

967 名前:132人目の素数さん:2005/05/06(金) 19:30:36
有限有限→有限集合

968 名前:132人目の素数さん:2005/05/07(土) 21:38:30
>>963
斜辺と他の一辺の長さが等しくて
直角三角形になるのか?


969 名前:132人目の素数さん:2005/05/07(土) 21:53:30
>>963
何かおかしい気がする

970 名前:132人目の素数さん:2005/05/08(日) 02:44:05
>>962
難しい。
わからん。
誰か答え出してくれ。

971 名前:132人目の素数さん:2005/05/10(火) 00:00:02
>>962
6人の人間がいれば、その中のある3人は、互いに仲良しであるか、互いに中が悪いことを示せ、という
問題を解いたことがあると楽(?)
X⊂Nとする。A(X)={(i,j)∈X^2|i<j},Bn={1,2,…,n}とおく。次の*を示す。
任意のm∈N及び任意の無限集合X⊂N及び任意のf:A(X)→Bm に対して、f(i,j)=f(j,k)=f(i,k)を満たす
3つの自然数i<j<k (i,j,k∈X)が存在する。…*
証明)数学的帰納法で示す。m=1のときは明らか。m=k (k≧1)のとき成り立つとすると、m=k+1のとき…任意の
無限集合X⊂N及び任意のf:A(X)→Bmを取る。Xの最小元をaとする。任意のi∈X−{a}に対してf(a,i)∈Bm={1,2,…,m}
なので、Xt={i∈X−{a}|f(a,i)=t}とおけばX−{a}=X1∪X2∪…∪Xm と分割できる。これとX−{a}が無限集合である
ことから、あるp(1≦p≦m)が存在してXpは無限集合となる。p=mとしてよい。任意にi,j∈Xm (i<j)をとる。
もしf(i,j)=mだとするとf(a,i)=f(a,j)=f(i,j)=mとなるから、このa<i<j (i,j,a∈X)が求める3つの自然数で
ある。よって、常にf(i,j)≠mだとしてよい。このときf(i,j)∈B(m−1)=Bkとなる。…(1)また、Xm⊂XだからA(Xm)⊂A(X)
である。そこで、fの定義域をA(X)からA(Xm)に制限した写像をgとするとgは(1)よりg:A(Xm)→Bkであるから、これ及びXmは
無限集合及びm=kのときの仮定からg(x,y)=g(y,z)=g(x,z)を満たすx<y<z(x,y,z∈Xm)が存在する。…(2)gはfの
制限写像であり、Xm⊂Xであることから(2)は「f(x,y)=f(y,z)=f(x,z)を満たすx<y<z(x,y,z∈X)が存在する」と
言い換えられる。よってm=k+1のときも成立し、*は成り立つ。
>>962の答え…出来ない。
証明)題意を満たす分割N=N1∪N2∪…∪Nmが存在したとする。X=N (当然Xは無限集合)とおき、f:A(X)→Bm を
f(i,j)=t (j−i∈Ntのとき)と定義する。このとき、*からf(i,j)=f(j,k)=f(i,k)…(3)を満たす3つの自然数
i<j<k(i,j,k∈X)が存在することになる。f(i,j)=sとする。x=k−j,y=j−i,z=k−iとおくと、(3)から
x,y,z∈Nsであり、さらにx+y=k−j+j−i=k−i=z なので、矛盾。よって、題意を満たす分割は出来ない。

972 名前:132人目の素数さん:2005/05/11(水) 09:41:16
グラフ理論の問題だな

973 名前:132人目の素数さん:2005/05/19(木) 22:10:26
結局>>919の答えはいくつなんだべさ

974 名前:132人目の素数さん:2005/05/19(木) 22:32:48
>>973
お前には教えてやらん!

975 名前:132人目の素数さん:2005/05/20(金) 00:00:12
>>962-972
一応ここでガイシュツ
ttp://science3.2ch.net/test/read.cgi/math/1106654316/93-116

976 名前:132人目の素数さん:2005/05/23(月) 20:42:50
>>973
考え方教えたら、ホントに数えるか?
数えるんだったら教えてやらん事もないが。

977 名前:132人目の素数さん:2005/05/25(水) 18:48:18
△ABCはAB=ACたる二等辺三角形である。
Cから辺ABに垂線を下し,その足をMとし,Mから辺BCに垂線を下し,その足をNとする。
MN=3,AN=4のとき,ABの長さはいくらか。

978 名前:132人目の素数さん:2005/05/25(水) 20:07:17
アゲス

979 名前:132人目の素数さん:2005/05/25(水) 21:07:04
>>972 ご名答。

980 名前:132人目の素数さん:2005/05/26(木) 21:01:54
>>976
>考え方
別に2chなんてそれ示すだけで十分じゃん

981 名前:132人目の素数さん:2005/05/27(金) 22:19:51
>>980
面白くも何ともない問題を、解けないという奴に対しては、
少し遊びも良いんじゃないのかな。

>>980
まさかおまえさんも解けてなかったりして?

982 名前:132人目の素数さん:2005/05/27(金) 22:29:52
このスレも、もう終わりかなぁ… (AAいる?)

983 名前:132人目の素数さん:2005/05/28(土) 02:30:17
>>981
何か捻くれてるな。嫌な事でもあったのか?
でも変な挑発をここでするのは止してくれんかね。

984 名前:132人目の素数さん:2005/05/28(土) 15:02:01
>>977を解いたかたいますか?

985 名前:Mozilla in X11:2005/05/28(土) 15:11:42
>>984
とりあえず、AM を求めよう。そこから何かが始まるような気がする。

986 名前:132人目の素数さん:2005/05/28(土) 15:55:30
二百七十三日。


987 名前:132人目の素数さん:2005/05/29(日) 10:14:55
め埋

988 名前:132人目の素数さん:2005/05/29(日) 15:55:30
二百七十四日。


989 名前:132人目の素数さん:2005/05/30(月) 10:25:51
(゚Д゚≡゚Д゚)?

990 名前:132人目の素数さん:2005/05/30(月) 15:18:52
だれか>>919の答えおしえてたも。n=7ぐらいじゃないかとはおもうんだけど。

991 名前:132人目の素数さん:2005/05/30(月) 15:43:21
>>990
だが断る!
この岸部露伴がもっとも好きな事のひとつは 
答えを知りたいと思っているやつに『NO』と断ってやることだ・・・
             ___,,,,,..... -一ァ
         / ̄;;;´;;、;;;ヾ;;;, -──--、,!
.        /'´|;;;;,、;;;;;;;;;;/      ,!
.         /:.:.:.レ´:.ヾ;;;;;;i   断  だ ,!
       /:.:.:.:.:.:.:.:.:.:.:ヾ;i  る  が ,!
.      /:.;.イ:.:.:.:.:.:.:.:.:.:..ヽ       ,!
.       /レ' ;|:.:.:.:.:.:.:,:ィ:.:.:.:〉 __,.,!
     /-、ヽ,:|:.:.:,/ /:.:.://.:,:ィ:.:.:.,!
      /'ヽ、ヾi ゙´.:   /__;:;:-'"´ ,;|:.:.:.,!
.    /ゝ-`';:/ .:〈ニ=-=ニ二 ̄ヽレ',!
   /::::;;;;;/  ' ,, ニ`ー-,、__\〉ィ,!
.   /;:::::/ ::.    ::.,,\_ゞ;'> 〈;,!
  /i!:::::iヾ-'、::..       '';~ ,;:'/,!
. /;;;i!fi´l_、,.`        .: ,;:'  ,!
/;;;;;i' ('ー、ヽ      ..: ,;:''   ,!
ヽ、jゝ、`ヾ:、゙、   ,..:'.:'"    .: ,!
   ``ヽ.、_ ¨`  ,:'      (_r:,!
       ``ヽ.、..    ノr;ソ~,!
             ``ヾ、 / 7,!
                 ``ヽ,!

992 名前:132人目の素数さん:2005/05/30(月) 16:00:24
二百七十六日。

993 名前:132人目の素数さん:2005/05/30(月) 18:30:08


994 名前:132人目の素数さん:2005/05/30(月) 18:30:44


995 名前:竜太:2005/05/30(月) 23:33:32
関数f(x)のxにnを代入するとn番目の素数を出力するような関数f(x)を作りたまえ。
なお、この関数は式で表すことが条件である。(用いていい記号は、√,Σ,Π,lim,sin...など)

俺も作ったんだけど俺以外にも他に作れる人いるかなあと思って。
結構むずいかなぁ


996 名前:竜太:2005/05/30(月) 23:35:45
実際俺この関数作るのに一時間近くかかった。

997 名前:132人目の素数さん:2005/05/31(火) 00:50:55
>>995
>(用いていい記号は、√,Σ,Π,lim,sin...など)
 
などって・・・つかっていいシンボルをキチンと規程しないと問題にならん。


998 名前:竜太:2005/05/31(火) 00:52:05
アホが1匹釣れたw

999 名前:132人目の素数さん:2005/05/31(火) 01:38:13
n番目の素数を出力するアルゴリズムならあるよ

1000 名前: ◆27Tn7FHaVY :2005/05/31(火) 01:38:45
1000!

1001 名前:1001:Over 1000 Thread
このスレッドは1000を超えました。
もう書けないので、新しいスレッドを立ててくださいです。。。


全部 最新50
DAT2HTML 0.34a Converted.